initial version
authormdw <mdw>
Tue, 2 Oct 2001 19:06:59 +0000 (19:06 +0000)
committermdw <mdw>
Tue, 2 Oct 2001 19:06:59 +0000 (19:06 +0000)
15 files changed:
.cvsignore [new file with mode: 0644]
Makefile [new file with mode: 0644]
auth-mac.tex [new file with mode: 0644]
auth-sig.tex [new file with mode: 0644]
basics.tex [new file with mode: 0644]
chan.tex [new file with mode: 0644]
enc-ies.tex [new file with mode: 0644]
enc-intro.tex [new file with mode: 0644]
enc-pub.tex [new file with mode: 0644]
enc-symm.tex [new file with mode: 0644]
ips.sty [new file with mode: 0644]
ips.tex [new file with mode: 0644]
mkimages.pl [new file with mode: 0644]
slides.tex [new file with mode: 0644]
zk.tex [new file with mode: 0644]

diff --git a/.cvsignore b/.cvsignore
new file mode 100644 (file)
index 0000000..0382342
--- /dev/null
@@ -0,0 +1,2 @@
+slides
+ips.ps slides.ps ips.pdf slides.pdf ips.dvi slides.dvi ips.log auth-mac.aux auth-sig.aux basics.aux enc-ies.aux enc-intro.aux enc-pub.aux enc-symm.aux ips.aux ips.ans ips.toc ips.bbl ips.blg
diff --git a/Makefile b/Makefile
new file mode 100644 (file)
index 0000000..8630a4d
--- /dev/null
+++ b/Makefile
@@ -0,0 +1,38 @@
+# Makefile for IPS
+
+SOURCES = \
+       ips.tex slides.tex ips.sty \
+       basics.tex \
+       auth-mac.tex auth-sig.tex \
+       enc-intro.tex enc-pub.tex enc-symm.tex enc-ies.tex
+
+all: ips.dvi ips.ps ips.pdf slides.dvi slides.ps slides.pdf
+
+ips.dvi: $(SOURCES)
+       latex ips && bibtex ips && \
+       latex ips && latex ips
+ips.pdf: ips.dvi
+       pdflatex ips.tex && cp ips.pdf ..
+ips.ps: ips.dvi
+       dvips -o ips.ps ips.dvi
+
+slides.dvi: $(SOURCES)
+       @if [ ! -d slides ]; then \
+         mkdir slides; \
+         for i in $(SOURCES); do ln -s ../$$i slides; done; \
+       fi
+       cd slides && \
+       latex slides && bibtex slides && \
+       latex slides && latex slides && \
+       cp slides.dvi ..
+slides.pdf: slides.dvi
+       cd slides && pdflatex slides.tex && cp slides.pdf ..
+slides.ps: slides.dvi
+       dvips -o slides.ps slides.dvi
+
+clean:
+       rm -f ips.dvi ips.ps ips.pdf slides.dvi slides.ps slides.pdf
+       rm -f *.log *.bbl *.blg *.toc *.ans
+       rm -rf slides ips
+
+.PHONY: clean
diff --git a/auth-mac.tex b/auth-mac.tex
new file mode 100644 (file)
index 0000000..40f466d
--- /dev/null
@@ -0,0 +1,962 @@
+\xcalways\section{Message authentication codes}\x
+
+\xcalways\subsection{Definitions and security notions}\x
+
+\begin{slide}
+  \head{Definition of a MAC}
+
+  A MAC is a pair of algorithms $\mathcal{M} = (T, V)$:
+  \begin{itemize}
+  \item The \emph{tagging} algorithm $T\colon \{0, 1\}^k \times \{0, 1\}^*
+    \to \{0, 1\}^L$ is a probabilistic algorithm which, given a key and a
+    string, returns a \emph{tag}.  We write $\tau \in T_K(m)$.
+  \item The \emph{verification} algorithm $V\colon \{0, 1\}^k \times \{0,
+    1\}^* \times \{0, 1\}^L \to \{0, 1\}$ is a deterministic algorithm which,
+    given a key, a message and a tag, returns $1$ if the tag is valid, or $0$
+    otherwise; i.e., we require that $V_K(m, \tau) = 1 \iff \tau \in T_K(m)$.
+  \end{itemize}
+  The basic idea is that it's hard for an adversary to \emph{forge} a tag for
+  a message it's not seen before.
+\end{slide}
+
+\begin{slide}
+  \topic{informal security notion}
+  \head{Strong MACs, 1: informal security notion}
+  
+  Our standard notion of security for MACs is \emph{strong unforgeability
+    against chosen message attack}, or SUF-CMA, for short
+  \cite{Abdalla:2001:DHIES, Bellare:2000:AER}.  Let $A$ be an adversary which
+  is attempting to attack the MAC $\mathcal{M} = (T, V)$.
+  
+  We play a game with the adversary.  We invent a key $K \inr \{0, 1\}^k$.
+  The adversary \emph{wins} if, after requesting tags for some messages of
+  its choice, and checking some guesses, it can return a pair $(m, \tau)$
+  such that:
+  \begin{itemize}
+  \item the tag is correct, i.e., $V_K(m, \tau) = 1$; and
+  \item the tag is not one returned by the adversary's tagging oracle for
+    that message.
+  \end{itemize}
+\end{slide}
+
+\begin{slide}
+  \topic{strong MACs}
+  \head{Strong MACs, 2: the experiment}
+
+  We perform the following experiment with the adversary.
+  \begin{program}
+    Experiment $\Expt{suf-cma}{\mathcal{M}}(A)$: \+ \\
+      $K \getsr \{0, 1\}^k$; \\
+      $\Xid{T}{list} \gets \emptyset$; \\
+      $(m, \tau) \gets A^{\id{tag}(\cdot), V_K(\cdot, \cdot)}$; \\
+      \IF $V_K(m, \tau) \land (m, \tau) \notin \Xid{T}{list}$
+      \THEN \RETURN $1$; \\
+      \ELSE \RETURN $0$; \- \\[\smallskipamount]
+    Oracle $\id{tag}(m)$: \+ \\
+      $\tau \gets T_K(m)$; \\
+      $\Xid{T}{list} \gets \Xid{T}{list} \cup \{(m, \tau)\}$; \\
+      \RETURN $\tau$;
+  \end{program}
+\end{slide}
+
+\begin{slide}
+  \head{Strong MACs, 3: wrapping up the notation}
+
+  The \emph{success probability} of an adversary $A$ against the MAC
+  $\mathcal{M}$ in the sense of SUF-CMA is
+  \[ \Succ{suf-cma}{\mathcal{M}}(A) =
+       \Pr[\Expt{suf-cma}{\mathcal{M}}(A) = 1]. \]%
+  The \emph{insecurity} of a MAC $\mathcal{M}$ in the SUF-CMA sense is then
+  \[ \InSec{suf-cma}(\mathcal{M}; t, q_T, q_V) =
+       \max_A \Succ{suf-cma}{\mathcal{M}}(A) \]%
+  where the maximum is taken over all adversaries $A$ running in time $t$ and
+  making $q_T$ tagging and $q_V$ verification queries.
+  
+  If $\InSec{suf-cma}(\mathcal{M}; t, q_T, q_V) \le \epsilon$ then we say
+  that $\mathcal{M}$ is a \emph{$(t, q_T, q_V, \epsilon)$-secure MAC in the
+  SUF-CMA sense}.
+\end{slide}
+
+\begin{slide}
+  \topic{other notions}
+  \head{Other security notions for MACs}
+
+  There are a number of weaker security notions in use:
+  \begin{itemize}
+  \item The definition of a \emph{weak MAC} restricts the adversary from
+    returning any message with which it queried its tagging oracle.  The
+    strong MAC definition considers this OK, as long as the tag is different
+    from any returned by the oracle for that message.
+  \item Some definitions of MACs don't equip the adversary with a
+    verification oracle.  Our definition considers these to be $(t, q_T, 0,
+    \epsilon)$-secure.
+  \item You can have a MAC with a bounded domain $\{0, 1\}^L$ rather than
+    $\{0, 1\}^*$ as shown previously.
+  \item Further quantification is possible, e.g., counting the total number
+    of bytes queried, or the maximum size of a tagging query.
+  \end{itemize}
+\end{slide}
+
+%% IFF systems
+\begin{exercise}
+  %% A nice open-ended question.  Need to sort out IFF details (e.g.,
+  %% aircraft numbering or whatever.
+  An \emph{identify friend-or-foe} (IFF) system works as follows.  Each
+  `friendly' aircraft knows a common secret key.  When a new aircraft is
+  detected, a \emph{challenge} $x$ is sent.  If a correct \emph{response} is
+  returned, the new aircraft is presumed friendly; otherwise it is attacked.
+  Discuss the security requirements of IFF systems and construct a formal
+  model.  Decide which primitive is best suited to the job and relate the
+  security notions.  What sorts of resource constraints can be imposed on
+  adversaries attacking an IFF system?
+  \answer%
+  No prizes for guessing that a MAC is what's wanted, this being the MAC
+  section.  
+\end{exercise}
+
+\xcalways\subsection{Basic results}\x
+
+\begin{slide}
+  \topic{PRFs are MACs}
+  \head{PRFs are MACs, 1}
+  
+  If $F_K\colon \{0, 1\}^* \to \{0, 1\}^L$ is a $(t, q, \epsilon)$-secure
+  PRF, then it's also a $(t', q_T, q_V, \epsilon')$-secure MAC, with $q = q_T
+  + q_V + 1$, $t = t' + O(q)$, and $\epsilon \le \epsilon' + (q_V + 1)
+  2^{-L}$.  The constant hidden by the $O(\cdot)$ is small and depends on the
+  model of computation.
+  
+  Suppose $A$ can break $F$ used as a MAC in time $t$ and with $q_T$ and
+  $q_V$ queries to its tagging and verification oracles respectively.
+
+  If we can construct an adversary which distinguishes $F_K$ from a random
+  function using $A$ as an essential component, then we can prove the
+  result.
+\end{slide}
+
+\begin{slide}
+  \head{PRFs are MACs, 2: the distinguisher}
+
+  \begin{program}
+    Distinguisher $D^{F(\cdot)}$: \+ \\
+      $\Xid{T}{list} \gets \emptyset$; \\
+      $(m, \tau) \gets A^{T_F(\cdot), V_F(\cdot, \cdot)}$; \\
+      \IF $m \notin \Xid{T}{list} \land \tau = F(m)$
+      \THEN \RETURN $1$; \\
+      \ELSE \RETURN $0$; \- \\[\smallskipamount]
+    Oracle $T_F(m)$: \+ \\
+      $\Xid{T}{list} \gets \Xid{T}{list} \cup \{m\}$; \\
+      \RETURN $F(m)$; \- \\[\smallskipamount]
+    Oracle $V_F(m, \tau)$: \+ \\
+      \IF $\tau = F(m)$ \THEN \RETURN $1$; \\
+      \ELSE \RETURN $0$;
+  \end{program}
+\end{slide}
+
+\begin{slide}
+  \head{PRFs are MACs, 3: wrapping up}
+
+  The distinguisher simulates the tagging and verification oracles for the
+  MAC forger, using its supplied oracle.  If the forger succeeds, then the
+  distinguisher returns 1; otherwise it returns zero.
+
+  The probability that the distinguisher returns 1 given an instance $F_K$ of
+  the PRF is precisely $\Succ{suf-cma}{F}(A)$.
+  
+  The probability that it returns 0 given a random function depends on what
+  $A$ does when it's given a random function.  But we know that the
+  probability of it successfully guessing the MAC for a message for which it
+  didn't query $T$ can be at most $(q_V + 1) 2^{-L}$.  So
+  \[ \Adv{prf}{F}(D) \le \Succ{suf-cma}{F}(A) - (q_V + 1) 2^{-L}. \]
+  Let $q = q_T + q_V + 1$; then counting, rearranging, maximizing yields
+  \[ \InSec{suf-cma}(F; t, q_T, q_V) \le
+     \InSec{prf}(F; t + O(q), q) + (q_V + 1)2^{-L}. \]%
+\end{slide}
+
+\begin{exercise}
+  \begin{parenum}
+  \item Suppose that $F\colon \{0, 1\}^k \times \{0, 1\}^* \to \{0, 1\}^L$ is
+    a $(t, q, \epsilon)$-secure PRF.  Let $T^{(\ell)}_K(x)$ be the leftmost
+    $\ell$~bits of $F_K(x)$.  Demonstrate the security of $T^{(\ell)}(\cdot)$
+    as a MAC.
+  \item Discuss the merits of truncating MAC tags in practical situations.
+  \end{parenum}
+  \answer%
+  \begin{parenum}
+  \item The follows exactly the same pattern as the `PRFs are MACs' proof in
+    the slides: $T^{(\ell)}$ is a $(t, q_T, q_V, \epsilon + (q_V +
+    1)2^{-\ell})$-secure MAC, where $q_T + q_V = q$.
+  \item Obviously, truncating tags saves bandwidth.  There is a trade-off
+    between tag size and security, as the $2^{-\ell}$ term shows.  Note that
+    this term represents the probability of the adversary guessing the
+    correct tag when it's actually attacking a random function, and
+    therefore, when this occurs, the adversary has one `by accident'.
+    Including sequence numbers in packets ensures that replay of accidental
+    forgery (or honest messages) will be detected.  Hence, for some
+    applications, setting $\ell = 32$ or even lower is of no particular harm.
+    Perhaps more significantly, if the PRF isn't actually as good as it ought
+    to be, and (say) leaks key material very slowly, then truncating its
+    output can actually improve security.
+  \end{parenum}
+\end{exercise}
+
+\begin{exercise}
+  A traditional MAC construction is the \emph{CBC-MAC}: it works like this.
+  Suppose $F\colon \{0, 1\}^k \times \{0, 1\}^l \to \{0, 1\}^l$ is a PRF.
+  Split a message~$x$ into $l$-bit blocks $x_0, x_1, \ldots, x_{n-1}$
+  (applying some sort of padding if you need to).  Then we define the CBC-MAC
+  as $F^{(n)}_K(x)$, where
+  \[ F^{(1)}_K(x) = F_K(x);
+     \qquad F^{(i+i)}(x) = F_K(x_i \xor F^{(i)}_K(x)). \]%
+  In \cite{Bellare:1994:SCB}, Mihir Bellare, Joe Kilian and Phil Rogaway
+  introduced the world to the concrete-security approach and, almost
+  incidentally, proved that the CBC-MAC is a PRF (and therefore a MAC) for
+  any \emph{fixed sized} input.
+
+  Show that the CBC-MAC is easily broken if you're allowed to choose messages
+  of different lengths.
+  \answer%
+  Request tags $\tau$ for the message $x = x_0, x_1, \ldots, x_{n-1}$ and
+  $\tau'$ for $x' = x'_0 \xor \tau, x'_1, \ldots, x'_{n'-1}$.  Let $y = x_0,
+  x_1, \ldots, x_{n-1}, x'_0 , x'_1, \ldots, x'_{n'-1}$.  Note that
+  $F^{(n)}_K(y) = \tau$, and $F^{(n+1)}_K(y) = F_K(x'_0 \xor F^{(n)}_K(x)) =
+  F^{(1)}_K(x')$.  Hence, $F^{(n+n')}_K(y) = \tau'$, and we have a correct
+  forgery.
+\end{exercise}
+
+\begin{slide}
+  \topic{verification oracles}
+  \head{Verification oracles}
+  
+  We can leave verification oracles out of our analysis.  This simplifies
+  analysis, but produces slightly less satisfactory quantitative results.
+  
+  Suppose that $\mathcal{M} = (T, V)$ is a $(t, q_T, 0, \epsilon)$-secure
+  MAC.  Then, for any $q_V$,
+  \[ \InSec{suf-cma}(\mathcal{M}; t, q_T, q_V) \le
+     (q_V + 1)\InSec{suf-cma}(\mathcal{M}; t, q_T, 0). \]%
+  This bound is \emph{tight}: it's not possible for a general result like
+  this to do better.
+\end{slide}
+
+\begin{proof}
+  Consider an adversary $A$ which uses $q_V$ verification queries.  We assume
+  the following properties of $A$'s behaviour:
+  \begin{itemize}
+  \item No verification query contains a message and a tag for that message
+    received from the tagging oracle.
+  \item If a verification query succeeds, the message is not given in a query
+    to the tagging oracle.
+  \item Once a verification query succeeds, all subsequent verification
+    queries also succeed and the adversary returns a correct forgery (e.g.,
+    by simply repeating the succeessful query).
+  \end{itemize}
+  It's clear that any adversary can be transformed into one which has these
+  properties and succeeds with probability at least as high.
+  
+  Let $V$ be the event that at least one verification query succeeds, and let
+  $S$ be the event that $A$ succeeds.  Then
+  \begin{eqnarray*}[rl]
+    \Succ{suf-cma}{\mathcal{M}}(A)
+      &= \Pr[S \mid V] \Pr[V] + \Pr[S \mid \lnot V] \Pr[\lnot V] \\
+      &= \Pr[V] + \Pr[S \mid \lnot V] \Pr[\lnot V].
+  \end{eqnarray*}
+  Now consider these two adversaries:
+  \begin{program}
+    Adversary $A'^{T(\cdot), V(\cdot, \cdot)}$: \+ \\
+      $i \gets 0$; \\
+      $(m, \tau) \gets A^{T(\cdot), \Xid{V}{sim}(\cdot, \cdot)}$; \\
+      \RETURN $(m^*, \tau^*)$; \- \\[\smallskipamount]
+    Oracle $\Xid{V}{sim}(m, \tau)$: \+ \\
+      $i \gets i + 1$; \\
+      \IF $i < q_V$ \THEN \RETURN $V(m, \tau)$; \\
+      $(m^*, \tau^*) \gets (m, \tau)$; \\
+      \RETURN $1$;
+  \next
+    Adversary $Z^{T(\cdot), V(\cdot, \cdot)}$: \+ \\
+      \\
+      $(m, \tau) \gets A^{T(\cdot), \Xid{V}{zero}(\cdot, \cdot)}$; \\
+      \RETURN $(m, \tau)$; \- \\[\smallskipamount]
+    Oracle $\Xid{V}{zero}(m, \tau)$: \+ \\
+      \RETURN $0$;
+  \end{program}
+  The adversary $A'$ uses $q_V - 1$ verification queries.  It ignores the
+  output of $A$, returning instead $A$'s $q_V$-th verification query.  Thus,
+  by our assumptions on the behaviour of $A$, we have that $A'$ succeeds
+  precisely whenever one of $A$'s verification queries succeeds.  Thus:
+  \[ \Pr[V] = \Succ{suf-cma}{\mathcal{M}}(A')
+            \le \InSec{suf-cma}(\mathcal{M}; t, q_T, q_V - 1). \]%
+  Similarly, the adversary $Z$ succeeds with precisely the same probability
+  as $A$, given that all of its verification queries failed; i.e.,
+  \[ \Pr[S \mid \lnot V] \Pr[\lnot V] = \Succ{suf-cma}{\mathcal{M}}(Z)
+                         \le \InSec{suf-cma}(\mathcal{M}; t, q_T, 0). \]%
+  Because $A$ was chosen arbitrarily, we can maximize:
+  \begin{eqnarray*}[rl]
+    \InSec{suf-cma}(\mathcal{M}; t, q_T, q_V)
+    & \le \InSec{suf-cma}(\mathcal{M}; t, q_T, q_V - 1) +
+          \InSec{suf-cma}(\mathcal{M}; t, q_T, 0) \\
+    & \le (q_V + 1)\InSec{suf-cma}(\mathcal{M}; t, q_T, 0)
+  \end{eqnarray*}
+  as required.
+  
+  To show that the bound is tight, consider a random function $F$ used as a
+  MAC, with $L$-bit output.  Then we claim that $\InSec{suf-cma}(F; t, q_T,
+  q_V) = (q_V + 1)/2^L$.  To save typing, let $e_q = \InSec{suf-cma}(F; t,
+  q_T, q)$.  We prove the claim by induction on $q$.  Firstly, note that if
+  $q = 0$ then necessarily $e_q = 1/2^L$.  Now suppose that $e_{q-1} =
+  q/2^L$.  We're now allowed an extra query, so rather than returning the
+  result, we feed it to the verification oracle.  If it answers `yes' then we
+  return it; otherwise we guess randomly from the remaining $2^L - q$
+  possibilities.  Now
+  \begin{eqnarray*}[rl]
+    e_q &= e_{q-1} + (1 - e_{q-1}) \frac{1}{2^L - q} \\
+        &= \frac{q}{2^L} + \frac{2^L - q}{2^L} \cdot \frac{1}{2^L - q} \\
+        &= \frac{q + 1}{2^L}
+  \end{eqnarray*}
+  as claimed.
+\end{proof}
+
+\xcalways\subsection{The HMAC construction}\x
+
+\begin{slide}
+  \head{The HMAC construction \cite{Bellare:1996:KHF}, 1: motivation}
+
+  It ought to be possible to construct a decent MAC using a hash function.
+  Many attempts have failed, however.  For example, these constructions are
+  weak if used with Merkle-Damg\aa{}rd iterated hashes:
+  \begin{itemize}
+  \item Secret prefix: $T_K(m) = H(K \cat m)$.
+  \item Secret suffix: $T_K(m) = H(m \cat K)$.
+  \end{itemize}
+
+  It would be nice to have a construction whose security was provably related
+  to some plausible property of the underlying hash function.
+\end{slide}
+
+\begin{slide}
+  \head{The HMAC construction, 2: definition of NMAC}
+  Let $H\colon \{0, 1\}^* \to \{0, 1\}^k$ be an iterated hash, constructed
+  from the compression function $F\colon \{0, 1\}^k \times \{0, 1\}^L \to
+  \{0, 1\}^k$.  We define a keyed version of $H$.  Let $K \in \{0, 1\}^k$;
+  then we compute $H_K(x)$ as follows:
+  \begin{enumerate}
+  \item Pad and split $x$ into the $L$-bit blocks $x_0$, $x_1$, \ldots,
+    $x_{n-1}$ as before.
+  \item Set $I_0 = K$.  Let $I_{i+1} = F(I_i \cat x_i)$ for $0 \le i < n$.
+  \item The result $H_K(x) = I_n$.
+  \end{enumerate}
+  The NMAC (nested-MAC) construction requires two independent $k$-bit keys
+  $K_0$ and $K_1$.  The construction itself is simply:
+  \[ \Xid{T}{NMAC}^H_{K_0, K_1}(x) = H_{K_0}(H_{K_1}(x)). \]
+  NMAC is deterministic, so verification consists of computing the tag and
+  comparing.
+\end{slide}
+
+\begin{slide}
+  \head{The HMAC construction, 3: security of NMAC}
+
+  Consider a function $F\colon \{0, 1\}^k \times \{0, 1\}^* \to \{0, 1\}^k$.
+  We say that $F$ is \emph{$(t, q, \epsilon)$-weakly collision resistant} if,
+  for any adversary $A$ constrained to run in time $t$ and permitted $q$
+  oracle queries,
+  \[ \Pr[K \getsr \{0, 1\}^k;
+         (x, y) \gets A^{F_K(\cdot)}] \le \epsilon :
+         x \ne y \land F_K(x) = F_K(y) \]%
+  
+  If $H_K$ is a $(t, q_T, q_V, \epsilon)$-secure MAC on $k$-bit messages, and
+  moreover $(t, q_T + q_V, \epsilon')$-weakly collision resistant, then
+  $\Xid{T}{NMAC}^H$ is a $(t, q_T, q_V, \epsilon + \epsilon')$-secure MAC.
+\end{slide}
+
+\begin{slide}
+  \head{The HMAC construction, 4: NMAC security proof}
+
+  Let $A$ be an adversary which forges a $\Xid{T}{NMAC}^H$ tag in time $t$,
+  using $q_T$ tagging queries and $q_V$ verification queries with probability
+  $\epsilon$.  We construct an adversary $A'$ which forces a $H$ tag for a
+  $k$-bit in essentially the same time.
+  \begin{program}
+    Adversary $A'^{T(\cdot), V(\cdot, \cdot)}$ \+ \\
+      $K \getsr \{0, 1\}^k$; \\
+      $(m, \tau) \gets A^{T(H_K(\cdot)), V(H_K(\cdot), \cdot)}$; \\
+      \RETURN $(H_K(m), \tau)$;
+  \end{program}
+  $A'$ might fail even though $A$ succeeded only if the message it returns,
+  $H_K(m)$, collides with one of its tagging queries.  But since $H_K$ is
+  $(t, q_T + q_V, \epsilon')$-weakly collision resistant, this happens with
+  at most probability $\epsilon'$.  Hence, $A'$ succeeds with probability at
+  least $\epsilon - \epsilon'$.  Rearrangement yields the required result.
+\end{slide}
+
+\begin{slide}
+  \head{The HMAC construction, 5: from NMAC to HMAC}
+
+  Implementing NMAC involves using strange initialization vectors and
+  generally messing about with your hash function.  HMAC is an attempt to
+  capture the provable security properties using a plain ol' hash function.
+
+  Suppose $H$ is an iterated hash function with a $k$-bit output and $L$-bit
+  blocks (with $L \ge k$).  We set $\id{ipad}$ to be the byte $\hex{36}$
+  repeated $L/8$ times, and $\id{opad}$ to be the byte $\hex{5C}$ repeated
+  $L/8$ times.  Select a key $K$ of $L$ bits: if your key is shorter, pad it
+  by appending zero bits; if it's longer, hash it with $H$ and then pad.
+
+  The HMAC tagging function is then defined as
+  \[ \Xid{T}{HMAC}^H_K(m) =
+     H(K \xor \id{opad} \cat H(K \xor \id{ipad} \cat m)). \]%
+\end{slide}
+
+\begin{slide}
+  \head{The HMAC construction, 6: comparison with NMAC}
+
+  Comparing the two constructions, we see that
+  \[ \Xid{T}{HMAC}^H_K =
+     \Xid{T}{NMAC}^{H'}_{F(I \cat K \xor \id{opad}),
+                         F(I \cat K \xor \id{ipad})}. \]%
+  Here, $I$ is $H$'s initialization vector, $F$ is the compression function;
+  $H'$ denotes a keyed hash function that is `like' $H$ but performs padding
+  as if there were an extra initial block of message data for each message.
+  
+  The proof of NMAC assumes that the two keys are random and independent.
+  This isn't the case in HMAC, but a little handwaving about pseudorandomness
+  of the compression function makes the problem go away.
+\end{slide}
+
+\begin{exercise}
+  Suppose that $F\colon \{0, 1\}^k \times \{0, 1\}^{t+\ell} \to \{0,
+  1\}^t$ is a PRF.  Let $x \in \{0, 1\}^*$ be a message.  We define the
+  function $H_K(x)$ as follows:
+  \begin{itemize}
+  \item Pad $x$ to a multiple of $\ell$ bits using some injective
+    mapping.  Break the image of $x$ under this mapping into $\ell$-bit
+    blocks $x_0, x_1, \ldots, x_{n-1}$.
+  \item For $0 \le i \le n$, define $H^{(i)}_K(x)$ by
+    \[ H^{(0)}_K(x) = I; \qquad 
+       H^{(i+1)}_K(x) = F_K(H^{(i)}(x) \cat x_i) \]%
+    where $I$ is some fixed $t$-bit string (e.g., $I = 0^t$).
+  \item Then set $H_K(x) = H^{(n)}_K(x)$.
+  \end{itemize}
+  We define two (deterministic) MACs $\mathcal{M}^i = (T^i, V^i)$ (for
+  $i \in \{0, 1\}$) using the $H_K$ construction.  Verification in each
+  case consists of computing the tag and comparing to the one offered.
+  \begin{eqlines*}
+    T^0_K(x) = H_K(x); \qquad T^1_K(x) = H_K(x \cat K)
+    V^i_K(x, \tau) = \begin{cases}
+      1 & if $\tau = T^i_K(x)$ \\
+      0 & otherwise
+    \end{cases}
+  \end{eqlines*}
+  Decide whether each of these constructions is secure.  A full proof is
+  rather hard: an informal justification would be good.
+  \answer%
+  $\mathcal{M}^0$ is secure; $\mathcal{M}^1$ isn't, under the sole
+  assumption that $F$ is a PRF.
+
+  To see that $\mathcal{M}^0$ is secure, it suffices to show that $T^0$
+  is a PRF.  This is actually quite involved.  Given an adversary $A$
+  attacking $T^1$ as a PRF, we construct an adversary $B$ attacking $F$,
+  which simply computes $H$ as required, using the oracle supplied.  To
+  complete the proof, we need to show a bound on the
+  information-theoretic security of $H$ when instantiated using a random
+  function $F$.  For the sake of simplicity, we allow the adversary $A$
+  to query on \emph{padded} messages, rather than the raw unpadded
+  messages.  We count the number $q'$ of individual message blocks.
+
+  As the game with $A$ progresses, we can construct a directed
+  \emph{graph} of the query results so far.  We start with a node
+  labelled $I$.  When processing an $H$-query, each time we compute $t'
+  = F(t \cat x_i)$, we add a node $t'$, and an edge $x_i$ from $t$ to
+  $t'$.  The `bad' event occurs whenever we add an edge to a previously
+  existing node.  We must show, firstly, that the
+  adversary cannot distinguish $H$ from a random function unless the bad
+  event occurs; and, secondly, that the bad event doesn't occur very
+  often.
+
+  The latter is easier: our standard collision bound shows that the bad
+  event occurs during the game with probability at most $q'(q' - 1)/2$.
+
+  The former is trickier.  This needs a lot more work to make it really
+  rigorous, but we show the idea.  Assume that the bad event has not
+  occured. Consider a query $x_0, x_1, \ldots, x_{n-1}$.  If it's the
+  same as an earlier query, then $A$ learns nothing (because it could
+  have remembered the answer from last time).  If it's \emph{not} a
+  prefix of some previous query, then we must add a new edge to our
+  graph; then either the bad event occurs or we create a new node for
+  the result, and because $F$ is a random function, the answer is
+  uniformly random.  Finally, we consider the case where the query is a
+  prefix of some earlier query, or queries.  But these were computed at
+  random at the time.
+
+  At the end of all of this, we see that
+  \[ \InSec{prf}(T^0; t, q) \le
+       \InSec{prf}(F; t, q') + \frac{q'(q' - 1)}{2} \]%
+  and hence
+  \[ \InSec{suf-cma}(\mathcal{M}^0; t, q) \le 
+       \InSec{prf}(F; t, q') + \frac{q'(q' - 1)}{2} + \frac{1}{2^t}. \]%
+
+  Now we turn our attention to $T^1$.  It's clear that we can't simulate
+  $T^1$ very easily using an oracle for $F$, since we don't know $K$
+  (and indeed there might not be a key $K$).  The intuitive reason why
+  $T^1$ is insecure is that $F$ might have leak useful information if
+  its input matches its key.  This doesn't affect the strength of $F$ as
+  a PRF because you have to know the key before you can exploit this
+  leakage; but $T^1$ already knows the key, and this can be exploited to
+  break the MAC.
+
+  To show that this is insecure formally, let $F'$ be defined as
+  follows:
+  \[ F'_K(x) = \begin{cases}
+       K      & if $x = p \cat K \cat q$ where
+                $|p| = t$ and $|q| = \ell - k$ \\
+       F_K(x) & otherwise
+     \end{cases}. \]%
+  We choose a simple injective padding scheme: if $x$ is a message then
+  we form $x' = x \cat 1 \cat 0^n$, where $0 \le n < \ell$ and $|x'|$ is
+  a multiple of $\ell$.  If $T^1$ is instantiated with this PRF then it
+  is insecure as a MAC: submitting a tagging query for the empty string
+  $\emptystring$ reveals the key $K$, which can be used to construct a
+  forgery.
+
+  To complete the proof, we must show that $F'$ is a PRF.  Let $A$ be an
+  adversary attacking $F'$.  We consider a series of games; for each
+  $\G{i}$, let $S_i$ be the event that $A$ returns $1$ in that game.
+  Game~$\G0$ is the standard attack game with $A$ given an oracle for a
+  random function; game~$\G1$ is the same, except that $A$ is given an
+  oracle for $F'_K$ for some $K \inr \{0, 1\}^k$.  Then
+  $\Adv{prf}{F'}(A) = \Pr[S_1] - \Pr[S_0]$.  Let game~$\G2$ be the same
+  as $\G1$, except that if $A$ makes any query of the form $p \cat K
+  \cat q$ with $|p| = t$ and $|q| = \ell - k$ then the game halts
+  immediately, and let $F_2$ be the event that this occurs.  By
+  Lemma~\ref{lem:shoup} (page~\pageref{lem:shoup}), then, $|{\Pr[S_2]} -
+  \Pr[S_1]| \le \Pr[F_2]$.  Let game~$\G3$ be the same as $\G2$ except
+  that we give $A$ an oracle for $F_K$ rather than $F'_K$.  Since $F$
+  and $F'$ differ only on queries of the form $p \cat K \cat q$, we have
+  $\Pr[S_3] = \Pr[S_2]$.  But $\Pr[S_3] - \Pr[S_0] = \Adv{prf}{F}(A) \le
+  \InSec{prf}(F; t, q)$.  Hence, $\Adv{prf}{F'}(A) \le \InSec{prf}{F}(A)
+  - \Pr[F_2]$.
+
+  Finally, we bound the probability of $F_2$.  Fix an integer $n$.
+  Consider an adversary $B$ attacking $F$ which runs as follows.  It
+  initially requests $F(0), F(1), \ldots, F(n - 1)$ from its oracle.  It
+  then runs $A$, except that, for each oracle query $x$, it parses $x$
+  as $p \cat K' \cat q$ with $|p| = t$, $|K'| = k$ and $|q| = \ell - k$;
+  then, if $F_{K'}(0) = F(0) \land F_{K'}(1) = F(1) \land \cdots \land
+  F_{K'}(n - 1) = F(n - 1)$, $B$ immediately returns $1$, claiming that
+  its oracle $F$ is the function $F_{K'}$; if this never occirs, $B$
+  returns $0$.  Clearly, if $B$ is given an instance $F_K$ of $F$ then
+  it succeeds with probability $\Pr[F_2]$; however, if $F$ is a random
+  function then $B$ returns $1$ with probability at most $q 2^{-nk}$.
+  Hence, $\Adv{prf}{F}(B) \le \Pr[F_2] - q 2^{-nk}$.  $B$ issues $q + n$
+  queries, and takes time $t + O(n q)$.  Wrapping everything up, we get
+  \[ \InSec{prf}(F'; t, q) \le 
+     2\cdot\InSec{prf}(F; t + O(q n), q + n) + \frac{q}{2^{nk}}. \]%
+  This completes the proof of generic insecurty for $\mathcal{M}^1$.
+\end{exercise}
+
+\xcalways\subsection{Universal hashing}\x
+
+\begin{slide}
+  \topic{almost-universal hash functions}
+  \head{Universal hashing, 1: definition}
+  
+  Consider a family of hash functions $H\colon \keys H \times \dom H \to
+  \ran H$.  We define
+  \[ \InSec{uh}(H) =
+     \max_{x \ne y} \Pr[K \getsr \keys H : H_K(x) = H_K(y)]. \]%
+  If $\InSec{uh}(H) \le \epsilon$ then we say that $H$ is
+  \emph{$\epsilon$-almost universal}.  Note that the concept of
+  almost-universality is not quantified by running times.
+  
+  If $H$ is $1/|{\ran H}|$-almost universal, then we say that $H$ is
+  \emph{universal}.  Sometimes it's said that this is the best possible
+  insecurity: this isn't true.
+\end{slide}
+
+\begin{proof}[Counterexample]
+  Here's a function $H\colon \{0, 1, 2\} \times \{0, 1, 2, 3\} \to \{0, 1\}$
+  which is $\frac{1}{3}$-almost universal, though $|{\ran H}| = 2$:
+  \begin{quote} \item
+    \begin{tabular}[C]{c|cccc}
+      & 0 & 1 & 2 & 3 \\ \hlx{vhv}
+    0 & 0 & 1 & 0 & 1 \\
+    1 & 0 & 0 & 1 & 1 \\
+    2 & 0 & 1 & 1 & 0
+    \end{tabular}
+  \end{quote}
+\end{proof}
+
+\begin{slide}
+  \topic{dynamic view}
+  \head{Universal hashing, 2: a dynamic view}
+  
+  Suppose that $H$ is $\epsilon$-almost universal.  Consider this experiment:
+  \begin{program}
+    Experiment $\Expt{uh}{H}(A)$: \+ \\
+      $(x, y) \gets A$; \\
+      $K \getsr \keys H$; \\
+      \IF $x \ne y \land H_K(x) = H_K(y)$ \THEN \RETURN $1$; \\
+      \ELSE \RETURN $0$;
+  \end{program}
+  The adversary may make random decisions before outputting its selection
+  $x$, $y$.  We show that $\Pr[\Expt{uh}{H}(A) = 1] \le \InSec{uh}(H) =
+  \epsilon$.
+  
+  Let $\rho \in \{0, 1\}^*$ be $A$'s coin tosses: $A$ chooses $x$ and $y$ as
+  functions of $\rho$.  For some \emph{fixed} $\rho$,
+  \[ \Pr[K \getsr \keys H : H_K(x(\rho)) = H_K(y(\rho))] \le
+     \InSec{uh}(H). \]%
+\end{slide}
+
+\begin{slide}
+  \head{Universal hashing, 3: the dynamic view (cont.)}
+  
+  Now we treat $\rho$ as a random variable, selected from some distribution
+  $P$ on the set $\{0, 1\}^*$.  We see that
+  \begin{eqnarray*}[Ll]
+    \Pr[\rho \getsr P; K \getsr \keys H : H_K(x(\rho)) = H_K(y(\rho))] \\
+    &= \sum_{\rho \in \{0, 1\}^*}
+       P(\rho) \cdot \Pr[K \getsr \keys H : H_K(x(\rho)) = H_K(y(\rho))] \\
+    &\le \sum_{\rho \in \{0, 1\}^*} P(\rho) \cdot \InSec{uh}(H)
+     = \InSec{uh}(H).
+  \end{eqnarray*}
+  Thus, no adversary can succeed in producing a collision in an
+  $\epsilon$-almost universal hash with probability better than $\epsilon$.
+  But obviously the adversary can ignore its coin tosses and simply return
+  the best colliding pair.  Hence the two notions are completely equivalent.
+\end{slide}
+
+\begin{slide}
+  \topic{composition}
+  \head{Universal hashing, 4: composition}
+  
+  Suppose that $G$ is $\epsilon$-almost universal, and $G'$ is
+  $\epsilon'$-almost universal, and $\dom G = \ran G'$.  We define the
+  composition $G \compose G'$ to be the family $H\colon (\keys G \times
+  \keys G') \times \dom G' \to \ran G$ by $H_{k, k'}(m) =
+  G_k(G'_{k'}(m))$.
+  
+  Then $H$ is $(\epsilon + \epsilon')$-almost universal.  To see this, fix $x
+  \ne y$, and choose $K = (k, k') \inr \keys G \times \keys G'$.  Let $x' =
+  G'_{k'}(x)$ and $y' = G'_{k'}(y)$.  Following our previous result, we see:
+  \begin{eqnarray*}[rl]
+    \Pr[H_K(x) = H_K(y)]
+    &= \Pr[G_k(G'_{k'}(x)) = G_k(G'_{k'}(y))] \\
+    &= \Pr[G_k(x') = G_k(y')] \\
+    &= \Pr[G_k(x') = G_k(y') \mid x' \ne y'] \Pr[G'_{k'}(x) \ne G'_{k'}(y)]\\
+    &\le \epsilon + \epsilon'.
+  \end{eqnarray*}
+\end{slide}
+
+\begin{slide}
+  \topic{the collision game}
+  \head{Universal hashing, 5: the collision game}
+  
+  Suppose that, instead of merely a pair $(x, y)$, our adversary was allowed
+  to return a \emph{set} $Y$ of $q$ elements, and measure the probability
+  that $H_K(x) = H_K(y)$ for some $x \ne y$ with $x, y \in Y$, and for $K
+  \inr \keys H$.
+  
+  Let $\InSec{uh-set}(H; q)$ be maximum probability acheivable for sets $Y$
+  with $|Y| \le q$.  Then
+  \[ \InSec{uh-set}(H; q) \le \frac{q(q - 1)}{2} \cdot \InSec{uh}(H) .\]
+\end{slide}
+
+\begin{proof}
+  This is rather tedious.  We use the dynamic view.  Suppose $A$ returns $(x,
+  Y)$ with $|Y| = q$, and succeeds with probability $\epsilon$.  Consider
+  \begin{program}
+    Adversary $A'$: \+ \\
+      $(x, Y) \gets A$; \\
+      $y \getsr Y$; \\
+      \RETURN $(x, Y \setminus \{y\})$;
+  \end{program}
+  The worst that can happen is that $A'$ accidentally removes the one
+  colliding element from $Y$.  This occurs with probability $2/q$.  So
+  \[ \Succ{uh-set}{H}(A') \ge \frac{q - 2}{q} \Succ{uh-set}{H}(A). \]
+  Rearranging and maximzing gives
+  \[ \InSec{uh-set}(H; q) \le
+     \frac{q}{q - 2} \cdot \InSec{uh-set}(H; q - 1). \]
+  Note that $\InSec{uh-set}(H; 2) = \InSec{uh}(H)$ is our initial notion.  A
+  simple inductive argument completes the proof.
+\end{proof}
+
+\begin{slide}
+  \topic{a MAC}
+  \head{Universal hashing, 6: a MAC}
+  
+  Suppse that $H\colon \{0, 1\}^k \times \{0, 1\}^* \to \{0, 1\}^l$ is an
+  almost universal hash fucntion, and $F\colon \{0, 1\}^{k'} \times \{0,
+  1\}^l \to \{0, 1\}^L$ is a PRF\@.  Define a MAC $\Xid{\mathcal{M}}{UH}^{H,
+    F} = (\Xid{T}{UH}^{H, F}, \Xid{V}{UH}^{H, F})$ where:
+  \begin{eqnarray*}[rl]
+    \Xid{T}{UH}^{H, F}_{K, K'}(m) &= F_{K'}(H_K(m)) \\
+    \Xid{V}{UH}^{H, F}_{K, K'}(m, \tau) &= \begin{cases}
+      1 & if $\tau = F_{K'}(H_K(m))$ \\
+      0 & otherwise
+    \end{cases}.
+  \end{eqnarray*}
+  We have
+  \begin{eqnarray*}[Ll]
+     \InSec{suf-cma}(\Xid{\mathcal{M}}{UH}^{H, F}; t, q_T, q_V) \\
+     & \le
+     (q_V + 1) \biggl(\InSec{prf}(F; t, q_T + 1) + \frac{1}{2^L} +
+                      \frac{q_T(q_T - 1)}{2} \cdot \InSec{uh}(H)\biggr).
+  \end{eqnarray*}
+\end{slide}
+
+\begin{proof}
+  We shall prove the result for $q_V = 0$ and $q_T = q$, and appeal to the
+  earlier result on verification oracles.
+  
+  Suppose $A$ attacks the scheme $\Xid{\mathcal{M}}{UH}^{H, F}$ in time $t$,
+  issuing $q$ tagging queries.  Consider a distinguisher $D$, constructed
+  from a forger $A$:
+  \begin{program}
+    Distinguisher $D^{F(\cdot)}$: \+ \\
+      $K \getsr \{0, 1\}^k$; \\
+      $\Xid{T}{list} \gets \emptyset$; \\
+      $(m, \tau) \gets A^{\id{tag}(K, \cdot)}$; \\
+      \IF $m \notin \Xid{T}{list} \land \tau = F(H_K(m))$
+      \THEN \RETURN $1$; \\
+      \ELSE \RETURN $0$; \- \\[\smallskipamount]
+    Oracle $\id{tag}(K, m)$: \+ \\
+      $\Xid{T}{list} \gets \Xid{T}{list} \cup \{m\}$; \\
+      \RETURN $F(H_K(m))$; \- \\[\smallskipamount]
+  \end{program}
+  Note that $A$ isn't provided with a verification oracle: that's because we
+  didn't allow it any verification queries.
+
+  We can see immediately that
+  \[ \Pr[K \getsr \{0, 1\}^{k'} : D^{F_K(\cdot)} = 1] =
+     \Succ{suf-cma}{\Xid{\mathcal{M}}{UH}^{H, F}}(A). \]%
+  
+  We must now find an upper bound for $\Pr[F \getsr \Func{l}{L} :
+  D^{F(\cdot)}]$.  Suppose that the adversary returns the pair $(m^*,
+  \tau^*)$, and that its tagging oracle queries and their answers are $(m_i,
+  \tau_i)$ for $0 \le i < q$.  Consider the event $C$ that $H_K(m) =
+  H_K(m')$ for some $m \ne m'$, with $m, m' \in \{m^*\} \cup \{\,m_i \mid 0
+  \le i < q\,\}$.
+  
+  If $C$ doesn't occur, then $F$ has not been queried before at $H_K(m)$, but
+  there's a $2^{-L}$ probability that the adversary guesses right anyway.  If
+  $C$ does occur, then we just assume that the adversary wins, even though it
+  might not have guessed the right tag.
+  
+  By our result on the collision game, $\Pr[C] \le q \cdot \InSec{uh}(H)$.
+  Then
+  \[ \Succ{prf}{F}(D) \ge
+     \Succ{suf-cma}{\Xid{\mathcal{M}}{UH}^{H, F}}(A) -
+       \frac{1}{2^L} - \frac{q(q - 1)}{2} \cdot \InSec{uh}(H). \]%
+  The result follows.
+\end{proof}
+
+\begin{slide}
+  \topic{almost XOR-universality}
+  \head{Almost XOR-universality, 1: definition}
+  
+  Consider a family of hash functions $H\colon \keys H \times \dom H \to
+  \{0, 1\}^L$.  Define
+  \[ \InSec{xuh}(H) =
+     \max_{x \ne y, \delta}
+       \Pr[K \getsr \keys H : H_K(x) \xor H_K(y) = \delta]. \]%
+  If $\InSec{xuh}(H) < \epsilon$ then we say that $H$ is
+  \emph{$\epsilon$-almost XOR-universal}, or \emph{AXU}.  Setting $\delta =
+  0$ shows that
+  \begin{eqnarray*}[rl]
+     \InSec{xuh}(H)
+     & \ge \max_{x \ne y} \Pr[K \getsr \keys H : H_K(x) \xor H_K(y) = 0] \\
+     & = \InSec{uh}(H).
+  \end{eqnarray*}
+  We can take a dynamic view of almost XOR-universality using the same
+  technique as for almost universal functions.
+  
+  If $H$ is $2^{-L}$-almost XOR universal then we say that $H$ is
+  \emph{XOR-universal}.  This is the best acheivable.
+\end{slide}
+
+\begin{proof}
+  Fix some pair $x \ne y$.  Choose $\delta \inr \{0, 1\}^L$.  Then for any
+  \emph{fixed} $K \in \keys H$, $H_K(x)$ and $H_K(y)$ are fixed, so $H_K(x)
+  \xor H_K(y)$ is fixed, and $\Pr[H_K(x) \xor H_K(y) = \delta] = 2^{-L}$.
+  Using the same trick as for proving the dynamic view:
+  \begin{eqnarray*}[Ll]
+    \Pr[\delta \getsr \{0, 1\}^L; K \getsr \keys H :
+        H_K(x) \xor H_K(y) = \delta] \\
+    & = \sum_{K \in \keys H} \frac{1}{|{\keys H}|}
+          \Pr[\delta \getsr \{0, 1\}^L; K \getsr \keys H :
+              H_K(x) \xor H_K(y) = \delta] \\
+    & = \sum_{K \in \keys H} \frac{1}{|{\keys H}|} 2^{-L} = 2^{-L}.
+  \end{eqnarray*}
+  Since $H$ is arbitrary, this proves the lower bound on the almost
+  XOR-universality.
+\end{proof}
+
+\begin{slide}
+  \topic{composition}
+  \head{Almost XOR-universality, 2: composition}
+  
+  We extend our result about composition of almost-universal functions.
+  Suppose that $G$ is $\epsilon$-almost XOR universal, and $G'$ is
+  $\epsilon'$-almost universal (it doesn't have to be almost XOR-universal),
+  and $\dom G = \ran G'$.
+
+  Then the composition $H = G \compose G'$ is $(\epsilon + \epsilon')$-almost
+  XOR-universal.  The proof is simple, and very similar to the
+  almost-universal case.
+\end{slide}
+
+\begin{slide}
+  \topic{a better MAC}
+  \head{Almost XOR-universality, 3: a better MAC}
+
+  The security result for the UH-based MAC contains a factor $q_T$, which
+  it'd be nice to remove.  Our new scheme uses an AXU hash $H\colon \keys H
+  \times \{0, 1\}^* \to \{0, 1\}^l$ and a PRF $F\colon \keys F \times \{0,
+  1\}^l \to \{0, 1\}^L$.
+  
+  We first present a stateful version $\Xid{\mathcal{M}}{XUH}^{H, F}$.
+  Choose $(K, K') \inr \keys H \times \keys F$, and initialize a counter $i
+  \gets 0$.  The tagging and verification algorithms are then:
+  \begin{program}
+    Algorithm $\Xid{T}{XUH}^{H, F}_{K, K'}(m)$: \+ \\
+      $\tau \gets (i, H_K(m) \xor F_{K'}(i))$; \\
+      $i \gets i + 1$; \\
+      \RETURN $\tau$;
+  \next
+    Algorithm $\Xid{V}{XUH}^{H, F}_{K, K'}(m, \tau)$: \+ \\
+      $(s, \sigma) \gets \tau$; \\
+      \IF $\sigma = H_K(m) \xor F_{K'}(i)$ \THEN \RETURN $1$; \\
+      \ELSE \RETURN $0$;
+  \end{program}
+  Note that verification is stateless.
+\end{slide}
+
+\begin{slide}
+  \head{Almost XOR-universality, 4: security of AXU-based MACs}
+  
+  For the stateful scheme presented earlier, provided $q_T \le 2^l$, we have
+  \begin{eqnarray*}[Ll]
+     \InSec{suf-cma}(\Xid{\mathcal{M}}{XUH}^{H, F}; t, q_T, q_V) \\
+     & \le (q_V + 1)(\InSec{prf}(F; t, q_T + 1) + \InSec{xuh}(H) + 2^{-L}).
+  \end{eqnarray*}
+\end{slide}
+
+\begin{slide}
+  \head{Almost XOR-universality, 5: randomized AXU-based MAC}
+  
+  We can avoid statefulness by using randomization.  This new scheme is
+  $\Xid{\mathcal{M}}{XUH$\$$}^{H, F} = (\Xid{T}{XUH$\$$}^{H, F},
+  \Xid{V}{XUH$\$$}^{H, F})$:
+  \begin{program}
+    Algorithm $\Xid{T}{XUH$\$$}^{H, F}_{K, K'}(m)$: \+ \\
+      $s \getsr \{0, 1\}^l$; \\
+      $\tau \gets (s, H_K(m) \xor F_{K'}(s))$; \\
+      \RETURN $\tau$;
+  \next
+    Algorithm $\Xid{V}{XUH$\$$}^{H, F}_{K, K'}(m, \tau)$: \+ \\
+      $(s, \sigma) \gets \tau$; \\
+      \IF $\sigma = H_K(m) \xor F_{K'}(i)$ \THEN \RETURN $1$; \\
+      \ELSE \RETURN $0$;
+  \end{program}
+  \begin{eqnarray*}[Ll]
+     \InSec{suf-cma}(\Xid{\mathcal{M}}{XUH$\$$}^{H, F}; t, q_T, q_V) \\
+     & \le (q_V + 1)
+         \Bigl(\InSec{prf}(F; t, q_T + 1) + \InSec{xuh}(H) + 2^{-L} +
+               \frac{q_T(q_T - 1)}{2^{l+1}}\Bigr).
+  \end{eqnarray*}
+\end{slide}
+
+\begin{proof}
+  We prove the result with $q_V = 0$ and $q_T = q$, and appeal to the result
+  on verification oracles.  Let $m_i$ be the message specified in the $i$-th
+  tagging query ($0 \le i < q$), and let $(s_i, \sigma_i) = (s_i, H_K(m) \xor
+  F_{K'}(s_i))$ be the tag returned.  We call the $s_i$ the \emph{nonce}.
+  
+  We prove the result for the stateless scheme.  The bound $q \le 2^l$
+  ensures that the nonces are all distinct (we have $s_i = i$).  The security
+  bound for the randomized version merely has as an extra term upper bound
+  for the probability of a nonce collision.
+
+  Let $A$ be an adversary attacking the MAC in time $t$, and using $q$
+  tagging queries.  Then we present the following pair of adversaries:
+  \begin{program}
+    Distinguisher $D^{F(\cdot)}$: \+ \\
+      $i \gets 0$; \\
+      $\Xid{T}{list} \gets \emptyset$; \\
+      $K \getsr \keys H$; \\
+      $(m, \tau) \gets A^{\id{tag}}$; \\
+      $(s, \sigma) \gets \tau$; \\
+      \IF $(m, \tau) \notin \Xid{T}{list} \land
+           \sigma = H_K(m) \xor F(s)$
+      \THEN \RETURN $1$; \\
+      \ELSE \RETURN $0$; \- \\[\smallskipamount]
+    Oracle $\id{tag}(m)$: \+ \\
+      $\tau \gets (i, H_K(m) \xor F(i))$; \\
+      $\Xid{T}{list} \gets \Xid{T}{list} \cup \{(m, \tau)\}$; \\
+      $i \gets i + 1$;
+      \RETURN $\tau$;
+  \next
+    Collision-finder $C$: \+ \\ 
+      $i \gets 0$; \\
+      $(m, \tau) \gets A^{\id{tag}}$; \\
+      $(s, \sigma) \gets \tau$; \\
+      \IF $s \ge i \lor m = m_s$ \THEN \ABORT; \\
+      \RETURN $(m, m_s, \sigma \xor \sigma_s)$; \- \\[\smallskipamount]
+    Oracle $\id{tag}(m)$: \+ \\
+      $m_i \gets m$; \\
+      $\sigma_i \getsr \{0, 1\}^L$; \\
+      $\tau \gets (i, \sigma_i)$; \\
+      $i \gets i + 1$; \\
+      \RETURN $\tau$;
+  \end{program}
+
+  We need to find a lower bound on the advantage of $D$.  If $F$ is chosen
+  from the PRF then $D$ returns $1$ precisely when $A$ finds a valid
+  forgery.  We now examine the setting in which $F$ is a random function.
+  
+  Let $S$ be the event that $A$ succeeds in returning a valid forgery when
+  $F$ is random, and let $N$ be the event that the nonce $s$ returned by $A$
+  is not equal to any nonce $s_i$ returned by the tagging oracle.  Suppose
+  $N$ occurs: then the random function $F$ has never been queried before at
+  $F$, and $\Pr[F(s) = \sigma \xor H_K(m)]$ is precisely $2^{-L}$.
+  
+  So suppose instead that $N$ doesn't occur.  Then, since the $s_i$ are
+  distinct, there is a unique $i$ such that $s = s_i$.  For $A$ to win, we
+  must have $m \ne m_i$ (for if $m = m_i$ then the only valid tag is
+  $H_K(m_i) \xor F(s_i) = \sigma_i$, which was already returned by the
+  tagging oracle).  If $A$'s forgery is successful then
+  \[ \sigma = H_K(m) \xor F(s)
+     \qquad \text{and} \qquad
+     \sigma_i = H_K(m_i) \xor F(s_i) \]%
+  but $s = s_i$, whence
+  \[ H_K(m_i) \xor H_K(m) = \sigma \xor \sigma_i. \]
+  Since the $s_i$ are distinct and $F$ is a random function, the $\sigma_i$
+  are independent uniformly-distributed random strings from $\{0, 1\}^L$.
+  Hence the collision-finder $C$ succeeds with probability $\Pr[S \land
+  \lnot N] \le \InSec{xuh}(H)$.
+
+  Wrapping up, we have
+  \begin{eqnarray*}[rl]
+    \Adv{prf}{F}(D)
+    & \ge \Succ{suf-cma}{\Xid{\mathcal{M}}{XUH}^{H, F}}(A) -
+      (\Pr[S \mid N] \Pr[N] + \Pr[S \mid \lnot N] \Pr[\lnot N]) \\
+    & \ge \Succ{suf-cma}{\Xid{\mathcal{M}}{XUH}^{H, F}}(A) -
+      (2^{-L} + \InSec{xuh}(H)).
+  \end{eqnarray*}
+  Maximizing and rearranging yields the required result.
+\end{proof}
+
+\begin{remark*}
+  Note that our bound has a $2^{-L}$ term in it that's missing from
+  \cite{Goldwasser:1999:LNC}.  We believe that their proof is wrong in its
+  handling of the XOR-collision probability.
+\end{remark*}
+
+\endinput
+
+%%% Local Variables: 
+%%% mode: latex
+%%% TeX-master: "ips"
+%%% End: 
diff --git a/auth-sig.tex b/auth-sig.tex
new file mode 100644 (file)
index 0000000..b3322ac
--- /dev/null
@@ -0,0 +1,609 @@
+\xcalways\section{Digital signatures}\x
+
+\xcalways\subsection{Definitions and security notions}\x
+
+\begin{slide}
+  \topic{syntax}
+  \head{Syntax of digital signature schemes}
+
+  A \emph{digital signature scheme} is a triple $\mathcal{S} = (G, S, V)$ of
+  algorithms:
+  \begin{itemize}
+  \item The \emph{key-generation} algorithm $G$ is a probabilistic algorithm
+    which, given no arguments (or a security parameter $1^k$ represented in
+    unary, in the asymptotic setting) returns a pair $(P, K)$ of public and
+    private values.
+  \item The \emph{signing} algorithm $S$ is a probabilistic algorithm.
+    If $(P, K) \in G$, and $m \in \{0, 1\}^*$ is some message, then $S(K, m)$
+    (usually written $S_K(m)$) returns a \emph{signature} $\sigma$ on $m$.
+  \item The \emph{verification} algorithm $V$ is a deterministic algorithm
+    returning a single bit. If $(P, K) \in G$ then $V(P, m) = 1$ iff $\sigma
+    \in S_K(m)$.  We usually write $V_P(M)$ instead of $V(P, M)$.
+  \end{itemize}
+  There are many other similar formulations of digital signature schemes.
+\end{slide}
+
+\begin{slide}
+  \topic{forgery goals}
+  \head{Forgery goals}
+  
+  We recognize several different types of forgeries which can be made:
+  \begin{itemize}
+  \item An \emph{existiential forgery} occurs when an adversary creates a
+    valid signature for some arbitrary message not of its choosing.
+  \item An \emph{selective forgery} occurs when an adversary creates a valid
+    signature for a message that it chooses.
+  \item A \emph{universal forgery} occurs when an adversary creates a valid
+    signature for some specific given message.
+  \item A \emph{total break} occurs when the adversary acquires the secret
+    key $K$, or equivalent information, and can continue to sign given
+    messages.
+  \end{itemize}
+  The formal definitions don't quite match up to these categories.
+\end{slide}
+
+\begin{slide}
+  \topic{attack types}
+  \head{Types of attacks}
+
+  We recognize a number of different types of attack:
+  \begin{itemize}
+  \item In a \emph{key-only} attack, the adversary is given only a public
+    key.
+  \item In a \emph{known-signature} attack, the adversary given the public
+    key and a collection of messages with valid signatures.
+  \item In a \emph{chosen-message} attack, the adversary is provided with an
+    oracle which signs messages of the adversary's choosing.
+  \end{itemize}
+  In order to have `won' the game, we require that the adversary return a
+  signature on a message which wasn't one of the known-signature messages or
+  a query to the signing oracle.
+\end{slide}
+
+\begin{slide}
+  \topic{funny abbreviations}
+  \head{Funny abbreviations}
+  
+  For each goal/attack type pair, we have a notion of security for digital
+  signature schemes.  To save typing, there are standard (ish) abbreviations
+  for these notions, e.g., EUF-CMA.
+  
+  The first part is the forger's goal: EUF, SUF, UUF for existentially,
+  selectively and universally unforgeable, respectively.
+
+  The second part is the attack time: KOA, KSA, CMA for key-only,
+  known-signature and chosen-message attack, respectively.
+
+  The strongest notion is EUF-CMA, and that's what we concentrate on.
+\end{slide}
+
+\begin{slide}
+  \topic{formal definitions}
+  \head{Formal definition of EUF-CMA}
+  
+  Consider this experiment involving a signature scheme $\mathcal{S} = (G, S,
+  V)$ and an adversary:
+  \begin{program}
+    Experiment $\Expt{euf-cma}{\mathcal{S}}(A)$: \+ \\
+      $(P, K) \gets G$; \\
+      $\Xid{m}{list} \gets \emptyset$; \\
+      $(m, \sigma) \gets A^{\id{sign}(\cdot)}(P)$; \\
+      \IF $m \notin \Xid{m}{list} \land V_P(m, \sigma) = 1$
+      \THEN \RETURN $1$; \\
+      \ELSE \RETURN $0$; \- \\[\smallskipamount]
+    Oracle $\id{sign}(m)$: \+ \\
+      $\Xid{m}{list} \gets \Xid{m}{list} \cup \{m\}$; \\
+      \RETURN $S_K(m)$;
+  \end{program}
+\end{slide}
+
+\begin{slide}
+  \head{Formal definition of EUF-CMA (cont.)}
+
+  We define:
+  \begin{eqnarray*}[rl]
+    \Succ{euf-cma}{\mathcal{S}}(A)
+    &= \Pr[\Expt{euf-cma}{\mathcal{S}}(A) = 1]; \\
+    \InSec{euf-cma}(\mathcal{S}; t, q)
+    &= \max_A \Succ{euf-cma}{\mathcal{S}}(A).
+  \end{eqnarray*}
+  where the maximum is taken over adversaries $A$ running in time $t$ and
+  issuing $q$ signing queries.
+
+  If $\InSec{euf-cma}(\mathcal{S}; t, q) \le \epsilon$ then we say that
+  $\mathcal{S}$ is a \emph{$(t, q, \epsilon)$-secure EUF-CMA digital
+  signature scheme}.
+\end{slide}
+
+\xcalways\subsection{Signature schemes from trapdoor one-way functions}\x
+
+\begin{slide}
+  \topic{RSA}
+  \head{RSA as a digital signature scheme}
+
+  RSA is, we assume, a one-way trapdoor permutation.  We'd like to be able to
+  turn this into a signature scheme.
+  
+  The obvious thing to do is just define $S_{(n, d)}(m) = m^d \bmod n$ and
+  $V_{(n, e)}(m, \sigma) = 1 \iff \sigma^e \equiv m \pmod{n}$.  But this
+  doesn't work.
+  \begin{itemize}
+  \item It allows key-only existential forgery.  Suppose we're given a public
+    key $(n, e)$.  Choose a signature $\sigma \in \Z/n\Z$.  Compute $m =
+    \sigma$.  Then $(m, \sigma)$ is a valid signed message.
+  \item It allows universal forgery under chosen message attack.  Suppose
+    we're given a public key $(n, e)$, and asked to forge a signature for
+    message $m$.  Choose $k \inr (\Z/n\Z)^*$, and request a signature
+    $\sigma'$ of $m' = m k^e$.  Then $\sigma' = m^d k^{ed} = m^d k$, so
+    $\sigma = \sigma' k^{-1}$ is a valid signature on $m$.
+  \end{itemize}
+\end{slide}
+
+\begin{slide}
+  \topic{hash-then-sign}
+  \head{Hash-then-sign}
+  
+  We could fix the problems with RSA if we preprocessed the message before
+  signing.  The first idea is to use a collision-resistant hash function $H$,
+  and to produce a signature on a message $m$, you compute $\sigma = H(m)^d
+  \bmod n$.
+
+  This doesn't work either.
+  \begin{itemize}
+  \item Collision-resistance isn't sufficient for security.  Just because $H$
+    is collision-resistant doesn't mean that $H(x) H(y) \ne H(x y)$ with
+    non-negligible probability.  If $H$ is partially multiplicative then
+    universal or selective forgery is still possible.
+  \item Recall the definition of a one-way function: if $x \inr \dom f$ then
+    finding $x'$ such that $f(x') = f(x)$ is hard, given only $f(x)$.  But
+    the range of a hash function like SHA-1 is only a tiny portion of the
+    domain of RSA.
+  \end{itemize}
+\end{slide}
+
+\begin{slide}
+  \topic{\PKCS1 padding}
+  \head{\PKCS1 signature padding \cite{RSA:1993:PRE}}
+  
+  Let $n = p q$ be an RSA modulus, with $2^{8(k-1)} < n < 2^{8k)}$ -- i.e.,
+  $n$ is a $k$-byte number.  Let $m$ be the message to be signed.
+  
+  We compute $\hat{m} = 0^8 \cat T \cat 1^z \cat 0^8 \cat I_H \cat H(m)$ for
+  some hash function $m$, where:
+  \begin{itemize}
+  \item $|\hat{m}| = 8k$, i.e., $\hat{m}$ is a $k$-byte string;
+  \item $0^8$ is the string of 8 zero-bits;
+  \item $T = 00000001$ is an 8-bit \emph{type} code;
+  \item $1^z$ is a padding string of one-bits (\PKCS1 requires $z \ge 64$);
+    and
+  \item $I_H$ is an identifier for the chosen hash function $H$.
+  \end{itemize}
+  This bit string is then converted into an integer, using a big-endian
+  representation.  Note that $\hat{m} < n$, since its top byte is zero.
+\end{slide}
+
+\begin{slide}
+  \head{Fixing RSA, 2: \PKCS1 padding (cont.)}
+
+  Diagramatically, \PKCS1 signature looks like this:
+  \begin{tabular}[C]{r|c|c|c|c|c|} \hlx{c{2-6}v}
+    \hex{00} & \hex{01} &
+    \hex{FF} \hex{FF} \ldots \hex{FF} &
+    \hex{00} &
+    $I_H$ &
+    $H(m)$
+    \\ \hlx{vc{2-6}}
+  \end{tabular}
+  
+  This partially obscures the multiplicative structure of RSA, but doesn't
+  expand the range of the transform at all.  \PKCS1 padding only uses a tiny
+  part of the domain of the RSA function.
+  
+  In \cite{Brier:2001:CRS}, Brier, Clavier, Coron and Naccache analyse
+  general affine padding schemes, of which the \PKCS1 scheme is an example,
+  and show that if the `message' -- the hash -- is more than a third the size
+  of the modulus.  In particular, existential forgery is possible in
+  polynomial time, and selective forgery in subexponential time (though much
+  faster than factoring the modulus).
+\end{slide}
+
+\xcalways\subsection{Full-domain hashing}\x
+
+\begin{slide}
+  \topic{description}
+  \head{Full-domain hashing}
+
+  Suppose we have an \emph{ideal hash}, whose range covers the whole of the
+  domain of our RSA transformation.  Would this be secure?  If we model the
+  ideal hash as a random oracle, the answer is yes.
+  
+  Let $\mathcal{T} = (G, f, T)$ be a trapdoor one-way function generator.
+  Then we define the signature scheme $\Xid{\mathcal{S}}{FDH}^{\mathcal{T},
+    H} = (\Xid{G}{FDH}^{\mathcal{T}, H(\cdot)}, \Xid{S}{FDH}^{\mathcal{T},
+    H(\cdot)}, \Xid{V}{FDH}^{\mathcal{T}, H(\cdot)})$ as follows:
+  \begin{itemize}
+  \item $\Xid{G}{FDH}^{\mathcal{T}, H(\cdot)}(\cdot) = G(\cdot)$, i.e., we
+    use $\mathcal{T}$'s key generator directly.
+  \item $\Xid{S}{FDH}^{\mathcal{T}, H(\cdot)}_K(m) = T_K(H(m))$: we
+    implicitly (and deterministically) coerce $H$'s output so that $H(\cdot)$
+    is uniformly distributed over $\ran f_P$.
+  \item $\Xid{V}{FDH}^{\mathcal{T}, H(\cdot)}_P(m, \sigma) = 1 \iff
+    f_P(\sigma) = H(M)$.
+  \end{itemize}
+\end{slide}
+
+\begin{slide}
+  \topic{security results}
+  \head{Security results for FDH}
+  
+  The full-domain hash signature scheme $\Xid{\mathcal{S}}{FDH}^{\mathcal{T},
+  H}$ is EUF-CMA if $\mathcal{T} = (G, f, T)$ is a trapdoor one-way
+  permutation.  The standard quantitative result is:
+  \[ \InSec{euf-cma}(\Xid{\mathcal{S}}{FDH}^{\mathcal{T}, H}; t, q_S, q_H)
+     \le
+     q_H \InSec{owf}(\mathcal{T}; t + O(q_H t_f)). \]%
+  Here, $t_f$ is the length of time required to compute $f$.  The additional
+  $q_H$ term is the number of random oracle (hashing) queries.  This doesn't
+  count queries made by the signing oracle.
+  
+  This isn't a terribly promising bound.  The problem comes because there is
+  a conflict between getting the adversary to invert the required point, and
+  being able to answer signing queries.
+\end{slide}
+
+\begin{proof}
+  Let $A$ be an adversary against $\Xid{\mathcal{S}}{FDH}^{\mathcal{T}}$
+  making $q_S$ signing and $q_H$ random oracle queries: we present an
+  inverter $I$ for $\mathcal{T}$.
+  \begin{program}
+    Inverter $I(P, y)$: \+ \\
+      $n \getsr \{0, 1, \ldots, q_H - 1\}$; \\
+      $i \gets 0$; \\
+      $\Xid{I}{map} \gets \emptyset$; \\
+      $\Xid{H}{map} \gets \emptyset$; \\
+      $(m, \sigma) \gets A^{\id{sign}(\cdot), \id{hash}(\cdot)}(P)$; \\
+      \RETURN $\sigma$;
+  \newline
+    Oracle $\id{sign}(m)$: \+ \\
+      \IF $m \notin \dom \Xid{H}{map}$ \THEN \\ \quad \= \+ \kill
+        $h' \getsr \dom f_P$; \\
+        $h \gets f_P(h')$; \\
+        $\Xid{H}{map} \gets \Xid{H}{map} \cup \{m \mapsto h\}$; \\
+        $\Xid{I}{map} \gets \Xid{I}{map} \cup \{h \mapsto h'\}$; \- \\
+      $h \gets \Xid{h}{map}(m)$; \\
+      \IF $h \notin \dom \Xid{I}{map}$ \THEN \ABORT; \\
+      \RETURN $\Xid{I}{map}(h)$;
+  \next
+    Oracle $\id{hash}(x)$: \+ \\
+      \IF $x \in \dom \Xid{H}{map}$ \THEN \RETURN $\Xid{H}{map}(x)$; \\
+      \IF $i = n$ \THEN $h \gets y$; \\
+      \ELSE \\ \quad \= \+ \kill
+        $h' \getsr \dom f_P$; \\
+        $h \gets f_P(h')$; \\
+        $\Xid{I}{map} \gets \Xid{I}{map} \cup \{h \mapsto h'\}$; \- \\
+      $\Xid{H}{map} \gets \Xid{H}{map} \cup \{x \mapsto h\}$; \\
+      \RETURN $h$;
+  \end{program}
+  Note that the inverter `cheats' a little.  One of its random oracle replies
+  is chosen to be the inverter's input $y$.  This is OK, because the rules of
+  the inverter's game say that that $y = f(x)$ for some $x$ chosen uniformly
+  at random from $\dom f_P$, and hence $y$ is also uniformly distributed and
+  independent, as required.
+
+  The inverter also `cheats' because it ensures that it knows inverses for
+  all of the queries except for the $n$-th.
+  
+  Let the $q_H$ query strings to the oracle \id{hash} be $M = \{m_0, m_1,
+  \ldots, m_{q_H-1}\}$.  Let $m$ be the message returned by $A$ and let
+  $\sigma$ be the returned signature.
+  
+  Consider the event $N$ that $m \notin M$; i.e., the random oracle was not
+  queried on the returned message.  Then, since $A$ has knowledge of neither
+  $H(M)$ nor $y$, and both are uniformly distributed over $\dom f_P$, the
+  probability that it succeeds in producing a valid forgery is equal to its
+  probability of successfully returning a correct inversion of $y$.
+  
+  Now consider the complement event $\lnot N$.  Then there is at least a
+  $1/q_H$ probability that $m = m_n$ -- the one which the inverter must
+  invert -- in which case $I$ succeeds if $A$ does.  (The probability might
+  be greater in the event that the adversary queries on the same string
+  several times, though doing so doesn't make a great deal of sense.)
+
+  Let $F$ be the event that $A$ returns a correct forgery, and let $V$ be the
+  event that $I$ returns a correct inversion.  Obviously,
+  \[ \Pr[F] = \Pr[F \land N] + \Pr[F \land \lnot N]
+     \quad \text{so} \quad
+     \Pr[F \land \lnot N] = \Pr[F] - \Pr[F \land N]. \]%
+  From the above discussion, we ahave
+  \[ \Pr[V \land N] = \Pr[F \land N]
+     \quad \text{and} \quad
+     \Pr[V \land \lnot N] \ge \frac{1}{q_H} \Pr[F \land \lnot N]. \]%
+  Finally,
+  \begin{eqnarray*}[rl]
+    \Pr[V] &= \Pr[V \land N] + \Pr[V \land \lnot N] \\
+           &\ge \Pr[F \land N] + \frac{1}{q_H} \Pr[F \land \lnot N] \\
+           &\ge \frac{1}{q_H}
+                  (\Pr[F \land N] + \Pr[F] - \Pr[F \land \lnot N]) \\
+           &= \frac{1}{q_H} \Pr[F].
+  \end{eqnarray*}
+  The result follows.
+\end{proof}
+
+\begin{remark*}
+  Most statements of this result seem to charge the adversary for hash
+  queries made by the experiment and by the constructed inverter, which seems
+  unreasonable.  The above result shows that the standard security bound
+  still holds, even under more generous accounting.
+\end{remark*}
+
+\xcalways\subsection{The Probabilistic Signature Scheme}\x
+
+\begin{slide}
+  \head{The Probabilistic Signature Scheme \cite{Bellare:1996:ESD}}
+  
+  We can improve the security bounds on FDH by making the signing operation
+  randomized.
+  
+  Let $\mathcal{T} = (G, f, T)$ be a trapdoor one-way function generator, as
+  before, but this time we require that the problem of inverting $f$ be
+  \emph{self-reducible}.
+  
+  For some public parameter $P$, choose $n$ such that $2^{8n} \le
+  |{\dom f_P}| < 2^{8n+1}$, and fix a security parameter $k$.  Then we need
+  two random oracles $H\colon \{0, 1\}^* \to \{0, 1\}^k$ and $H'\colon \{0,
+  1\}^k \to \{0, 1\}^{8n-k}$.
+\end{slide}
+
+\begin{slide}
+  \topic{description}
+  \head{Definition of PSS}
+  
+  We define the signature scheme $\Xid{\mathcal{S}}{PSS}^{\mathcal{T}} =
+  (\Xid{G}{PSS}^{\mathcal{T}, H(\cdot), H'(\cdot)},
+  \Xid{S}{PSS}^{\mathcal{T}, H(\cdot), H'(\cdot)}, \Xid{V}{PSS}^{\mathcal{T},
+    H(\cdot), H'(\cdot)})$:
+  \begin{program}
+    $\Xid{G}{PSS}^{\mathcal{T}, H(\cdot), H'(\cdot)}(x)$: \+ \\
+      $(P, K) \gets G(x)$; \\
+      \RETURN $(P, K)$;
+  \newline
+    $\Xid{S}{PSS}^{\mathcal{T}, H(\cdot), H'(\cdot)}(K, m)$: \+ \\
+      $r \getsr \{0, 1\}^k$; \\
+      $h \gets H(m \cat r)$; \\
+      $h' \gets (r \cat 0^{8n-2k}) \xor H'(x)$; \\
+      $y \gets 0^8 \cat h \cat h'$; \\
+      \RETURN $T_K(y)$;
+  \next
+    $\Xid{V}{PSS}^{\mathcal{T}, H(\cdot), H'(\cdot)}
+      (P, m, \sigma)$: \+ \\
+      $y \gets f_P(\sigma)$; \\
+      \PARSE $y$ \AS $z \cat k\colon h \cat 8n - k\colon h'$; \\
+      \PARSE $h' \xor H'(h)$ \AS $k\colon r \cat z$; \\
+      \IF $z = 0 \land z' = 0 \land h = H(m \cat r)$
+      \THEN \RETURN $1$; \\
+      \ELSE \RETURN $0$;
+  \end{program}
+\end{slide}
+
+\begin{slide}
+  \head{Diagram of PSS}
+
+  %%                 m <- {0,1}^*                r <-R {0, 1}^k
+  %%                     |                             |
+  %%                     |                             |
+  %%                     v                             |
+  %%                    [H]<---------------------------|
+  %%                     |                             |
+  %%                     |                             |
+  %%                     |                             v
+  %%                     |                            [||]<---- 0^{8n-2k}
+  %%                     |                             |
+  %%                     |                             |
+  %%                     v                             v
+  %%                     |-----------[H']------------>(+)
+  %%                     |                             |
+  %%                     |                             |
+  %%                     |                             v
+  %%    < 00 >   < s = H(m || r) >          < (r || 0^{8n-2k}) (+) H'(s) >
+
+  \vfil
+  \[ \begin{graph}
+    []!{0; <1cm, 0cm>:}
+    {m \in \{0, 1\}^*}="m" [rrrr] {r \inr \{0, 1\}^k}="r"
+    "m" :[d] *+[F]{H}="h"
+        :[ddd] *+=(2, 0)+[F:thicker]{\strut s = H(m \cat r)}
+    []!{-(2.5, 0)} *+=(1, 0)+[F:thicker]{\strut \hex{00}}
+    "r" :[dd] *{\ocat}="cat" [r] *+[r]{\mathstrut 0^{8n-2k}} :"cat"
+    :[d] *{\xor}="x" [uu] :"h"
+    "m" [ddd] :[rr] *+[F]{H'} :"x"
+    :[d] *+=(4, 0)+[F:thicker]{\strut t = (r \cat 0^{8n-2k}) \xor H'(s)}
+  \end{graph} \]
+  \vfil
+\end{slide}
+
+\begin{slide}
+  \topic{security results}
+  \head{Security results for PSS}
+  
+  For the PSS scheme we've seen, we have
+  \begin{eqnarray*}[Ll]
+    \InSec{euf-cma}(\Xid{\mathcal{S}}{PSS}^{\mathcal{T}};
+                    t, q_S, q_H, q_{H'}) \le \\
+    & \InSec{owf}(\mathcal{T}; t + O(t_n (q_S + q_H) + q_{H'})) +
+      \frac{3(q_H + q_{H'} + 2)}{2^k}.
+  \end{eqnarray*}
+  Here, $q_H$ and $q_H$ are the number of queries to the random oracles $H$
+  and $H'$, and $t_n$ is a magical constant relating to, among other things,
+  the time required for the self-reduction on the original problem and the
+  difference between $|{\dom f_P}|$ and $2^{8n}$.  For RSA or Rabin with a
+  multiple-of-8-bit modulus, $t_n$ will be some small multiple of $k$ times
+  the length of time for a public key operation.
+\end{slide}
+
+\begin{proof}
+  Suppose $A$ can forge a signature under the
+  $\Xid{\mathcal{S}}{PSS}^{\mathcal{T}}$ scheme.  We present an inverter $I$
+  which inverts $\mathcal{T}$.
+
+  We shall need to make use of the self-reducibility property of inverting
+  $\mathcal{T}$.  To do this in an abstract way, we require two algorithms:
+  \begin{itemize}
+  \item $\id{sr-choose}(P, y)$ returns a pair $(s, y')$, where $s$ is some
+    state information, and $y'$ is a new problem instance uniformly selected
+    from the domain of $f_P$; and
+  \item $\id{sr-solve}(s, x)$ solves an instance of the problem given a
+    solution to the instance created by \id{sr-choose}.
+  \end{itemize}
+  If $P$ is a public parameter for $f$, $y$ is some point in $\ran f_P$,
+  $\id{sr-choose}(P, y)$ returns $(s, y')$ and $f_P(x') = y'$ then we require
+  that $\id{sr-solve}(s, x')$ returns an $x$ such that $f_P(x) = y$.
+
+  By way of example, we present implementations of these algorithms for the
+  RSA function, where $P = (n, e)$:
+  \begin{program}
+    Algorithm $\id{sr-choose}(P, y)$: \+ \\
+      $(n, e) \gets P$; \\
+      \REPEAT $z \getsr \{1, 2, \ldots, n - 1\}$; \\
+      \UNTIL $\gcd(z, n) = 1$; \\
+      $s \gets (n, z^{-1} \bmod n)$; \\
+      \RETURN $(s, y z^e \bmod n)$;
+  \next
+    Algorithm $\id{sr-solve}(s, x')$: \+ \\
+      $(n, i) \gets s$; \\
+      \RETURN $x' i \bmod n$;
+  \end{program}
+  (The loop in \id{sr-choose} hardly ever needs more than one iteration.
+  Indeed, if the condition $\gcd{z, n} = 1$ fails, we've found a factor,
+  and could use that to solve the problem instance directly.)  We shall
+  assume that the \id{sr-choose} algorithm effectively completes in
+  deterministic time.)
+  
+  We shall also need to be able to convert between elements of $\ran f_P$ and
+  binary strings.  We shall assume that there is a `natural' mapping between
+  the integers $\{0, 1, \ldots, |{\ran f_P}| - 1\}$ and the elements of $\ran
+  f_P$, and omit explicit conversions.
+
+  The inverter algorithm is shown in figure~\ref{fig:pss-inverter}.
+
+  \begin{figure}
+  \begin{program}
+    Inverter $I(P, y)$: \+ \\
+      $\Xid{H}{map} \gets \emptyset$; \\
+      $\Xid{H'}{map} \gets \emptyset$; \\
+      $\Xid{I}{map} \gets \emptyset$; \\
+      $\Xid{m}{list} \gets \emptyset$; \\
+      $(m, \sigma) \gets A^{\id{sign}(\cdot), H(\cdot), H'(\cdot)}(P)$; \\
+      \IF $m \in \Xid{m}{list}$ \THEN \ABORT; \\
+      $y' \gets f_P(\sigma)$; \\
+      \PARSE $y'$ \AS $z \cat k\colon h \cat 8n - k\colon g$; \\
+      \PARSE $g \xor H'(h)$ \AS $k\colon r \cat z'$; \\
+      $x \gets m \cat r$; \\
+      \IF $z \ne 0 \lor z' \ne 0$ \THEN \ABORT; \\
+      \IF $h \ne H(x)$ \THEN \ABORT; \\
+      \IF $x \notin \dom \Xid{I}{map}$ \THEN \ABORT; \\
+      \RETURN $\id{sr-solve}(\Xid{I}{map}(x), \sigma)$;
+  \newline
+    Oracle $\id{sign}(m)$: \+ \\
+      $r \getsr \{0, 1\}^k$; \\
+      $x \gets m \cat r$; \\
+      \IF $x \in \dom \Xid{H}{map}$ \THEN \ABORT; \\
+      \REPEAT \\ \quad \= \+ \kill
+        $\sigma' \getsr \ran f_P$; \\
+        $y' \gets f_P(\sigma')$; \- \\
+      \UNTIL $y' < 2^{8n}$; \\
+      \PARSE $y'$ \AS $z \cat k\colon h \cat 8n - k\colon g$; \\
+      \PARSE $g$ \AS $k\colon r' \cat g'$; \\
+      $h' \gets (r \xor r') \cat g'$; \\
+      \IF $h \in \dom \Xid{H'}{map}$ \THEN \ABORT; \\
+      $\Xid{H}{map} \gets \Xid{H}{map} \cup \{x \mapsto h\}$; \\
+      $\Xid{H'}{map} \gets \Xid{H'}{map} \cup \{h \mapsto h'\}$; \\
+      $\Xid{m}{list} \gets \Xid{m}{list} \cup \{m\}$; \\
+      \RETURN $\sigma'$;
+  \next
+    Oracle $H(x)$: \+ \\
+      \IF $x \in \dom \Xid{H}{map}$ \THEN \RETURN $\Xid{H}{map}(x)$; \\
+      \REPEAT \\ \quad\=\+\kill
+        $(s, y') \gets \id{sr-choose}(P, y)$; \\
+        \PARSE $x$ \AS $m \cat k\colon r$; \- \\
+      \UNTIL $y' < 2^{8n}$; \\
+      \PARSE $y'$ \AS $z \cat k\colon h \cat 8n - k\colon g$; \\
+      \PARSE $g$ \AS $k\colon r' \cat g'$; \\
+      $h' \gets (r \xor r') \cat g'$; \\
+      \IF $h \in \dom \Xid{H'}{map}$ \THEN \ABORT; \\
+      $\Xid{H}{map} \gets \Xid{H}{map} \cup \{x \mapsto h\}$; \\
+      $\Xid{H'}{map} \gets \Xid{H'}{map} \cup \{h \mapsto h'\}$; \\
+      $\Xid{I}{map} \gets \Xid{I}{map} \cup \{x \mapsto s\}$; \\
+      \RETURN $h$; \- \\[\smallskipamount]
+    Oracle $H'(x)$: \+ \\
+      \IF $x \in \dom \Xid{H'}{map}$ \THEN \RETURN $\Xid{H'}{map}(x)$; \\
+      $h' \gets \{0, 1\}^{8n - k}$ \\
+      $\Xid{H'}{map} \gets \Xid{H'}{map} \cup \{h \mapsto h'\}$; \\
+      \RETURN $h'$;
+  \end{program}
+  \caption{From the security proof of PSS -- the inverter for the trapdoor
+    one-way function}
+  \label{fig:pss-inverter}
+  \end{figure}
+  
+  The oracle $H$ constructs related instances of the original inversion
+  problem and uses them to build the random oracle results.  Because the
+  subroutine \id{sr-choose} selects problem instances uniformly over $\{0,
+  1, \ldots, |{\ran f_P}| - 1\}$, and we iterate until $y' < 2^{8n}$, the
+  selected instance $y'$ is uniform over $\{0, 1\}^{8n}$.  We then construct
+  $H$ and $H'$ replies as appropriate for the adversary to construct $y'$ as
+  a message representative for its chosen message $m$, such that if it
+  responds with a signature for $m$ then we can use \id{sr-solve} to solve
+  our initial problem.
+  
+  The oracle \id{sign} chooses oracle responses in order to fit in with a
+  previously chosen inverse (computed the easy way, by choosing a point in
+  the domain of $f_P$ and computing its image).
+
+  The oracle $H'$ just chooses random values.
+  
+  Most of the \ABORT statements in the main inverter routine detect incorrect
+  signatures.  The final one, asserting $x \notin \Xid{I}{map}$, can't happen
+  unless the signaure is a duplicate of one we already gave.
+  
+  The \ABORT{}s in $H$ and \id{sign} detect conditions in which the
+  adversary has successfully distinguished its simulated environment from
+  that of the standard forging experiment.
+  
+  The inverter succeeds whenever a correct forgery is made.  To conclude the
+  proof, we must bound the probability that an \ABORT occurs in $H$ or
+  \id{sign}.
+  
+  The \ABORT in $H$ occurs when the chosen $H(x)$ collides with a value for
+  which $H'$ is already defined.  Since $H(x)$ values are chosen uniformly
+  from $\{0, 1\}^k$, this occurs with probability no more than $(q_H +
+  q_{H'} + 2)/2^k$.
+
+  The first \ABORT in \id{sign} occurs when the chosen point $x = m \cat r$
+  already has an image defined in $H$: this happens with probability at most
+  $q_H/2^k$.  The second occurs when $h$ already has an image in $H'$, and
+  occurs with probability at most $(q_H + q_{H'})/2^k$.
+  
+  The loops in $H$ and \id{sign} don't complete in a deterministic amount
+  of time.  But we can abort if they appear to be taking too long, and choose
+  the maximum number of iterations to achieve any given failure probability.
+  Let $d = |{\dom f_P}|$.  Then the probability that any particular iteration
+  fails to select an appropriate problem instance is $1 - 2^{8n}/d$.  For RSA
+  or Rabin with a modulus bit length which is a multiple of 8, this is less
+  than $\frac{1}{2}$.  We choose the number of iterations in the entire
+  program to be such that we have a $2^{-k}$ probability of failure.  Thus,
+  we set our maximum as $-k/\log_2 (1-2^{8n}/d)$ iterations in the entire
+  program.
+
+  Fitting all of this together, we see that
+  \[ \Succ{owf}{\mathcal{T}}(I) \ge
+       \Succ{euf-cma}{\Xid{\mathcal{S}}{PSS}^{\mathcal{T}}} -
+       \frac{3(q_H + q_{H'} + 2)}{2^k}. \]%
+  completing the proof.
+\end{proof}
+
+\endinput
+
+%%% Local Variables: 
+%%% mode: latex
+%%% TeX-master: "ips"
+%%% End: 
diff --git a/basics.tex b/basics.tex
new file mode 100644 (file)
index 0000000..6d5be48
--- /dev/null
@@ -0,0 +1,1115 @@
+\xcalways\section{Basics}\x
+
+\xcalways\subsection{Introduction and motivation}\x
+
+\begin{slide}
+  \topic{joke}
+  \head{Mandatory joke}
+  
+  An elderly Frenchman rises every morning at 5, goes out to the street in
+  front of his house, and sprinkles a white powder up and down the street.
+  
+  One day, a neighbour, who has watched his routine for many years, confronts
+  him.  `What is this powder you sprinkle on the street every morning,
+  Pierre?'
+  
+  `It is elephant powder, \emph{mon ami},' the gentleman says.  `It keeps the
+  elephants away.'
+  
+  `But Pierre,' says the neighbour.  `Everybody knows that there are no
+  elephants in France.'
+  
+  Says the older man matter-of-factly, `I guess it must be working, then.'
+\end{slide}
+
+The joke has its mandatory corresponding serious message.  Cryptography is
+like elephant powder.  If it seems to work, and keeps the attackers -- our
+elephants -- away, then we trust it.  This isn't really very satisfactory.
+When the elephant powder fails, a 2-ton elephant turns up, with three of its
+chums in a Mini, and you notice it hiding upside-down in your bowl of
+custard.  Let's face it: elephants aren't good at being surreptitious.
+
+But if your cryptography is no good, you may never know.
+
+\begin{slide}
+  \topic{serious message}
+  \head{Cryptography is elephant powder}
+
+  So, what can we do about the situtation?
+  \begin{itemize}
+  \item Design simple cryptographic primitives, e.g., block ciphers, hash
+    functions.  Develop techniques for analysing them, and attempt to stay
+    ahead of the bad guy.
+  \item Build useful constructions from trusted primitives in a modular way.
+    \emph{Prove that the constructions are secure.}
+  \end{itemize}
+  Here we look at this second part of the approach.
+\end{slide}
+
+\xcalways\subsection{Approach}\x
+
+\begin{slide}
+  \head{The provable security approach}
+
+  \begin{itemize}
+  \item Define notions of security.  Now we know what to aim for, and what to
+    expect of our components.
+  \item Prove how the security of a construction relates to the security of
+    its primitives.  If it breaks, we know who to blame.
+  \end{itemize}
+\end{slide}
+
+\begin{slide}
+  \topic{adversaries}
+  \head{Adversaries}
+
+  We model our adversary as a \emph{probabilistic algorithm}; i.e., the
+  algorithm is allowed to \emph{flip coins} to make decisions.  The
+  adversary's output (for some input) follows a probability distribution.  We
+  define what it means for an adversary to \emph{break} our construction, and
+  examine the probability with which this happens.
+
+  We provide the adversary with \emph{oracles}:
+  \begin{itemize}
+  \item Oracles compute using secrets hidden from the adversary.
+  \item We count the number of \emph{queries} made to an oracle.
+  \item We can restrict the types of queries the adversary makes.
+  \end{itemize}
+
+  Oracles are written as superscripts.  For example, an adversary given a
+  chosen-plaintext oracle might be written as $A^{E_K(\cdot)}$.
+\end{slide}
+
+\begin{slide}
+  \topic{the asymptotic approach}
+  \head{The asymptotic approach, 1}
+  
+  A function $\nu\colon \N \to \R$ is \emph{negligible} if, for any integer
+  $c$, there exists an $n \in \N$ such that $0 \le \nu(k) < k^{-c}$ for all
+  $k \ge n$.  That is, $\nu(k)$ is `eventually' less than any polynomial
+  function of $k$.
+  
+  We examine families of constructions, with a \emph{security parameter} $k$.
+  We say that a function is (asymptotically) secure in some sense if, for any
+  polynomial $p(k)$ there is a negligible function $\nu(k)$ such that, for
+  any construction in the family, parameterized by $k$, no adversary which
+  runs for time $p(k)$ has success probability better than $\nu(k)$.
+\end{slide}
+
+\begin{slide}
+  \head{The asymptotic approach, 2}
+  
+  Suppose we build an encryption scheme from a one-way function.  We'd like
+  to prove that the encryption is good if the one-way function is secure.  We
+  do this by contradiction:
+  \begin{enumerate}
+  \item Suppose an adversary $A$ breaks the encryption scheme with
+    better-than-negligible probability.
+  \item Show a polynomial-time \emph{reduction}: an algorithm which uses $A$
+    to break the one-way function, in polynomial time, and with
+    better-than-negligible probability,
+  \item Claim that this violates the assumption of a secure one-way function.
+  \end{enumerate}
+  
+  This doesn't work with real constructions.  We don't know where the
+  asymptotics set in, and they can conceal large constants.  It's still
+  better than nothing.
+\end{slide}
+
+\begin{slide}
+  \topic{the concrete (quantitative) approach}
+  \head{The concrete (quantitative) approach}
+
+  We constrain the resources we allow the adversary:
+  \begin{itemize}
+  \item Running time (including program size).
+  \item Number of oracle queries.
+  \item Maximum size of oracle queries.
+  \end{itemize}
+  Write that something is \emph{$(t, q, \epsilon)$-secure} if no adversary
+  which runs in time $t$ and makes $q$ oracle queries can break it with
+  probability better than $\epsilon$.
+  
+  We make statements like `foo is $(t, q, 2 q \epsilon)$-secure if bar is $(t
+  + O(q), 2 q, \epsilon)$-secure'. 
+  
+  This is a much more satisfactory approach.  However, we still have to
+  \emph{assume} the security of our primitive operations.
+\end{slide}
+
+\xcalways\subsection{Notation}\x
+
+\begin{slide}
+  \topic{boolean operators}
+  \head{Notation, 1: boolean operators}
+
+  If $P$ and $Q$ are \emph{predicates} -- i.e., either true or false -- then:
+  \begin{itemize}
+  \item $P \land Q$ is true if both $P$ \emph{and} $Q$ is true;
+  \item $P \lor Q$ is true if either $P$ \emph{or} $Q$ (or both) is true;
+  \item $\lnot P$ is true if $P$ is false; and
+  \item $P \implies Q$ is true if $Q$ is true or $P$ is false.
+  \end{itemize}
+\end{slide}
+
+\begin{slide}
+  \topic{sets}
+  \head{Notation, 2: sets}
+
+  For our purposes, we can think of sets as being collections of objects.
+  
+  We use the usual notations for set membership ($x \in X$), intersection ($X
+  \cap Y$), union ($X \cup Y$) and subset containment ($X \subseteq Y$).  The
+  \emph{empty set}, which contains no elements, is written $\emptyset$.
+  
+  The notation $\{f(x) \mid P(x)\}$ describes the set containing those items
+  $f(x)$ for which the predicate $P(x)$ is true.
+
+  The \emph{cardinality} $|X|$ of a (finite) set $X$ is the number of
+  elements in the set.
+  
+  The power set $\powerset(X)$ of a set $X$ is the set of all subsets of $X$.
+\end{slide}
+
+\begin{slide}
+  \head{Notation, 3: sets (cont.)}
+
+  The \emph{cartesian product} of two sets $X \times Y$ is the set of all
+  ordered pairs $\{ (x, y) \mid x \in X \land y \in Y \}$.  We use exponents
+  to indicate the product of a set with itself: hence, $X^2 = X \times X$.
+  
+  A \emph{relation} $R$ is a subset of a cartesian product.  We write $R(x,
+  y)$ if $(x, y) \in R$.  Relations between two sets are often written as
+  infix symbols: e.g., $x \sim y$.
+\end{slide}
+  
+\begin{slide}
+  \topic{functions}
+  \head{Notation, 4: functions}
+
+  A \emph{function} $f\colon X \to Y$ is a mapping which assigns every
+  element $x$ in the \emph{domain} $X$ a corresponding element $f(x)$ in the
+  \emph{range} (or sometimes \emph{codomain}) $Y$.  The notation $\dom f$
+  describes the domain of an arbitrary function; $\ran f$ describes its
+  range.
+
+  We sometimes apply the function notation to sets, indicating that the
+  function should be applied pointwise; i.e., $f(Z) = \{ f(z) \mid z \in Z
+  \}$.  The \emph{image} of a function $f$ is the set $f(\dom f)$.
+  
+  If $f\colon X \to Y$ preserves equality, i.e., $f(x) = f(x') \implies x =
+  x'$ for all $x, x' \in X$, then we say $f$ is \emph{injective} (or
+  \emph{1-to-1}).  If $f(X) = Y$ then we say that it is \emph{surjective} (or
+  \emph{onto}).  If $f$ is both injective and surjective then we say that it
+  is \emph{bijective}.  In this case, there is a well-defined inverse
+  $f^{-1}\colon Y \to X$ defined by $f(f^{-1}(y)) = y$ for all $y \in Y$.
+\end{slide}
+
+\begin{slide}
+  \head{Notation, 5: functions (cont.)}
+
+  We can consider a function $f\colon X \to Y$ to be a particular sort of
+  relation $f \subseteq X \times Y$, subject to the constraint that if $(x,
+  y) \in f$ and $(x, y') \in f$ then $y = y'$.
+
+  We shall use this view in some of the algorithms we present.  In addition,
+  we shall use the \emph{maplet} notation $x \mapsto y$ rather than the
+  ordered pair notation $(x, y)$ to reinforce the notion of a mapping.
+
+  We might write, for example,
+  \begin{program}
+    $f \gets f \cup \{ x \mapsto y \}$;
+  \end{program}
+  to augment a function by the addition of a new mapping.  This is clearly
+  only valid if $x \notin \dom f$ (or $f(x) = y$) initially.
+\end{slide}
+
+\begin{slide}
+  \topic{strings}
+  \head{Notation, 6: strings}
+  
+  An \emph{alphabet} is a finite set of \emph{symbols}.  The one we'll use
+  most of the time is the set $\Sigma = \{0, 1\}$ of \emph{bits}.
+  
+  Suppose $A$ is an alphabet.  The set of sequences of exactly $n$ symbols
+  from $A$ is written $A^n$.  Hence, $\{0, 1\}^{64}$ is the set of all 64-bit
+  sequences.  The set of (finite) \emph{strings} over an alphabet $A$ is $A^*
+  = \bigcup_{i \in \N} A^n$.  The empty string is named $\emptystring$.
+  
+  The \emph{length} of a string $a \in A^*$, written $|a|$, is the natural
+  number $n$ where $a \in A^n$.
+  
+  If $x, y \in A^*$ are strings then their \emph{concatenation}, written $x
+  \cat y$, or sometimes just $x y$, is the result of writing the symbols in
+  $x$ followed by the symbols in $y$, in order.  We have $|x y| = |x| + |y|$.
+\end{slide}
+
+\begin{slide}
+  \head{Notation, 7: strings (cont.)}
+  
+  There are natural (bijective) mappings between bit strings and natual
+  numbers.
+  
+  If $x = x_0 x_1 \ldots x_{n-1} \in \{0, 1\}^*$ then we can associate with
+  it the natural number
+  \[ \overrightarrow{x} = \sum_{0 \le i < n} 2^i x_i. \]
+
+  The other natural mapping is
+  \[ \overleftarrow{x} = \sum_{0 \le i < n} 2^{n-i-1} x_i. \]
+  It doesn't matter which you choose, as long as you're consistent.
+  
+  For simplicity's sake, we shall tend to switch between strings and the
+  numbers (and occasionally more exotic objects) they represent implictly.
+\end{slide}
+
+\begin{slide}
+  \topic{parsing}
+  \head{Notation, 8: parsing}
+
+  We'll find it useful to be able to break up strings in the algorithms we
+  present.  We use the statement
+  \begin{program}
+    \PARSE $x$ \AS $n_0\colon x_0, n_1\colon x_1, \ldots, n_k\colon x_k$;
+  \end{program}
+  to mean that the string $x$ is broken up into individual strings $x_0$,
+  $x_1$, \ldots, $x_k$, such that
+  \begin{itemize}
+  \item $x = x_0 \cat x_1 \cat \cdots \cat x_k$; and
+  \item $|x_i| = n_i$ for all $0 \le i \le k$.
+  \end{itemize}
+  We may omit one of the $n_i$ length indicators, since it can be deduced
+  from the length of the string $x$ and the other $n_j$.
+\end{slide}
+
+\begin{slide}
+  \topic{vectors}
+  \head{Notation, 9: vectors}
+  
+  A \emph{vector} $\vect{x}$ is a finite ordered collection of elements from
+  some set $X$.  If $\vect{x}$ contains $n$ elements then we write $\vect{x}
+  \in X^n$, and that $|\vect{x}| = n$.  We write the individual elements as
+  $\vect{x}[0], \vect{x}[1], \ldots, \vect{x}[n - 1]$.
+
+  We shall use set membership notation for vectors; i.e., we write $x \in
+  \vect{x}$ if there is an $i$ ($0 \le i < |\vect{x}|$) such that
+  $\vect{x}[i] = x$.
+
+  When we apply functions to vectors, we mean that they are applied
+  pointwise, as for sets.  Thus, if we write that $\vect{y} =
+  f(\vect{x})$ then $|\vect{y}| = |\vect{x}|$ and $\vect{y}[i] =
+  f(\vect{x}[i])$ for all $0 \le i < |\vect{x}|$.
+\end{slide}
+
+\begin{slide}
+  \topic{distributions and randomness}
+  \head{Notation, 10: distributions and randomness}
+  
+  A \emph{probability distribution} over a (countable) set $X$ is a
+  function $\mathcal{D}\colon X \to [0, 1]$ such that
+  \[ \sum_{x \in X} \mathcal{D}(x) = 1. \]
+  
+  The \emph{support} of $\mathcal{D}$, written $\supp \mathcal{D}$, is the
+  set $\{ x \in X \mid \mathcal{D}(x) \ne 0 \}$; i.e., those elements of $X$
+  which occur with nonzero probability.
+  
+  We write $x \getsr \mathcal{D}$ in algorithms to indicate that $x$ is to be
+  chosen independently at random, according to the distribution
+  $\mathcal{D}$.  The notation $x \inr \mathcal{D}$ indicates that $x$ has
+  been chosen at independently at random according to $\mathcal{D}$.
+  
+  The \emph{uniform distribution} over a (finite) set $X$ is
+  $\mathcal{U}_X\colon X \to [0, 1]$ defined by $\mathcal{U}_X(x) = 1/|X|$
+  for all $x \in X$.  We shall write $x \getsr X$ and $x \inr X$ as
+  convenient shorthands, meaning $x \getsr \mathcal{U}_X$ and $x \inr
+  \mathcal{U}_X$ respectively.
+\end{slide}
+
+\xcalways\subsection{Background}\x
+
+\begin{slide}
+  \topic{distinguishability}
+  \head{Distinguishability, 1}
+
+  Suppose that $\mathcal{X}$ and $\mathcal{Y}$ are two probability
+  distributions.
+
+  Let $X$ be a random variable distributed according to $\mathcal{X}$, and
+  let $Y$ be a random variable distributed according to $\mathcal{Y}$.  We
+  say that $\mathcal{A}$ and $\mathcal{B}$ are \emph{identically
+  distributed} if, for all possible values $x$ of $X$, we have
+  \[ \Pr[X = x] = \Pr[Y = x]. \]
+
+  Equivalently, we require that, for all $x \in \supp \mathcal{X}$ we have
+  \[ x \in \supp \mathcal{Y} \land \mathcal{X}(x) = \mathcal{Y}(y). \]
+\end{slide}
+
+\begin{slide}
+  \head{Distinguishability, 2: statistical closeness}
+  
+  Now we generalize the setting slightly.  Consider two \emph{families} of
+  distributions, $\{\mathcal{X}_k\}_{k \in \N}$ and $\{\mathcal{Y}_k\}_{k \in
+    \N}$, parameterized by a security parameter $k$.
+  
+  Fix a value of $k$.  Again, let $X$ be distributed according to
+  $\mathcal{X}_k$, and let $Y$ be distributed according to $\mathcal{Y}_k$.
+  We say that $\{\mathcal{X}_k\}_{k \in \N}$ and $\{\mathcal{Y}_k\}_{k \in
+    \N}$ are \emph{statistically close} if there is a negligible function
+  $\nu(\cdot)$ such that for any $k \in \N$, and for all $x \in
+  \supp \mathcal{X}_k$,
+  \[ |{\Pr[X = x]} - \Pr[Y = x]| \le \nu(k). \]
+  
+  (Reminder: Saying that $\nu\colon \N \to \R$ is \emph{negligible} means
+  that, for any $c \in \Z$ there is an $n \in N$ such that $\nu(k) <
+  k^{-c}$.)
+\end{slide}
+
+\begin{slide}
+  \head{Distinguishability, 3: computational indistinguishability}
+
+  We say that two families of distributions are computationally
+  indistinguishable if no `efficient' algorithm can tell them apart with
+  better than `negligible' probability.
+  
+  So, we say that $\{\mathcal{X}_k\}_{k \in \N}$ and $\{\mathcal{Y}_k\}_{k
+    \in \N}$ are \emph{computationally indistinguishable} if, for any
+  probabilistic polynomial-time algorithm $A$, there is a negligible function
+  $\nu(\cdot)$ such that, for any $k$:
+  \[ \Pr[x \getsr \mathcal{X}_k; b \gets A(x) : b = 1] -
+     \Pr[x \getsr \mathcal{Y}_k; b \gets A(x) : b = 1] \le \nu(k). \]%
+\end{slide}
+
+\begin{slide}
+  \topic{collisions}
+  \head{Collisions}
+  
+  Suppose we throw $q$ balls into $n$ bins at random (with $q \le n$).  Let
+  $C_{q, n}$ be the event that, at the end of this, we have a bin containing
+  more than one ball -- a \emph{collision}.
+  
+  Let $B_{q, n}$ be the event that the $q$-th ball collides with a
+  previous one.  Obviously, the worst case for this is when none of the other
+  balls have collided, so
+  \[ \Pr[B_{q, n}] \le \frac{q - 1}{n}. \]
+  Then
+  \begin{eqnarray*}[rl]
+    \Pr[C_{q, n}]
+    &\le \Pr[B_{2, n}] + \Pr[B_{3, n}] + \cdots + \Pr[B_{q, n}] \\
+    &= \frac{1}{n} + \frac{2}{n} + \cdots + \frac{q - 1}{n} \\
+    &= \frac{q(q - 1)}{2 n}.
+  \end{eqnarray*}
+  This is an extremely useful result, and we shall need it often.
+\end{slide}
+
+\xcalways\subsection{Primitives}\x
+
+\begin{slide}
+  \topic{one-way functions}
+  \head{One-way functions, 1: introduction}
+
+  Intuition: a one-way function is easy to compute but hard to invert.
+
+  Choose a function $f\colon X \to Y$.  Let $I$ be a prospective inverter.
+  Now we play a game:
+  \begin{enumerate}
+  \item Choose $x \in X$ at random.  Compute $y = f(x)$.
+  \item Let $x'$ be the output of $I$ when run on input $y$.
+  \item We say that $I$ `wins' if $f(x') = y$; otherwise it loses.
+  \end{enumerate}
+  Note that we don't care whether $x = x'$.
+
+  Examples: SHA-1, or $x \mapsto g^x \bmod p$.
+\end{slide}
+
+\begin{slide}
+  \head{One-way functions, 2: formalism}
+
+  The \emph{success probability} of an inverter $I$ against the function $f$
+  is
+  \[ \Succ{owf}{f}(I) =
+     \Pr[x \getsr X;
+         y \gets f(x);
+         x' \gets I(y) :
+         f(x') = y] \]%
+  where the probability is taken over all choices of $x$ and all coin flips
+  made by $I$.
+  
+  We measure the \emph{insecurity} of a one-way function (OWF) by maximizing
+  over possible inverters:
+  \[ \InSec{owf}(f; t) = \max_I \Succ{owf}{f}(I) \]
+  where the maximum is taken over all $I$ running in time $t$.
+
+  If $\InSec{owf}(f; t) \le \epsilon$ then we say that $f$ is a \emph{$(t,
+  \epsilon)$-secure one-way function}.
+\end{slide}
+
+\begin{slide}
+  \head{One-way functions, 3: trapdoors}
+  
+  Intuition: a \emph{trapdoor} is secret information which makes inverting a
+  one-way function easy.  A trapdoor one-way function generator $\mathcal{T}
+  = (G, f, T)$ is a triple of algorithms:
+
+  \begin{itemize}
+  \item The probabilistic algorithm $G$ is given some parameter $k$ and
+    returns a pair $(P, K)$, containing the public parameters $P$ for
+    computing an instance of the one-way function, and the secret trapdoor
+    information $K$ for inverting it.  We write $(P, K) \in G(k)$.
+    
+  \item The algorithm $f$ implements the one-way function.  That is, if $(P,
+    K) \in G(k)$ then $f(P, \cdot)$ (usually written $f_P(\cdot)$) is a
+    one-way function.
+    
+  \item The algorithm $T$ inverts $f$ using the trapdoor information.  That
+    is, if $(P, K) \in G(k)$ and $y = f_P(x)$ for some $x$, then $x =
+    T(K, y)$.  We usually write $T_K(\cdot)$ instead of $T(K, \cdot)$.
+  \end{itemize}
+\end{slide}
+
+\begin{slide}
+  \topic{pseudorandom generators (PRGs)}
+  \head{Pseudorandom generators (PRGs)}
+  
+  A pseudorandom generator (PRG) `stretches' an input seed into a longer
+  string which `looks' random.
+  
+  Let $G\colon \{0, 1\}^k \to \{0, 1\}^L$ be a function from $k$-bit strings
+  to $L$-bit strings.  The \emph{advantage} of a distinguisher $D$ against
+  $G$ is:
+  \begin{eqnarray*}[rl]
+    \Adv{prg}{G}(D) = &
+       \Pr[x \getsr \{0, 1\}^k; y \gets G(x) : D(y) = 1] - {}\\
+    &  \Pr[y \getsr \{0, 1\}^L : D(y) = 1].
+  \end{eqnarray*}
+  The \emph{insecurity} is simply the maximum advantage:
+  \[ \InSec{prg}(G; t) = \max_D \Adv{prg}{G}(D) \]
+  where the maximum is taken over all distinguishers $D$ running in time
+  $t$.  If $\InSec{prg}(G; t) \le \epsilon$ then we also say that $G$ is a
+  $(t, \epsilon)$-secure PRG\@.
+\end{slide}
+
+\begin{exercise}
+  We say that a PRG $g\colon \{0, 1\}^k \to \{0, 1\}^L$ is \emph{trivial} if
+  $k \ge L$.
+  \begin{enumerate}
+  \item Show that trivial PRGs exist.
+  \item Show that if $g$ is nontrivial, then $g$ is also a one-way function,
+    with
+    \[ \InSec{owf}(g; t) \le \InSec{prg}(g; t) + 2^{k-L}. \]
+  \end{enumerate}
+  \answer%
+  \begin{parenum}
+  \item The identity function $I(x) = x$ is a trivial PRG, with
+    $\InSec{prg}(I, t) = 0$, as is easily seen from the definition.
+  \item Suppose $A$ inverts $g$.  Then consider adversary $B(y)$: \{ $x \gets
+    A(y)$; \IF $g(x) = y$ \THEN \RETURN $1$; \ELSE \RETURN $0$;~\}.  If $y$
+    is the output of $g$, then $A$ inverts $y$ with probability
+    $\Succ{owf}{g}(A)$; if $y$ is random in $\{0, 1\}^L$ then there is a
+    probability at least $1 - 2^{k-L}$ that $y$ has \emph{no} inverse,
+    proving the result.  Note that \cite{Wagner:2000:PSU} provides a better
+    security bound than this simplistic analysis.
+  \end{parenum}
+\end{exercise}
+
+\begin{exercise}
+  \label{ex:dbl-prg}
+  Suppose that we have a \emph{length-doubling} PRG $g\colon \{0, 1\}^k \to
+  \{0, 1\}^{2k}$.  Let $g_0(\cdot)$ be the first $k$ bits of $g(x)$ and
+  $g_1(\cdot)$ be the second $k$ bits.  Define the sequence of generators
+  $g^{(i)}$ (for $i >= 1$) by
+  \[ g^{(1)}(x) = g(x); \qquad
+     g^{(i+1)}(x) = g_0(x) \cat g^{(i)}(g_1(x)). \]%
+  Relate the security of $g^{(i)}$ to that of $g$.
+  \answer%
+  Let $A$ be an adversary running in time $t$ and attacking $g^{(i+1)}$.
+  Firstly, we attack $g$: consider adversary $B(y)$: \{ \PARSE $y$ \AS $y_0,
+  k\colon y_1$; $z \gets g^{(i)}$; $b \gets A(y_0 \cat z)$; \RETURN $b$;~\}.
+  Then $\Adv{prg}{g}(B) \ge \Adv{prg}{g^{(i+1)}}(A) + \delta$, so
+  $\InSec{prg}(g^{(i+1)}; t) \le \InSec{prg}(g; t) + \delta$, where
+  \begin{eqnarray*}[rl]
+    \delta = &\Pr[x_0 \gets \{0, 1\}^k; x_1 \gets \{0, 1\}^k;
+                  y \gets g^{(i)}(x) : A(x_0 \cat y) = 1] - \\
+              &\Pr[y \getsr \{0, 1\}^{(i+2)k} : A(y) = 1].
+  \end{eqnarray*}
+  We attack $g^{(i)}$ to bound $\delta$: consider adversary $C(y)$: \{ $x_0
+  \getsr \{0, 1\}^k$; $b \gets A(x_0 \cat y)$; \RETURN $b$;~\}.  Now $\delta
+  \le \Adv{prg}{g^{(i)}}(C) \le \InSec{prg}(g^{(i)}; t)$.  So by induction,
+  \[ \InSec{prg}(g^{(i)}; t) \le i \cdot \InSec{prg}(g; t). \]
+\end{exercise}
+
+\begin{slide}
+  \topic{advantage}
+  \head{Notes about advantage}
+
+  Advantage is a concept used in many definitions of security notions:
+  \begin{eqnarray*}[rl]
+     \Adv{}{}(A) = &
+        \Pr[\text{$A$ returns 1 in setting $a$}] - {} \\
+     &  \Pr[\text{$A$ returns 1 in setting $b$}].
+  \end{eqnarray*}
+  \begin{enumerate}
+  \item We have $-1 \le \Adv{}{}(A) \le +1$.
+  \item Zero means that the adversary couldn't distinguish.
+  \item Negative advantage means the adversary got them the wrong way
+    around.  There is another adversary which uses the same resources but has
+    positive advantage.
+  \item \label{item:adv-guess} If $A$ is attempting to guess some hidden bit
+    $b^* \inr \{0, 1\}$, we have
+    \[ \Pr[b \gets A : b = b^*] = \frac{\Adv{}{}(A)}{2} + \frac{1}{2}. \]
+  \end{enumerate}
+\end{slide}
+
+\begin{proof}
+  Let $b$ be the bit that $A$ returns, and let $b^*$ be the experiment's
+  hidden bit.  Then
+  \[ \Adv{}{}(A) = \Pr[b = 1 \mid b^* = 1] - \Pr[b = 1 \mid b^* = 0]. \]
+  Addressing the above claims in order:
+  \begin{enumerate}
+  \item By definition of probability, $0 \le \Pr[b = 1 \mid b^* = 1] \le 1$
+    and $0 \le \Pr[b = 1 \mid b^* = 0]$, so their absolute difference can be
+    at most 1.
+  \item This is a corollary of \ref{item:adv-guess}.
+  \item Consider the adversary $\bar{A}$ which runs $A$ and returns the
+    complement bit $\bar{b} = b \xor 1$.  Then
+    \begin{eqnarray*}[rl]
+      \Adv{}{}(\bar{A})
+      &= \Pr[\bar{b} = 1 \mid b^* = 1] - \Pr[\bar{b} = 1 \mid b^* = 0] \\
+      &= \Pr[b = 0 \mid b^* = 1] - \Pr[b = 0 \mid b^* = 0] \\
+      &= (1 - \Pr[b = 1 \mid b^* = 1]) - (1 - \Pr[b = 1 \mid b^* = 0]) \\
+      &= \Pr[b = 1 \mid b^* = 0] - \Pr[b = 1 \mid b^* = 1] \\
+      &= -\Adv{}{}(A).
+    \end{eqnarray*}
+  \item Note that $\Pr[b^* = 1] = \Pr[b^* = 0] = \frac{1}{2}$.  Then
+    \begin{eqnarray*}[rl]
+      \Pr[b = b^*]
+      &= \Pr[b = 1 \land b^* = 1] + \Pr[b = 0 \land b^* = 0] \\
+      &= \frac{1}{2}(\Pr[b = 1 \mid b^* = 1] + \Pr[b = 1 \mid b^* = 0]) \\
+      &= \frac{1}{2}(\Pr[b = 1 \mid b^* = 1] +
+                     (1 - \Pr[b = 0 \mid b^* = 0])) \\
+      &= \frac{1}{2}(1 + \Pr[b = 1 \mid b^* = 1] -
+                           \Pr[b = 0 \mid b^* = 0]) \\
+      &= \frac{\Adv{}{}(A)}{2} + \frac{1}{2}.
+    \end{eqnarray*}
+  \end{enumerate}
+  All present and correct.
+\end{proof}
+
+\begin{slide}
+  \topic{pseudorandom functions (PRFs)}
+  \head{Pseudorandom functions (PRFs)}
+  
+  A \emph{pseudorandom function family} (PRF) is a collection of functions
+  $F_K\colon \{0, 1\}^l \to \{0, 1\}^L$, where $K$ is some index, typically
+  from a set of fixed-size strings $\{0, 1\}^k$.
+
+  We want to say that $F$ is a strong PRF if adversaries find it hard to
+  distinguish an instance $F_K$ from a function chosen completely at random
+  with the same `shape'.
+
+  We provide the adversary with an \emph{oracle}, either for a randomly
+  selected $F_K$, or for completely random function, and ask it to try and
+  say which it is given.
+
+  We write $\Func{l}{L}$ as the set of \emph{all} functions from $\{0, 1\}^l$
+  to $\{0, 1\}^L$.
+\end{slide}
+
+\begin{slide}
+  \head{Pseudorandomness, 3: PRFs (cont.)}
+
+  We define the advantage of a distinguisher $D$ against the PRF $F$ as
+  follows:
+  \begin{eqnarray*}[rl]
+    \Adv{prf}{F}(D) = &
+       \Pr[K \getsr \{0, 1\}^k : D^{F_K(\cdot)} = 1] - {}\\
+    &  \Pr[R \getsr \Func{l}{L} : D^{R(\cdot)} = 1].
+  \end{eqnarray*}
+  The insecurity of the PRF is then measured as
+  \[ \InSec{prf}(F; t, q) = \max_D \Adv{prf}{F}(D) \]
+  where the maximum is taken over all distinguishers $D$ which run for time
+  $t$ and make $q$ oracle queries.  As is usual, if $\InSec{prf}(F; t, q)
+  \le \epsilon$ then we say that $F$ is a $(t, q, \epsilon)$-secure PRF.
+\end{slide}
+
+\begin{slide}
+  \topic{pseudorandom permutations (PRPs)}
+  \head{Pseudorandom permutations (PRPs)}
+
+  We define a \emph{pseudorandom permutation family} (PRP) in a similar way
+  to the PRFs we've already seen.  Suppose $F_K\colon \{0, 1\}^L \to \{0,
+  1\}^L$ is a family of permutations, indexed by elements of some finite set,
+  e.g., $\{0, 1\}^k$.  Let $\Perm{L}$ be the set of \emph{all} permutations
+  over the set of $L$-bit strings $\{0, 1\}^L$.
+
+  The advantage of a distinguisher $D$ against the PRP $F$ is
+  \begin{eqnarray*}[rl]
+    \Adv{prp}{F}(D) = &
+       \Pr[K \getsr \{0, 1\}^k : D^{F_K(\cdot)} = 1] - {}\\
+    &  \Pr[R \getsr \Perm{L} : D^{R(\cdot)} = 1].
+  \end{eqnarray*}
+
+  We define $\InSec{prp}(F; t, q) = \max_D \Adv{prp}{F}(D)$ exactly as for
+  PRFs, and the notion of $(t, q, \epsilon)$-security is the same.
+\end{slide}
+
+\begin{slide}
+  \head{Pseudorandomness, 5: super PRPs}
+  
+  PRPs are bijective.  A \emph{super PRP} is a PRP which remains secure when
+  the distinguisher is allowed to make inverse queries:
+  \begin{eqnarray*}[rl]
+    \Adv{sprp}{F}(D) = &
+      \Pr[D^{F_K(\cdot), F_K^{-1}(\cdot)} = 1 \mid
+          K \getsr \{0, 1\}^k] - {} \\
+    & \Pr[R \getsr \Perm{L} : D^{R(\cdot), R^{-1}(\cdot)} = 1].
+  \end{eqnarray*}
+  Since there are two oracles, we count queries to both when evaluating the
+  insecurity:
+  \[ \InSec{sprp}(F; t, q, q') = \max_D \Adv{sprp}{F}(D) \]
+  where the maximum is taken over all distinguishers $D$ which run for time
+  $t$, make $q$ queries to the standard oracle, and $q'$ queries to the
+  inverse oracle.  If $\InSec{sprp}(F; t, q, q') \le \epsilon$ then we say
+  $F$ is a $(t, q, q', \epsilon)$-secure super PRP\@.
+\end{slide}
+
+\begin{exercise}
+  Note that the key length hasn't appeared anywhere in the definition of
+  insecurity for a PRP.  Derive lower bounds for the insecurity of a PRP with
+  a $k$-bit key.
+  \answer%
+  Let $E\colon \{0, 1\}^k \times \{0, 1\}^L \to \{0, 1\}^L$ be a PRP.  Fix
+  $n$ and $c$.  Then consider adversary $S^{E(\cdot)}$: \{ \FOR $i = 0$ \TO
+  $c - 1$ \DO $y[i] \gets E(i)$; \FOR $K = 0$ \TO $n - 1$ \DO \{ $i \gets 0$;
+  $\id{good} \gets 1$; \WHILE $i < c \land \id{good} = 1$ \DO \{ \IF $E_K(i)
+  \ne y[i]$ \THEN $\id{good} \gets 0$;~\} \IF $\id{good} = 1$ \THEN \RETURN
+  $1$;~\}~\}.  Then $\Adv{prp}{E}(S) \ge n(2^{-k} - 2^{-Lc})$.
+\end{exercise}
+
+\begin{slide}
+  \head{Block ciphers: PRPs are PRFs}
+
+  We model block ciphers as families of PRPs (not super PRPs).  Most of the
+  analysis works best on PRFs, though.  We show that a PRP makes a `pretty
+  good' PRF, as long as it's not over-used.
+
+  Let $F$ be any PRP family.  Then
+  \[ \InSec{prf}(F; t, q) \le
+     \InSec{prp}(F; t, q) + \frac{q(q - 1)}{2^{L+1}}. \]%
+  This is a useful result.  As long as $q^2$ is small compared to $2^L$ --
+  the block size -- then a PRP makes a good PRF.
+  
+  The value $2^{L/2}$ is often called the \emph{Birthday bound} (after the
+  birthday `paradox').  We shall meet it often when we examine modes of
+  operation.
+\end{slide}
+
+\begin{proof}
+  Let $F$ be a PRP family, and $K$ a randomly chosen key from the keyspace of
+  $F$; let $R \inr \Func{L}{L}$ be a random function, and let $P \inr
+  \Perm{L}$ be a random permutation.
+  
+  Let $A$ be any distinguisher running in time $t$, and making $q$ oracle
+  queries.  Then:
+  \begin{eqnarray*}[rl]
+    \Adv{prf}{F}(A)
+    & = \Pr[A^{F_K(\cdot)} = 1] - \Pr[A^{R(\cdot)} = 1] \\
+    & = \Pr[A^{F_K(\cdot)} = 1] - \Pr[A^{P(\cdot)} = 1] +
+        \Pr[A^{P(\cdot)} = 1] - \Pr[A^{R(\cdot)} = 1] \\
+    & = \Adv{prp}{F}(A) + \Pr[A^{P(\cdot)} = 1] - \Pr[A^{R(\cdot)} = 1] \\
+    & = \Adv{prp}{F}(A) + \Pr[A^{R(\cdot)} = 0] - \Pr[A^{P(\cdot)} = 0].
+  \end{eqnarray*}
+  Now we need to bound the quantity $\Pr[A^{R(\cdot)} = 0] - \Pr[A^{P(\cdot)}
+  = 0]$.  Let $D$ be the event that all of the \emph{distinct} oracle queries
+  to $R$ yield distinct results.  Then
+  \begin{eqnarray*}[rl]
+    \Pr[A^{R(\cdot)} = 0]
+    & = \Pr[A^{R(\cdot)} = 0 \mid D] \Pr[D] +
+        \Pr[A^{R(\cdot)} = 0 \mid \lnot D] \Pr[\lnot D] \\
+    & = \Pr[A^{P(\cdot)} = 0] \Pr[D] +
+        \Pr[A^{R(\cdot)} = 0 \mid \lnot D] \Pr[\lnot D] \\
+    & \le \Pr[A^{P(\cdot)} = 0] + \Pr[\lnot D] \\
+    & \le \Pr[A^{P(\cdot)} = 0] + \frac{q(q - 1)}{2^{L+1}}.
+  \end{eqnarray*}
+  Substituting this into the above equation yields
+  \[ \Adv{prf}{F}(A) \le \Adv{prp}{F}(A) + \frac{q(q - 1)}{2^{L+1}}. \]
+  Select $A$ such that $\Adv{prf}{F}(A) = \InSec{prf}(F; t, q)$.  We know
+  $\Adv{prp}{F}(A) \le \InSec{prp}(F; t, q)$ by definition.  The result
+  follows.
+\end{proof}
+
+\begin{slide}
+  \topic{hash functions}
+  \head{Hash functions, 1: properties}
+
+  Hash functions like MD5 and SHA-1 are extremely useful primitives.  What
+  properties do we expect of them?  This out to be an extremely difficult
+  question to answer.
+  \begin{itemize}
+  \item One-wayness.  We've seen a definition for this already.  But it's not
+    enough.
+  \item Collision-resistance.  This is the property usually claimed as the
+    requirement for use in digital signature systems.  We'll look at this
+    later.
+  \item Randomness.  What does this mean, when the function is completely
+    public?  A distinguishability criterion is clearly hopeless.
+  \end{itemize}
+\end{slide}
+  
+\begin{slide}
+  \head{Hash functions, 2: Merkle-Damg\aa{}rd iterated hashing
+    \cite{Damgaard:1990:DPH, Merkle:1991:FSE}}
+  
+  Let $F\colon \{0, 1\}^{k+L} \to \{0, 1\}^k$ be a \emph{compression}
+  function.  Now consider the function $H\colon \{0, 1\}^* \to \{0, 1\}^k$
+  which transforms an input string $x$ as follows:
+  \begin{enumerate}
+  \item Pad $x$ to a multiple of $L$ bits in some injective way.  Divide the
+    padded message into $L$-bit blocks $x_0$, $x_1$, \ldots, $x_{n-1}$.
+  \item Fix some $L$-bit constant $I$.  Let $I_0 = I$.  Define $I_{i+1} =
+    F(I_i \cat x_i)$ for $0 \le i < n$.
+  \item The result $H(x) = I_n$.
+  \end{enumerate}
+  
+  Suppose we have two strings $x \ne y$, such that $H(x) = H(y)$; i.e., a
+  \emph{collision}.  Then \emph{either} we can find a collision for $F$
+  \emph{or} a string $z$ for which $F(z) = I$.  (This is why initialization
+  vectors for hash functions have such obviously regular forms.)
+\end{slide}
+
+\begin{proof}
+  Let $x_0, x_1, \ldots, x_{n-1}$ and $x'_0, x'_1, \ldots, x'_{n'-1}$ be
+  the $l$-bit blocks of two distinct (padded) messages, and without loss
+  of generality suppose that $n \ge n'$.  Let $I_0 = I'_0 = I$, let
+  $I_{i+1} = F(I_i \cat x_i)$, and $I'_{i+1} = F(I'_i \cat x'_i)$.  We
+  have $I_n = I'_{n'}$.
+  
+  We prove the result by induction on $n$.  The case $n = 0$ is trivially
+  true, since there is only one zero-block message.  Suppose, then, that the
+  result is true for $n$-block messages.  There are three cases to consider.
+  Firstly, if $n' = 0$ then $F(I_n \cat x_n) = I$.  Secondly, if $I_n \ne
+  I'_{n'}$ or $x_n \ne x'_{n'}$, then we have a collision, for $F(I_n \cat
+  x_n) = I_n = I'_{n'} = F(I'_{n'} \cat x'_{n'})$.  Finally, if $I_n =
+  I'_{n'}$ and $x_n = x'_{n'}$ then we remove the final block from both
+  messages, and because the remaining messages must differ in at least one
+  block, we can apply the inductive hypothesis on these shorter messages to
+  complete the proof.
+\end{proof}
+
+\begin{slide}
+  \head{Hash functions, 2: any-collision resistance}
+  
+  The statement usually made about a `good' hash function $h$ is that it
+  should be `difficult' to find a collision: i.e., two preimages $x \ne y$
+  where $H(x) = H(y)$.  How do we formalize this?  Here's one attempt:
+  \begin{eqlines*}
+    \Succ{acr}{H}(A) = \Pr[(x, y) \gets A : x \ne y \land H(x) = H(y)]; \\
+    \InSec{acr}(H; t) = \max_A \Succ{acr}{H}(A).
+  \end{eqlines*}
+  But this doesn't work.  There clearly \emph{exists} an adversary which
+  already `knows' the a collision for $H$ and just outputs the right answer.
+  It succeeds very quickly, and with probability 1.  So this definition is
+  impossible to satisfy.
+\end{slide}
+
+\begin{slide}
+  \head{Hash functions, 3: targetted collision resistance}
+
+  The concept of targetted collision resistance is relatively new, but quite
+  promising.  It replaces a single hash function with a \emph{family} of hash
+  functions.  They're not really keyed, because the indices aren't kept
+  secret.
+
+  When making a signature, an index $i$ is chosen at random, and the
+  signature for message $m$ is formed over the pair $(i, H_i(M))$.
+
+  TCR-hash functions are the subject of ongoing research.  No practical
+  designs exist at the moment.
+\end{slide}
+
+\begin{slide}
+  \head{Hash functions, 4: targetted collision resistance (cont.)}
+
+  Consider the following experiment:
+  \begin{program}
+    $\Expt{tcr}{H}(A)$: \+ \\
+      $(x, s) \gets A(\cookie{find})$; \\
+      $i \getsr \keys H$; \\
+      $y \gets A(\cookie{collide}, i, s)$; \\
+      \IF $x \ne y \land H_i(x) = H_i(y)$
+      \THEN \RETURN $1$; \\
+      \ELSE \RETURN $0$;
+  \end{program}
+  The security of a TCR-hash function is measured as:
+  \[ \InSec{tcr}(H; t) = \max_A \Pr[\Expt{tcr}{H}(A) = 1] \]
+  where the maximum is taken over all adversaries running in time $t$.  We
+  define $(t, \epsilon)$-security as usual.
+\end{slide}
+
+\begin{slide}
+  \head{Hash functions, 5: random oracles \cite{Bellare:1993:ROP}}
+
+  In practice, we expect much more than just collision resistance from hash
+  functions: we expect a certain amount of `random' behaviour.  But this is
+  hard to quantify.
+  
+  One approach is to model a hash function as a `random oracle', i.e., an
+  oracle containing a function chosen at random, used by the construction
+  under consideration, and made available to the adversary.  The idea is that
+  reductions can `capture' the knowledge of an adversary by examining the
+  queries it makes to its random oracle.
+
+  Hash functions \emph{aren't} random oracles.  But a random oracle proof is
+  better than nothing.
+\end{slide}
+
+\xcalways\subsection{Standard assumptions}\x
+
+\begin{slide}
+  \head{Standard assumptions}
+
+  There are a number of `standard' assumptions that are made about the
+  difficulty of various problems:
+  \begin{itemize}
+  \item IFP, the Integer Factorization Problem;
+  \item QRP, the Quadratic Residuosity Problem;
+  \item DLP, the Discrete Logarithm Problem;
+  \item RSAP, the RSA Problem; and
+  \item CDH, the Computational Diffie-Hellman problem and its variants
+  \end{itemize}
+  \cite{Menezes:1997:HAC} has excellent material on the above.
+\end{slide}
+
+\begin{slide}
+  \topic{integer factorization}
+  \head{The Integer Factorization Problem, 1}
+    
+  We often assume that large integers of the form $n = p q$, where $p$ and
+  $q$ are primes of roughly the same size, are `hard' to factor.
+  Specifically, there is no algorithm which will factor such an $n$ in time
+  bounded by a polynomial function of $\log n$.
+
+  The difficulty of various other problems, e.g., Quadratic Residuosity, or
+  RSA, depend on the difficulty of factoring; however, it is not yet known
+  whether the ability to solve QRP or RSAP can be used to factor.
+\end{slide}
+
+\begin{slide}
+  \head{The Integer Factorization Problem, 2: square roots}
+  
+  The problem of extracting square roots modulo $n = p q$ is provably as hard
+  as factoring.  This is the basis of Rabin's public key encryption and
+  digital signature schemes.  We shall analyse these later.
+  
+  Suppose $Q(n, y)$ is an algorithm which returns an $x$ such that $x^2
+  \equiv y \pmod{n}$, provided such an $x$ exists.  Then we can find a
+  nontrivial factor of $n$ as follows:
+  \begin{program}
+    Algorithm $\id{find-factor}(n)$: \+ \\
+      \REPEAT \\ \quad\=\+\kill
+        $x \getsr \{1, 2, \ldots, n - 1\}$; \\
+        $y \gets x^2 \bmod n$; \\
+        $x' \gets Q(n, y)$; \\
+        $p \gets \gcd(x + x', n)$; \\
+        \IF $p \notin \{1, n\}$ \THEN \RETURN $p$; \- \\
+      \FOREVER;
+  \end{program}
+\end{slide}
+
+\begin{proof}
+  The program attempts to find two square roots of $y$ mod $n$.  It's easy to
+  see that this might lead to factors of $n$.  If $x^2 \equiv x'^2 \pmod{n}$
+  then $x^2 - x'^2 = k n$ for some constant $k$.  Then $(x + x')(x - x')$ is
+  a factorization of $k n$.  It remains to prove the probability bound on $x
+  + x'$ being a nontrivial factor of $n$.
+
+  Let $n$ be an odd composite.  Then, if $x \not\equiv \pm y \pmod{n}$ but
+  $x^2 \equiv y^2 \pmod{n}$, then $\gcd(x + y, n)$ is a nontrivial factor of
+  $n$.
+  
+  Firstly, we claim that, if $p$ is an odd prime then the congruence $x^2
+  \equiv y \pmod{p}$ has precisely two solutions $x$, $x'$ such that $x
+  \equiv -x' \pmod{p}$.  Let $g$ be primitive mod $p$, with $x = g^\alpha$,
+  $x' = g^\beta$.  Then $g^{2 \alpha} \equiv g^{2 \beta} \pmod{p}$, so $2
+  \alpha \equiv 2 \beta \pmod{p - 1}$.  But $p - 1$ is even, by hypothesis,
+  so $\alpha \equiv \beta \pmod{(p - 1)/2}$.  But $g^{(p-1)/2} \equiv -1
+  \pmod{p}$; hence $x \equiv \pm x' \pmod{p}$, proving the claim.
+  
+  There must exist odd primes $p$, $q$, such that $p|n$ and $q|n$, and $x
+  \equiv -y \pmod{p}$ and $x \equiv y \pmod{q}$, for if not, then $x \equiv
+  \pm y \pmod{n}$ contrary to hypothesis.  But then $x + y \equiv 0
+  \pmod{p}$, so $p|(x + y)$; but $x + y \equiv 2 x \not\equiv 0 \pmod{q}$,
+  since $q$ is odd.  Hence, $p$ divides $x + y$, but $q$ does not, so $\gcd(x
+  + y, n)$ is a nontrivial factor of $n$, completing the proof.
+\end{proof}
+
+\begin{slide}
+  \topic{quadratic residuosity}
+  \head{The Quadratic Residuosity Problem}
+
+  If there is an $x$ such that $x^2 \equiv y \pmod{n}$ then $y$ is a
+  \emph{quadratic residue modulo $n$}, and we write $y \in Q_n$; if there is
+  no such $x$ then $y$ is a \emph{quadratic nonresidue modulo $n$}.
+
+  If $p$ is prime, then we can use the \emph{Legendre symbol} to decide
+  whether $x$ is a quadratic residue mod $p$:
+  \[ \jacobi{x}{p} = x^{(p-1)/2} \bmod p =
+     \begin{cases}
+        0 & if $p$ divides $x$ \\
+       -1 & if $x$ is a quadratic nonresidue mod $p$ \\
+       +1 & if $x$ is a quadratic residue mod $p$
+     \end{cases}. \]%
+  The \emph{Jacobi symbol} (written the same way) is defined for odd~$n$: if
+  $n = p_1^{a_1} p_2^{a_2} \cdots p_k^{a_k}$ where the $p_i$ are prime, then
+  \[ \jacobi{x}{n} =
+     \jacobi{x}{p_1}^{a_1} \jacobi{x}{p_2}^{a_2} \cdots
+       \jacobi{x}{p_k}^{a_k}. \]%
+  This can be efficiently computed without knowledge of the factors of $n$
+  \cite[Section~2.4.5]{Menezes:1997:HAC}.
+\end{slide}
+
+\begin{slide}
+  \head{The Quadratic Residuosity Problem (cont.)}
+
+  If $\jacobi{x}{n} = -1$ then $x$ is certainly \emph{not} a quadratic
+  residue mod $n$; however, if $\jacobi{x}{n} = 1$ then $x$ might be a
+  quadratic residue or it might not; if not, then we say that $x$ is a
+  \emph{pseudosquare}.
+  
+  If $n = p q$ is a product of two primes and $x \inr (\Z/n\Z)^*$ is chosen
+  at random, then
+  \[ \Pr\Bigl[x \in Q_n \Bigm| \jacobi{x}{n} = 1\Bigr] = \frac{1}{2}. \]
+
+  The problem of distinguishing pseudosquares from quadratic residues is
+  called the Quadratic Residuosity Problem (QRP).  It is not known how to
+  solve this problem without factoring $n$.
+\end{slide}
+
+\begin{slide}
+  \topic{discrete logarithms}
+  \head{The Discrete Logarithm Problem}
+  
+  The (Integer) Discrete Logarithm Problem asks for the solution $x$ given a
+  congruence of the form $g^x \equiv y \pmod{n}$.  This seems to be about as
+  difficult as factoring.  (The ability to solve discrete logs modulo $n$ is
+  sufficient to factor $n$.  The best known algorithms for IFP and DLP have
+  the same running time.)
+
+  The problem generalizes to other cyclic groups, e.g., elliptic curves over
+  finite fields.
+\end{slide}
+
+\begin{slide}
+  \topic{self-reducibility}
+  \head{Self-reducibility, 1}
+  
+  The problems of square-root extraction, deciding quadratic residuosity, the
+  RSA problem, and finding discrete logarithms share the property of being
+  \emph{randomly self-reducible}; i.e., an instance of the problem can be
+  transformed into many different derived instances \emph{without skewing the
+    probability distribution of problem instances}, such that a solution to
+  one of the derived instances yields a solution to the original one.
+
+  This is a good property to have.  It implies that `most' problem instances
+  are as hard as the hardest instances.
+\end{slide}
+
+\begin{slide}
+  \head{Self-reducibility, 2: the RSA problem \cite{Rivest:1978:MOD}}
+  
+  The RSA problem is to compute $e$-th roots modulo $n = p q$, where $e$ is
+  relatively prime to $n$.  Suppose that the algorithm $S(n, e, y)$ returns a
+  value $x$ such that $x^e \equiv y \pmod{n}$ for `many' choices of $y$, or
+  the special symbol $\bot$ otherwise.  The following probabilistic algorithm
+  then solves the RSA problem for arbitary $y$:
+  \begin{program}
+    Algorithm $\id{solve-rsa}(n, e, y)$: \+ \\
+      \REPEAT \\ \quad\=\+\kill
+        $x' \getsr \{1, 2, \ldots, n - 1\}$; \\
+        \IF $\gcd(x', n) = 1$ \THEN \\ \quad\=\+\kill
+          $y' \gets y x'^e \bmod n$; \\
+          $x \gets S(n, e, y')$; \\
+          \IF $x \ne \bot$ \THEN \RETURN $x x'^{-1} \bmod n$; \-\- \\
+      \FOREVER;
+  \end{program}
+\end{slide}
+
+\begin{slide}
+  \topic{the Diffie-Hellman problem}
+  \head{The Diffie-Hellman problem \cite{Diffie:1976:NDC}}
+  
+  Let $G = \langle g \rangle$ be a cyclic group or order $q$.  Let $\alpha$
+  and $\beta$ be indices, $\alpha, \beta \in \Z/q\Z$.
+  
+  The (computational) \emph{Diffie-Hellman} problem is, given $g^\alpha$ and
+  $g^\beta$, to find $g^{\alpha\beta}$.
+  
+  The (computational) Diffie-Hellman \emph{assumption} is that there is no
+  probabilistic algorithm which solves the computational Diffie-Hellman
+  problem in time polynomial in $q$.
+  
+  Obviously, being able to compute discrete logs in $G$ efficiently would
+  yield a solution to the Diffie-Hellman problem.  But it may be the case
+  that the Diffie-Hellman problem is easier than the discrete log problem.
+
+  The Diffie-Hellman problem is self-reducible.
+\end{slide}
+
+\begin{slide}
+  \head{The Decisional Diffie-Hellman assumption \cite{Boneh:1998:DDP}}
+
+  The computational Diffie-Hellman assumption makes a statement only about
+  algorithms which compute the \emph{entire} answer $g^{\alpha\beta}$.  Since
+  Diffie-Hellman is frequently used for key-exchange, what can we say about
+  the ability of an adversary to guess any of the bits?
+  
+  The Decisional Diffie-Hellman (DDH) assumption asserts that, if you don't
+  know $\alpha$ or $\beta$, then it's hard to tell $g^{\alpha\beta}$ from a
+  random element of $G$; that is, that the distributions of the following
+  experiments are computationally indistinguishable:
+  \begin{program}
+    $\alpha \getsr \Z/q\Z;$ \\
+    $\beta  \getsr \Z/q\Z;$ \\
+    \RETURN $(g^\alpha, g^\beta, g^{\alpha\beta})$;
+  \next
+    $\alpha \getsr \Z/q\Z;$ \\
+    $\beta  \getsr \Z/q\Z;$ \\
+    $\gamma \getsr \Z/q\Z;$ \\
+    \RETURN $(g^\alpha, g^\beta, g^\gamma)$;
+  \end{program}
+\end{slide}
+
+\begin{slide}
+  \head{The Decisional Diffie-Hellman assumption (cont.)}
+  
+  If $A$ is an algorithm attempting to solve DDH in the group $G$, then we
+  write its advantage as
+  \begin{eqnarray*}[rl]
+    \Adv{ddh}{G}(A) =
+    & \Pr[\alpha \getsr \Z/q\Z; \beta \getsr \Z/q\Z :
+          A(g^\alpha, g^\beta, g^{\alpha\beta}) = 1] - {} \\
+    & \Pr[\alpha \getsr \Z/q\Z; \beta \getsr \Z/q\Z; 
+          \gamma \getsr \Z/q\Z :
+          A(g^\alpha, g^\beta, g^\gamma) = 1]
+  \end{eqnarray*}
+  and the insecurity of DDH in $G$ as
+  \[ \InSec{ddh}(G; t) = \max_A \Adv{ddh}{G}(A) \]
+  with the maximum over all algorithms $A$ running in time $t$.
+\end{slide}
+
+\begin{slide}
+  \head{The Decisional Diffie-Hellman assumption (cont.)}
+
+  If you can solve the computational Diffie-Hellman problem, you can solve
+  the decisional version.  If you can compute discrete logs, you can solve
+  both.
+  
+  There \emph{are} groups in which the computation problem is (believed to
+  be) hard, but the decisional problem is easy.  In particular, if the group
+  order $q$ has small factors then the decisional problem isn't hard.
+\end{slide}
+
+\endinput
+
+%%% Local Variables: 
+%%% mode: latex
+%%% TeX-master: "ips"
+%%% End: 
diff --git a/chan.tex b/chan.tex
new file mode 100644 (file)
index 0000000..6e43f21
--- /dev/null
+++ b/chan.tex
@@ -0,0 +1,13 @@
+\xcalways\section{Secure channels}\x
+
+%%%         * Secure channels: unauthenticated and authenticated links
+%%%           models; authenticators and simulation; the message transfer
+%%%           protocol and MT-authenticators; key-exchange, SK-security,
+%%%           perfect forward secrecy and Diffie-Hellman.
+
+\endinput
+
+%%% Local Variables: 
+%%% mode: latex
+%%% TeX-master: "ips"
+%%% End: 
diff --git a/enc-ies.tex b/enc-ies.tex
new file mode 100644 (file)
index 0000000..cd0f5c6
--- /dev/null
@@ -0,0 +1,388 @@
+\xcalways\section{Integrated public-key encryption schemes}\x
+
+The formulation here is original work by the author.  I've tried to
+generalize the work by (among others), Shoup, and Abdalla, Bellare and
+Rogaway.  The final proof is from a Usenet article prompted by David
+Hopwood, but based on the DHAES proof by ABR.
+
+\xcalways\subsection{Introduction and definitions}\x
+
+\begin{slide}
+  \topic{motivation}
+  \head{Motivation for integrated encryption}
+  
+  Public-key encryption schemes are very useful, but there are disadvantages:
+  \begin{itemize}
+  \item they tend not to be very fast; and
+  \item they don't tend to be able to encrypt arbitrarily large messages.
+  \end{itemize}
+  Symmetric schemes don't have these problems, but key distribution is
+  harder.  It seems sensible to construct `hybrid' schemes which use
+  public-key techniques for exchanging keys for symmetric encryption schemes.
+
+  Throughout the following, we'll consider our objective to be resistance to
+  \emph{adaptive chosen-ciphertext} attacks.
+\end{slide}
+
+\begin{slide}
+  \head{An obvious approach}
+  
+  A simple approach would be to generate a random key for some secure (i.e.,
+  IND-CCA) symmetric scheme, encrypt the message under that key, and, encrypt
+  the key under the recipient's public key (using some IND-CCA2 public-key
+  scheme).
+
+  This is obviously secure.  But the security results for most public-key
+  schemes are less than encouraging: the reductions, even for OAEP+, are
+  rather `flabby'.
+
+  Can we do \emph{better} than this na\"{\i}ve approach?
+\end{slide}
+
+\begin{slide}
+  \topic{key encapsulation}
+  \head{Key-encapsulation mechanisms \cite{Shoup:2001:PIS}}
+  
+  A \emph{key-encapsulation mechanism} (KEM) $\mathcal{K} = (G, X, R)$ is a
+  triple of algorithms:
+  \begin{itemize}
+  \item a probabilistic \emph{key-generation} algorithm~$G$, which takes no
+    input (or a security parameter for asymptotic types) and returns a pair
+    $(P, K)$;
+  \item a probabilistic \emph{exchange} algorithm~$E$, which is given a
+    as input a public key~$P$ and returns a \emph{public value}~$y$ and a
+    \emph{hash} $h \in \{0, 1\}^k$; and
+  \item a deterministic \emph{recovery} algorithm~$R$, which is given as
+    input a private key~$K$ and a public value~$y$ and returns a hash $h$.
+  \end{itemize}
+  We require for correctness that, if $(P, K) \in G$ and $(y, h) \in X_P$,
+  then $R_K(y) = h$.
+
+  The idea is that the key-encapsulation mechanism invents a key and public
+  value simultaneously.  We'll look at such mechanisms constructed from both
+  trapdoor one-way functions like RSA, and schemes like Diffie-Hellman.
+\end{slide}
+
+\begin{slide}
+  \topic{oracle hash decision problems}
+  \head{Oracle hash decision problems}
+
+  We say that a key-encapsulation mechanisms is secure if the corresponding
+  \emph{oracle hash decision problem} is hard.  Consider this experiment:
+  \begin{program}
+    Experiment $\Expt{ohd-$b$}{\mathcal{K}}(A)$: \+ \\
+      $(P, K) \gets G$; \\
+      $h_0 \getsr \{0, 1\}^k$; $(y, h_1) \gets X_P$; \\
+      $b' \gets A^{R_K(\cdot)}(P, y, h_b)$; \\
+      \RETURN $b'$;
+  \end{program}
+  We forbid the adversary from querying its $R_K$ oracle at $y$.  We define
+  advantage and insecurity in the usual way:
+  \begin{eqnarray*}[rl]
+    \Adv{ohd}{\mathcal{K}}(A) &=
+      \Pr[\Expt{ohd-$1$}{\mathcal{K}}(A) = 1] -
+      \Pr[\Expt{ohd-$0$}{\mathcal{K}}(A) = 1]
+    \\
+    \InSec{ohd}(\mathcal{K}; t, q) &=
+      \max_A \Adv{ohd}{\mathcal{K}}(A)
+  \end{eqnarray*}
+  where the maximum is taken over adversaries $A$ running in time $t$ and
+  issuing $q$ recovery queries.
+\end{slide}
+
+\xcalways\subsection{Constructions for key-encapsulation mechanisms}\x
+
+\begin{slide}
+  \topic{trapdoor one-way permutations}
+  \head{Constructing a KEM from a trapdoor OWP}
+  
+  Why might key-encapsulation mechanisms exist, and how do we construct them?
+  
+  In the random oracle model, we can construct a KEM from any trapdoor
+  one-way permutation.  Let $\mathcal{T} = (G, f, T)$ be such a one-way
+  permutation, and let $H\colon \{0, 1\}^* \to \{0, 1\}^k$ be a public random
+  oracle.  We can construct $\Xid{\mathcal{K}}{OWF}^{\mathcal{T}, H} = (G,
+  \Xid{X}{OWF}^{\mathcal{T}, H}, \Xid{R}{OWF}^{\mathcal{T}, H})$ as follows:
+  \begin{program}
+    Algorithm $\Xid{X}{OWF}^{\mathcal{T}, H(\cdot)}_P$: \+ \\
+      $x \getsr \dom f_P$; \\
+      $h \gets H(x)$; \\
+      \RETURN $(f_P(x), h)$;
+  \next
+    Algorithm $\Xid{R}{OWF}^{\mathcal{T}, H(\cdot)}_K(y)$: \+ \\
+      $x \gets T_K(y)$; \\
+      $h \gets H(x)$; \\
+      \RETURN $f_P(x)$;
+  \end{program}
+  The security of this scheme is then given by:
+  \[ \InSec{ohd}(\Xid{\mathcal{K}}{OWF}^{\mathcal{T}, H}; t, q_R, q_H) \le
+     2 \cdot \InSec{owf}(\mathcal{T}; t + O(q_H) + O(q_R)). \]%
+\end{slide}
+
+\begin{proof}
+  Let $A$ be an adversary solving the oracle hash decision problem.  Let $y^*
+  = f_P(x^*)$ be the challenge public value, and let $h^*$ be the challenge
+  hash.  Suppose that $A$ never queries its random oracle $H$ at $x^*$: then
+  obviously $h = H(x^*)$ is independent of $A$'s view, and $A$ has no
+  advantage, and its probability of guessing the hidden bit is precisely
+  $\frac{1}{2}$.
+  
+  Suppose that we choose which OHD experiment uniformly at random.  Let $S$
+  be the event that the adversary wins, i.e., it correctly guesses the hidden
+  bit $b$.  Then
+  \[ \Pr[S] =
+       \frac{\Adv{ohd}{\Xid{\mathcal{K}}{OWF}^{\mathcal{T}, H}}(A)}{2} +
+       \frac{1}{2}. \]%
+  Let $F$ be the event that $A$ queries $H$ at $x^*$.  Then by Shoup's Lemma
+  (lemma~\ref{lem:shoup}, page~\pageref{lem:shoup}),
+  \[ \left|\Pr[S] - \frac{1}{2}\right| \le \Pr[F]. \]
+
+  Now consider this adversary $I$, attempting to invert the one-way function.
+  \begin{program}
+    Inverter $I(P, y^*)$: \+ \\
+      $\Xid{H}{map} \gets \emptyset$; \\
+      $h^* \gets \{0, 1\}^k$; $x^* \gets \bot$; \\
+      $b \gets A^{\Xid{R}{sim}(\cdot), \Xid{H}{sim}(\cdot)}(P, y^*, h^*)$; \\
+      \RETURN $x^*$; \- \\[\smallskipamount]
+    Oracle $\Xid{R}{sim}(y)$: \+ \\
+      \IF $y \in \dom\Xid{H}{map}$ \THEN \RETURN $\Xid{H}{map}(y)$; \\
+      $h \gets \{0, 1\}^k$; \\
+      $\Xid{H}{map} \gets \Xid{H}{map} \cup \{ y \mapsto h \}$; \\
+      \RETURN $h$; \- \\[\smallskipamount]
+    Oracle $\Xid{H}{sim}(x)$: \+ \\
+      $y \gets f_P(x)$; \\
+      \IF $y = y^*$ \THEN \\ \quad \= \+ \kill
+        $x^* \gets x$; \\
+        \IF $y \notin \dom\Xid{H}{map}$ \THEN \\ \quad \= \+ \kill
+          $b^* \getsr \{0, 1\}$; \\
+          \IF $b^* = 1$ \THEN
+            $\Xid{H}{map} \gets \Xid{H}{map} \cup \{ y \mapsto h^* \}$ \-\-\\
+      \RETURN $\Xid{R}{sim}(y)$;
+  \end{program}
+  The simulation of the random and `recovery' oracles is a little subtle, but
+  from the adversary's point of view perfect.  Clearly, then, $I$ inverts
+  $f_P$ whenever $F$ occurs, and
+  \[ \Succ{owf}{\mathcal{T}}(I)
+     \ge \Pr[F]
+     \ge \frac{1}{2} \Adv{ohd}{\Xid{\mathcal{K}}{OWF}^{\mathcal{T}, H}}(A). \]%
+  proving the result.
+\end{proof}
+
+\begin{slide}
+  \topic{Diffie-Hellman}
+  \head{A KEM based on Diffie-Hellman \cite{Abdalla:2001:DHIES}}
+
+  Let $G = \langle g \rangle$ be a cyclic group, with $q = |G|$.  Then we can
+  construct a KEM based on the difficulty of the Diffie-Hellman problem in
+  $G$.  Let $H\colon G^2 \to \{0, 1\}^k$ be a public random oracle.
+  
+  We define $\Xid{\mathcal{K}}{DH}^{G, H} = (\Xid{G}{DH}^{G, H},
+  \Xid{X}{DH}^{G, H}, \Xid{R}{DH}^{G, H})$ as follows:
+  \begin{program}
+    Algorithm $\Xid{G}{DH}^{G, H(\cdot, \cdot)}$: \+ \\
+      $\alpha \getsr \Z/q\Z$; \\
+      \RETURN $(g^\alpha, \alpha)$;
+  \next
+    Algorithm $\Xid{X}{DH}^{G, H(\cdot, \cdot)}_a$: \+ \\
+      $\beta \getsr \Z/q\Z$; \\
+      $h \gets H(g^\beta, a^\beta)$; \\
+      \RETURN $(g^\beta, h)$;
+  \next
+    Algorithm $\Xid{R}{DH}^{G, H(\cdot, \cdot)}_\alpha(b)$: \+ \\
+      $h \gets H(b, b^\alpha)$; \\
+      \RETURN $h$;
+  \end{program}
+
+  But the question of this scheme's security is tricky.  It doesn't seem to
+  fit any of our standard problems.  We therefore have to invent a new one!
+\end{slide}
+
+\begin{slide}
+  \head{The Gap Diffie-Hellman problem}
+  
+  The `Gap Diffie-Hellman' problem is essentially the Computational problem,
+  but given an oracle which can answer the Decisional problem.
+
+  Consider this experiment.
+  \begin{program}
+    Experiment $\Expt{gdh}{G}(A)$: \+ \\
+      $\alpha \getsr \Z/q\Z$; $\beta \getsr \Z/q\Z$; \\
+      $c \gets A^{C(\cdot, \cdot)}(g^\alpha, g^\beta)$; \\
+      \IF $c = g^{\alpha\beta}$ \THEN \RETURN $1$; \\
+      \ELSE \RETURN $0$;
+  \next
+    Oracle $C(x, y)$: \+ \\
+      \IF $y = x^\alpha$ \THEN \RETURN $1$; \\
+      \ELSE \RETURN $0$;
+  \end{program}
+  We define the success probability of an adversary $A$ as
+  \[ \Succ{gdh}{G}(A) = \Pr[\Expt{gdh}{G}(A) = 1] \]
+  and the insecurity, against algorithms running in time $t$ and making $q_C$
+  oracle queries, is
+  \[ \InSec{gdh}(G; t, q_C) = \max_A \Succ{gdh}{G}(A). \]
+\end{slide}
+
+\begin{slide}
+  \head{Security of the Diffie-Hellman KEM}
+  
+  Now that we have our new problem, the security result is relatively easy.
+  \[ \InSec{ohd}(\Xid{\mathcal{K}}{DH}^{G, H}; t, q_R, q_H)
+     \le 2\cdot \InSec{gdh}(G; t + O(q_R + q_H), q_H). \]%
+
+  Note that it's very important that the random oracle is given \emph{both}
+  the public value $b$ and the shared secret $b^\alpha$!  Otherwise, the
+  scheme is \emph{malleable} in a group of composite order.  For example,
+  suppose that $q = 2r$ for some $r$; then if $\alpha$ is even, we have
+  $(b\cdot g^r)^\alpha = b^\alpha$.  Passing $b$ to the oracle ensures that
+  these queries given independent answers, even if the shared secret is the
+  same.
+\end{slide}
+
+\begin{proof}
+  Suppose that $A$ solves the oracle hash decision problem for
+  $\Xid{\mathcal{K}}{DH}^{G, H}$.  Let the input to~$A$ be the triple $(a, b,
+  h^*)$, where $a = g^\alpha$ and $b = g^\beta$; and let $h^* =
+  H(g^{\alpha\beta})$.  $A$ must decide whether $h = h^*$.  Clearly, if $A$
+  never queries $H$ at $(g^\beta, g^{\alpha\beta})$ then its advantage is
+  zero, since it has no information about $h^*$.
+
+  As in the one-way function case, we have
+  \[ \Pr[F] \ge \frac{1}{2} \Adv{ohd}{\Xid{\mathcal{K}}{DH}^{G, H}}(A) \]
+  where $F$ is the event that $A$ queries $H$ at $(g^\beta,
+  g^{\alpha\beta})$.
+  
+  We present an algorithm~$B$ which can solve an instance of the Gap
+  Diffie-Hellman problem in~$G$ whenever event $F$ occurs.
+  \begin{program}
+    Algorithm $B^{C(\cdot, \cdot)}(a^*, b^*)$: \+ \\
+      $\Xid{R}{map} \gets \emptyset$; $\Xid{H}{map} \gets \emptyset$; \\
+      $h^* \gets \{0, 1\}^k$; \\
+      $c^* \gets \bot$; \\
+      $b \gets A^{\Xid{R}{sim}(\cdot), \Xid{H}{sim}(\cdot)}
+        (a^*, b^*, h^*)$; \\
+      \RETURN $c^*$; \- \\[\smallskipamount]
+    Oracle $\Xid{R}{sim}(b)$: \+ \\
+      \IF $b \in \dom\Xid{R}{map}$ \\
+      \THEN \RETURN $\Xid{R}{map}(b)$; \\
+      $h \gets \{0, 1\}^k$; \\
+      $\Xid{R}{map} \gets \Xid{R}{map} \cup \{ b \mapsto h \}$; \\
+      \RETURN $h$;
+  \next
+    Oracle $\Xid{H}{sim}(b, c)$: \+ \\
+      \IF $C(b, c) = 0$ \THEN \\ \quad \= \+ \kill
+        \IF $(b, c) \in \dom\Xid{H}{map}$ \THEN
+          \RETURN $\Xid{H}{map}(b, c)$; \\
+        $h \gets \{0, 1\}^k$; \\
+        $\Xid{H}{map} \gets \Xid{H}{map} \cup \{ (b, c) \mapsto h \}$; \\
+        \RETURN $h$; \- \\
+      \IF $b = b^*$ \THEN \\ \quad \= \+ \kill
+        $c^* \gets c$; \\
+        \IF $c \notin \dom\Xid{R}{map}$ \THEN \\ \quad \= \+ \kill
+          $b^* \getsr \{0, 1\}$; \\
+          \IF $b^* = 1$ \THEN
+            $\Xid{R}{map} \gets \Xid{R}{map} \cup \{ y \mapsto h^* \}$ \-\-\\
+      \RETURN $\Xid{R}{sim}(c)$; \- \\
+  \end{program}
+  Hence, we have
+  \[ \Succ{gdh}{G}(A)
+     \ge \frac{1}{2} \Adv{ohd}{\Xid{\mathcal{K}}{DH}^{G, H}}(A) \]%
+  as required.
+\end{proof}
+
+\xcalways\subsection{An integrated encryption scheme}\x
+
+\begin{slide}
+  \head{An integrated encryption scheme}
+
+  Let $\mathcal{K} = (G, X, R)$ be a key-encapsulation mechanism, and let
+  $\mathcal{E} = (E, D)$ be a symmetric encryption scheme.  We define the
+  \emph{integrated encryption scheme} $\Xid{\mathcal{E}}{IES}^{\mathcal{K},
+  \mathcal{E}} = (\Xid{G}{IES}^{\mathcal{K}, \mathcal{E}},
+  \Xid{E}{IES}^{\mathcal{K}, \mathcal{E}}, \Xid{D}{IES}^{\mathcal{K},
+  \mathcal{E}})$ as follows:
+  \begin{program}
+    Algorithm $\Xid{G}{IES}^{\mathcal{K}, \mathcal{E}}$: \+ \\
+      $(P, K) \gets G$; \\
+      \RETURN (P, K);
+  \next
+    Algorithm $\Xid{E}{IES}^{\mathcal{K}, \mathcal{E}}_P(x)$: \+ \\
+      $(y_0, h) \gets X_P$; \\
+      $y_1 \gets E_h(x)$; \\
+      \RETURN $(y_0, y_1)$;
+  \next
+    Algorithm $\Xid{D}{IES}^{\mathcal{K}, \mathcal{E}}_K(y_0, y_1)$: \+ \\
+      $h \gets R_K(y_0)$; \\
+      $x \gets D_h(y_1)$; \\
+      \RETURN $x$;
+  \end{program}
+  The security of this scheme relates tightly to that of the individual
+  components:
+  \begin{eqnarray*}[Ll]
+     \InSec{ind-cca2}(\Xid{G}{IES}^{\mathcal{K}, \mathcal{E}}; t, q_D) \\
+     & \le
+       2 \cdot \InSec{ohd}(\mathcal{K}; t + O(q_D), q_D) +
+       \InSec{ftg-cca}(\mathcal{E}; t + O(q_D), 0, q_D).
+  \end{eqnarray*}
+  Note how weak the security requirements on the encryption scheme are: no
+  chosen-plaintext queries are permitted!
+\end{slide}
+
+\begin{proof}
+  Suppose $A$ attacks $\Xid{G}{IES}^{\mathcal{K}, \mathcal{E}}$ in the
+  IND-CCA2 sense.  We construct an adversary $B$ which attacks the KEM.
+  \begin{program}
+    Adversary $B^{R(\cdot)}(P, y, h)$: \+ \\
+      $(x_0, x_1, s) \gets A^{\Xid{D}{sim}(\cdot)}(\cookie{find}, P)$; \\
+      $b \gets \{0, 1\}$; \\
+      $z \gets E_h(x_b)$; \\
+      $b' \gets A^{\Xid{D}{sim}(\cdot)}(\cookie{guess}, (y, z), s)$; \\
+      \IF $b = b'$ \THEN \RETURN $1$; \\
+      \ELSE \RETURN $0$;
+  \next
+    Oracle $\Xid{D}{sim}(y_0, y_1)$: \+ \\
+      \IF $y_0 = y$ \THEN \RETURN $D_h(y_1)$; \\
+      $h' \gets R(y_0)$; \\
+      \RETURN $D_{h'}(y_1)$;
+  \end{program}
+  If the $h$-value matches the public value $y$ then $B$ provides a correct
+  simulation of $A$'s attack game, and hence wins with probability
+  \[ \frac{\Adv{ind-cca2}{\Xid{G}{IES}^{\mathcal{K}, \mathcal{E}}}}{2} +
+     \frac{1}{2}. \]%
+  We construct a new adversary $C$, attacking $\mathcal{E}$ in the FTG-CCA
+  sense, to help us bound $B$'s probability of success when $h$ is chosen
+  randomly.
+  \begin{program}
+    Adversary $C^{E(\cdot), D(\cdot)}(\cookie{find})$: \+ \\
+      $(P, K) \gets G$; \\
+      $y' \gets \bot$; \\
+      $(x_0, x_1, s_A) \gets A^{\Xid{D}{sim}(\cdot)}(\cookie{find}, P)$; \\
+      \RETURN $(x_0, x_1, (P, K, s_A))$;
+  \next
+    Adversary $C^{E(\cdot), D(\cdot)}(\cookie{guess}, y, s)$: \+ \\
+      $(P, K, s_A) \gets s$; \\
+      $(h', y') \gets X_P$; \\
+      $b <- A^{\Xid{D}{sim}(\cdot)}(\cookie{guess}, (y', y), s_A)$; \\
+      \RETURN $b$;
+  \newline
+    Oracle $\Xid{D}{sim}(y_0, y_1)$: \+ \\
+      \IF $y_0 = y'$ \THEN \RETURN $D(y_1)$; \\
+      $h \gets R_K(y_0)$; \\
+      \RETURN $D_h(y_1)$;
+  \end{program}
+  Clearly, $C$ provides the same environment as $B$ does when $h$ is random;
+  hence, in this case, $B$ wins with probability at most
+  \[ \frac{\Adv{ftg-cca2}{\mathcal{E}}(C)}{2} + \frac{1}{2}. \]
+  But
+  \[ \Adv{ohd}{\mathcal{K}}(B) =
+       \frac{1}{2}\left(
+           \Adv{ind-cca2}{\Xid{G}{IES}^{\mathcal{K}, \mathcal{E}}} +
+           \Adv{ftg-cca2}{\mathcal{E}}(C) \right). \]%
+  Rearranging yields the required result.
+\end{proof}
+
+\endinput
+
+%%% Local Variables: 
+%%% mode: latex
+%%% TeX-master: "ips"
+%%% End: 
diff --git a/enc-intro.tex b/enc-intro.tex
new file mode 100644 (file)
index 0000000..6199d4f
--- /dev/null
@@ -0,0 +1,103 @@
+\xcalways\section{Introduction to Encryption}\x
+
+\xcalways\subsection{Security notions and attacks}\x
+
+%%%         * Security notions and attacks: semantic security and find-then-
+%%%           guess indistinguishability; left-or-right and real-or-random
+%%%           indistinguishability; chosen plaintext and chosen ciphertext
+%%%           (lunchtime and adaptive) attacks; non-malleability; plaintext
+%%%           awareness; funny abbreviations (e.g., IND-CPA, NM-CCA2).
+
+\begin{slide}
+  \head{Security notions for encryption}
+
+  What does it mean to say that an encryption scheme is secure?
+\end{slide}
+
+\begin{slide}
+  \topic{adversarial goals}
+  \head{Encryption: adversarial goals 1}
+
+  \begin{description}
+  \item [Indistinguishability (find-then-guess)] The adversary chooses two
+    plaintexts.  One is selected at random, and the ciphertext is returned.
+    The adversary cannot guess which plaintext was chosen with probability
+    significantly better than $\frac{1}{2}$.
+  \item [Semantic security] An adversary given a ciphertext cannot compute
+    anything about the plaintext that it couldn't compute given only its
+    length.
+  \end{description}
+\end{slide}
+
+\begin{slide}
+  \head{Encryption: adversarial goals 2}
+
+  \begin{description}
+  \item [Indistinguishability (left-or-right)] The adversary is given an
+    oracle which accepts two plaintexts.  Before the game begins, a decision
+    is taken as to whether the oracle returns the result of encrypting the
+    `left' plaintext, or the `right' one.  The adversary cannot guess which
+    with probability significantly better than $\frac{1}{2}$.
+  \item [Indistinguishability (real-or-random)] The adversary is given an
+    oracle.  Before the game begins, a decision is taken as to whether the
+    oracle correctly encrypts the plaintexts it is given (`real') or whether
+    it returns a ciphertext for a randomly chosen plaintext of the same
+    length (`random').  The adversary cannot guess which with probability
+    significantly better than $\frac{1}{2}$.
+  \end{description}
+\end{slide}
+
+\begin{slide}
+  \head{Encryption: adversarial goals 3}
+
+  \begin{description}
+  \item [Non-malleability] An adversary cannot transform a ciphertext such
+    that the plaintexts of the two ciphertexts are related, with better than
+    negligible probability.
+  \item [Plaintext awareness] An adversary cannot create a ciphertext without
+    `knowing' (or easily being able to find out) the corresponding plaintext
+    (or knowing that the ciphertext is invalid), except with negligible
+    probability.
+  \end{description}
+\end{slide}
+
+\begin{slide}
+  \topic{types of attacks}
+  \head{Encryption: types of attacks}
+
+  \begin{description}
+  \item [Chosen plaintext] The adversary may encrypt plaintexts of its
+    choice.  In the asymmetric setting, it is given a public key; in the
+    symmetric setting, it is provided with an encryption oracle.
+  \item [Chosen ciphertext (lunchtime)] (Find-then-guess, semantic security
+    and non-malleability) As with chosen plaintext, but the adversary is
+    given an oracle which can decrypt ciphertexts during its first stage.
+  \item [Adaptive chosen ciphertexts] As with standard chosen ciphertexts,
+    except that the adversary is given the decryption oracle for its entire
+    run.  The adversary is forbidden from using the oracle to decrypt
+    ciphertexts which it is required to distinguish.
+  \end{description}
+\end{slide}
+
+\begin{slide}
+  \topic{funny abbreviations}
+  \head{Funny abbreviations}
+
+  The attack goals are given abbreviations: IND, NM, PA for
+  indistinguishability, non-malleability and plaintext awareness.
+
+  The attack types are given abbreviations too: CPA, CCA1, CCA2 for chosen
+  plaintext, chosen ciphertext and adaptive chosen ciphertext.
+
+  Hence, IND-CPA means `indistinguishable under chosen plaintext attack',
+  NM-CCA2 means `non-malleable under chosen ciphertext attack'.
+  
+  PA stands on its own (but there are two different meanings).
+\end{slide}
+
+\endinput
+
+%%% Local Variables: 
+%%% mode: latex
+%%% TeX-master: "ips"
+%%% End: 
diff --git a/enc-pub.tex b/enc-pub.tex
new file mode 100644 (file)
index 0000000..985690c
--- /dev/null
@@ -0,0 +1,1576 @@
+\xcalways\section{Public-key encryption}\x
+
+\xcalways\subsection{Syntax}\x
+
+\begin{slide}
+  \head{Syntax of public-key encryption schemes}
+
+  A public-key encryption scheme $\mathcal{E} = (G, E, D)$ is a triple of
+  algorithms:
+  \begin{itemize}
+  \item A probabilistic \emph{key-generation} algorithm $G(k)$ which returns
+    a matching public/private key pair $(P, K)$.
+  \item A probabilistic \emph{encryption} algorithm $E(P, m)$ which returns a
+    ciphertext $c$.  We write $E_P(m)$ for $E(P, m)$.
+  \item A deterministic \emph{decryption} algorithm $D(K, c)$.  If $(P, K)
+    \in G(k)$ and $c \in E(P, m)$ then $m = D(K, c)$.  We write $D_K(c)$ for
+    $D(K, c)$.
+  \end{itemize}
+  On those occasions it matters, we write $\mathcal{P} = \dom E_P$ as the
+  \emph{plaintext space}, and $\mathcal{C} = \dom D_K$ as the
+  \emph{ciphertext space}.  Hence, $E_P\colon \mathcal{P} \to \mathcal{C}$
+  and $D_K\colon \mathcal{C} \to \mathcal{P} \cup \{ \bot \}$ (allowing an
+  `invalid ciphertext' return).
+\end{slide}
+
+\xcalways\subsection{Semantic security and indistinguishability}\x
+
+\begin{slide}
+  \head{Notation for oracles}
+
+  We'll use the following decryption oracles throughout, to abbreviate the
+  properties of the various attacks:
+  \begin{tabular}[C]{l Mc Mc }
+                                \hlx*{hv}
+    Attack & D_0(c) & D_1(c) \\ \hlx{vhv}
+    CPA    & \bot   & \bot   \\
+    CCA1   & D_K(c) & \bot   \\
+    CCA2   & D_K(c) & D_K(c) \\ \hlx*{vh}
+  \end{tabular}
+  In all cases, we forbid the adversary from querying a decryption oracle on
+  a challenge ciphertext, i.e., one returned to it by the experiment.
+\end{slide}
+
+\begin{slide}
+  \topic{semantic security}
+  \head{Semantic security}
+  
+  Semantic security for $\mathcal{E} = (G, E, D)$, against an adversary
+  $A$ and attack $\id{atk} \in \{\text{cpa}, \text{cca1}, \text{cca2}\}$
+  is measured using the following game \cite{Bellare:2000:CST}:
+  \begin{program}
+    Experiment $\Expt{sem-\id{atk}-$b$}{\mathcal{E}}(A)$: \+ \\
+      $(P, K) \gets G$; \\
+      $(\mathcal{M}, s) \gets A^{D_0(\cdot)}(\cookie{select}, P)$; \\
+      $x_0 \getsr \mathcal{M}$; $x_1 \getsr \mathcal{M}$; \\
+      $y \gets E_P(x_1)$; \\
+      $(f, \alpha) \gets A^{D_1(\cdot)}(\cookie{predict}, y, s)$; \\
+      \IF $f(x_b) = \alpha$ \THEN \RETURN $1$; \\
+      \ELSE \RETURN $0$;
+  \end{program}
+  Here, $\mathcal{M}\colon \mathcal{P} \to [0, 1]$ is a \emph{distribution}
+  over the plaintext space, and $f\colon \mathcal{P} \to \ran f$ is a
+  \emph{function} on plaintexts, with $\alpha \in \ran f$.
+\end{slide}
+
+\begin{slide}
+  \head{Semantic security (cont.)}
+
+  We include the time required to sample the distribution $\mathcal{M}$ and
+  to compute the function $f$ in the adversary's running time.
+
+  We require that $\mathcal{M}$ is \emph{valid}: i.e., that all messages in
+  $\supp \mathcal{M}$ have the same length.
+\end{slide}
+
+\begin{slide}
+  \topic{indistinguishability}
+  \head{Indistinguishability}
+  
+  Indistinguishability for $\mathcal{E} = (G, E, D)$, against an adversary
+  $A$ and attack $\id{atk} \in \{\text{cpa}, \text{cca1}, \text{cca2}\}$
+  is measured using the following game:
+  \begin{program}
+    Experiment $\Expt{ind-\id{atk}-$b$}{\mathcal{E}}(A)$: \+ \\
+      $(P, K) \gets G$; \\
+      $(x_0, x_1, s) \gets A^{D_0(\cdot)}(\cookie{find}, P)$; \\
+      \IF $|x_0| \ne |x_1|$ \THEN \RETURN $0$; \\
+      $y \gets E_P(x_b)$; \\
+      $b' \gets A^{D_1(\cdot)}(\cookie{guess}, y, s)$; \\
+      \RETURN $b'$;
+  \end{program}
+  In the first stage, the adversary has to choose two plaintexts.  One is
+  then chosen by the experimenter, encrypted, and the corresponding
+  ciphertext given to the adversary.  The adversary must decide which
+  plaintext was encrypted.
+\end{slide}
+
+\begin{slide}
+  \topic{advantage and insecurity}
+  \head{Advantage and insecurity}
+  
+  For a public-key encryption scheme $\mathcal{E}$, under attack $\id{atk}
+  \in \{\text{cpa}, \text{cca1}, \text{cca2}\}$ by an adversary $A$, we
+  define $A$'s advantage by:
+  \begin{eqnarray*}[rl]
+    \Adv{ind-\id{atk}}{\mathcal{E}}(A) &=
+      \Pr[\Expt{ind-\id{atk}-$1$}{\mathcal{E}}(A) = 1] -
+      \Pr[\Expt{ind-\id{atk}-$0$}{\mathcal{E}}(A) = 1];
+  \\
+    \Adv{sem-\id{atk}}{\mathcal{E}}(A) &=
+      \Pr[\Expt{sem-\id{atk}-$1$}{\mathcal{E}}(A) = 1] -
+      \Pr[\Expt{sem-\id{atk}-$0$}{\mathcal{E}}(A) = 1].    
+  \end{eqnarray*}
+  We define insecurities for $\id{goal} \in \{\text{ind}, \text{sem}\}$ under
+  chosen plaintext attacks, and chosen ciphertext attacks $\id{cca} \in
+  \{\text{cca1}, \text{cca2}\}$ by:
+  \begin{eqnarray*}
+    \InSec{\id{goal}-cpa}(\mathcal{E}; t) &=
+      \max_A \Adv{\id{goal}-cpa}{\mathcal{E}}(A);
+  \\
+     \InSec{\id{goal}-\id{cca}}(\mathcal{E}; t, q_D) &=
+       \max_A \Adv{\id{goal}-\id{cca}}{\mathcal{E}}(A).
+  \end{eqnarray*}
+  where the maxima are taken over adversaries $A$ which run in time $t$ and
+  issue $q_E$ encryption and $q_D$ decryption queries.
+\end{slide}
+
+\begin{slide}
+  \topic{good news}
+  \head{Some good news}
+
+  Now there's some good news: \emph{semantic security and (find-then-guess)
+  indistinguishability are almost equivalent}.
+
+  More precisely, if we fix a collection of resource constraints $R$, we have
+  \[ \InSec{sem-\id{atk}}(\mathcal{E}; R) \le
+     \InSec{ind-\id{atk}}(\mathcal{E}; R) \le
+     2 \cdot \InSec{sem-\id{atk}}(\mathcal{E}; R). \]%
+  It's useful to realise that a relatively strong notion like semantic
+  security is actually equivalent to a simpler notion like
+  indistinguishability.  The latter is certainly easier to work with in
+  proofs of security.
+\end{slide}
+
+\begin{proof}
+  \label{pf:pub-ind-eq-sem}
+  Proving that $\text{IND-\id{atk}} \implies \text{SEM-\id{atk}}$ is
+  pretty easy, so we do that first.  Suppose that $A'$ attacks $\mathcal{E}$
+  in the semantic security sense.  Consider the find-then-guess adversary
+  $A$:
+  \begin{program}
+    Adversary $A^{D(\cdot)}(\cookie{find}, P)$: \+ \\
+      $(\mathcal{M}, s') \gets A'^{D(\cdot)}(\cookie{select}, P)$; \\
+      $x_0 \getsr \mathcal{M}$; $x_1 \getsr \mathcal{M}$; \\
+      \RETURN $(x_0, x_1, (x_1, s'))$;
+  \next
+    Adversary $A^{D(\cdot)}(\cookie{guess}, y, s)$: \+ \\
+      $(x_1, s') \gets s$; \\
+      $(f, \alpha) \gets A'^{D(\cdot)}(\cookie{predict}, y, s')$; \\
+      \IF $f(x_1) = \alpha$ \THEN \RETURN $1$; \\
+      \ELSE \RETURN $0$;
+  \end{program}
+  Here, $A$ is simulating the semantic security experiment itself, drawing
+  plaintexts from $A'$'s distribution $\mathcal{M}$ and evaluating its chosen
+  function.
+  
+  We study the result of the experiment
+  $\Expt{ind-\id{atk}-$b$}{\mathcal{E}}(A)$.  Firstly, suppose $b = 1$.
+  Then $y \in E_P(x_1)$, and $A$ succeeds with the same probability that $A'$
+  wins in $\Expt{sem-\id{atk}-$1$}{\mathcal{E}}(A')$.  Secondly, suppose $b
+  = 0$.  Then $y \in E_P(x_0)$, but we still compare $A'$'s answer against
+  $x_1$, as in $\Expt{sem-\id{atk}-$0$}{\mathcal{E}}(A')$, up to
+  relabelling of $x_0$ and $x_1$.  Hence,
+  \[ \Adv{ind-\id{atk}}{\mathcal{E}}(A) =
+       \Adv{sem-\id{atk}}{\mathcal{E}}(A'). \]%
+  
+  Now we show that $\text{SEM-\id{atk}} \implies \text{IND-\id{atk}}$.
+  Suppose that $A$ attacks $\mathcal{E}$ in the indistinguishability sense.
+  Then consider the adversary $A'$ attacking its semantic security:
+  \begin{program}
+    Adversary $A'^{D(\cdot)}(\cookie{select}, P)$: \+ \\
+      $(x'_0, x'_1, s) \gets A^{D(\cdot)}(\cookie{find}, P)$; \\
+      $\mathcal{M} \gets
+        \{ x'_0 \mapsto \frac{1}{2}, x'_1 \mapsto \frac{1}{2} \}$; \\
+      \RETURN $(x'_0, x'_1, s')$;
+  \next
+    Adversary $A'^{D(\cdot)}(\cookie{predict}, y, s')$: \+ \\
+      $(x'_0, x'_1, s) \gets s'$; \\
+      $b \gets A^{D(\cdot)}(\cookie{guess}, y, s)$; \\
+      \RETURN $(\lambda x.x, x'_b)$;
+  \end{program}
+  Here $A'$ is simply running the indistinguishability experiment.  In the
+  $\cookie{select}$ stage, it runs $A$'s $\cookie{find}$ stage, and returns
+  the uniform distribution over $A$'s two chosen plaintexts.  In the
+  $\cookie{predict}$ stage, $A'$ collects $A$'s $\cookie{guess}$ and returns
+  the identity function $\lambda x.x$ and the guessed plaintext.
+
+  In the case of experiment $\Expt{sem-\id{atk}-$1$}{\mathcal{E}}(A')$, the
+  rules are fair, and we win with probability
+  \[  p = \frac{1}{2} + \frac{\Adv{ind-\id{atk}}{\mathcal{E}}(A)}{2}. \]
+  In the case of $\Expt{sem-\id{atk}-$0$}{\mathcal{E}}(A')$, however, we
+  \emph{lose} in the event that $x_0 = x'_b$.  This happens if \emph{either}
+  $x_0 = x_1$ and we guess correctly (probability $p/2$), \emph{or} if $x_0
+  \ne x_1$ and we guess incorrectly (probability $(1 - p)/2$).  Hence, we see
+  that have
+  \[ \Adv{sem-\id{atk}}{\mathcal{E}}(A') =
+       \frac{1}{2} \Adv{ind-\id{atk}}{\mathcal{E}}(A) \]%
+  completing the proof.
+\end{proof}
+
+\xcalways\subsection{Non-malleability}\x
+
+\begin{slide}
+  \topic{definition}
+  \head{Non-malleability}
+  
+  The intuition behind non-malleability is that an adversary can't easily
+  take some ciphertext and turn it into some other ciphertext such that the
+  plaintexts have a simple relationship to each other.
+
+  Here's a relatively standard definition of non-malleability, from
+  \cite{Bellare:1998:RAN, Bellare:1999:NEE}:
+  \begin{program}
+    Experiment $\Expt{cnm-\id{atk}-$b$}{\mathcal{E}}(A)$: \+ \\
+      $(P, K) \gets G$; \\
+      $(\mathcal{M}, s) \gets A^{D_0(\cdot)}(\cookie{find}, P)$;
+      $x_0 \getsr \mathcal{M}$; $x_1 \getsr \mathcal{M}$;
+      $y \gets E_P(x_1)$; \\
+      $(R, \vect{y}) \gets A^{D_1(\cdot)}(\cookie{guess}, y, s)$;
+      $\vect{x} \gets D_K(\vect{y})$; \\
+      \IF $y \notin \vect{y} \land
+           R(x_b, \vect{x})$ \THEN \RETURN $1$;
+      \ELSE \RETURN $0$;
+  \end{program}
+  In the above, $\mathcal{M}$ is a valid distribution on plaintexts, and $R$
+  is an $n$-ary relation on plaintexts.  The adversary's advantage is
+  \[ \Adv{cnm-\id{atk}}{\mathcal{E}}(A) =
+       \Pr[\Expt{cnm-\id{atk}-$1$}{\mathcal{E}}(A) = 1] -
+       \Pr[\Expt{cnm-\id{atk}-$0$}{\mathcal{E}}(A) = 1]. \]%
+\end{slide}
+
+\begin{slide}
+  \topic{more good news}
+  \head{Non-malleability (more good news)}
+
+  The previous definition involved all sorts of nasties like distributions
+  and relations.  Fortunately, there's an approximately equivalent notion,
+  based on indistinguishability with a single \emph{parallel}
+  chosen-ciphertext query:
+  \begin{program}
+    Experiment $\Expt{nm-\id{atk}-$b$}{\mathcal{E}}(A)$: \+ \\
+      $(P, K) \gets G$; \\
+      $(x_0, x_1, s) \gets A^{D_0(\cdot)}(\cookie{find}, P)$;
+      $y \gets E_P(x_b)$; \\
+      $(\vect{y}, t) \gets A^{D_1(\cdot)}(\cookie{choose}, y, s)$;
+      \IF $y \in \vect{y}$ \THEN \RETURN $0$; \\
+      $\vect{x} \gets D_K(\vect{y})$;
+      $b \gets A^{D_1(\cdot)}(\cookie{guess}, \vect{x}, t)$;
+      \RETURN $b$;
+  \end{program}
+  We define advantage by
+  \[ \Adv{nm-\id{atk}}{\mathcal{E}}(A) =
+       \Pr[\Expt{nm-\id{atk}-$1$}{\mathcal{E}}(A) = 1] -
+       \Pr[\Expt{nm-\id{atk}-$0$}{\mathcal{E}}(A) = 1]. \]%
+\end{slide}
+
+\begin{proof}
+  Firstly, $\text{NM} \implies \text{CNM}$.  Suppose $A'$ attacks
+  $\mathcal{E}$ in the CNM sense.  Then we construct $A$ in the obvious way:
+  \begin{program}
+    Adversary $A^{D(\cdot)}(\cookie{find}, P)$: \+ \\
+      $(\mathcal{M}, s') \gets A'^{D(\cdot)}(\cookie{find}, P)$; \\
+      $x_0 \getsr \mathcal{M}$; $x_1 \getsr \mathcal{M}$; \\
+      \RETURN $(x_0, x_1, (x_1, s'))$;
+  \next
+    Adversary $A^{D(\cdot)}(\cookie{choose}, y, s)$: \+ \\
+      $(x_1, s') \gets s$; \\
+      $(R, \vect{y}) \gets A'^{D(\cdot)}(\cookie{guess}, y, s')$; \\
+      \RETURN $(\vect{y}, (x_1, R))$;
+  \next
+    Adversary $A^{D(\cdot)}(\cookie{guess}, \vect{x}, s)$: \+ \\
+      $(x_1, R) \gets s$; \\
+      \IF $R(x_1, \vect{x})$ \THEN \RETURN $1$; \\
+      \ELSE \RETURN $0$;
+  \end{program}
+  Studying the behaviour of $A$, we see that it succeeds precisely when $A'$
+  succeeds.  Hence
+  \[ \Adv{nm-\id{atk}}{\mathcal{E}}(A) =
+     \Adv{cnm-\id{atk}}{\mathcal{E}}(A'). \]%
+  
+  Secondly, suppose that $A$ attacks $\mathcal{E}$ in the NM sense.  Then we
+  construct $A'$ in a somewhat tricky manner: $A'$ aims to run the final
+  stage of $A$ from its relation $R$.
+  
+  We can suppose, without loss of generality, that $A$ doesn't require its
+  chosen ciphertext oracle $D$ during its final $\cookie{guess}$ phase.  If
+  it is allowed an adaptive chosen ciphertext oracle, it can make its
+  parallel query through $D$ (admittedly at the cost of $|\vect{y}|$
+  additional decryption queries).  Hence, we don't need to provide
+  $A(\cookie{guess})$ with a decryption oracle.
+  
+  The relation isn't allowed to be probabilistic.  Hence, we choose the coins
+  $\rho \in \{0, 1\}^n$ that $A(\cookie{guess})$ requires in advance.  Since
+  $A$'s running time is bounded, $n$ must also be bounded.  We encode the
+  values $(x'_0, x'_1, t, \rho)$ into the description of $R$ output by
+  $A'(\cookie{guess})$.
+
+  \begin{program}
+    Adversary $A'^{D(\cdot)}(\cookie{find}, P)$: \+ \\
+      $(x'_0, x'_1, s) \gets A^{D(\cdot)}(\cookie{find}, P)$; \\
+      $\mathcal{M} \gets
+        \{ x'_0 \mapsto \frac{1}{2}, x'_1 \mapsto \frac{1}{2} \}$; \\
+      \RETURN $(\mathcal{M}, (x'_0, x'_1, P, s))$;
+  \next
+    Adversary $A'^{D(\cdot)}(\cookie{guess}, y, s')$: \+ \\
+      $(x'_0, x'_1, P, s) \gets s'$; \\
+      $(\vect{y}, t) \gets A^{D(\cdot)}(\cookie{choose}, y, s)$; \\
+      $\rho \getsr \{0, 1\}^n$; \\
+      \RETURN $(R, \vect{y})$; \- \\[\smallskipamount]
+    Relation $R(x, \vect{x})$: \+ \\
+      $b' \gets A(\cookie{guess}, \vect{x}, t)$ (with coins $\rho$); \\
+      \IF $x = x'_{b'}$ \THEN \RETURN $1$; \\
+      \ELSE \RETURN $0$;
+  \end{program}
+  
+  The analysis of $A'$'s success probability is very similar to the proof
+  that $\text{SEM} \implies \text{IND}$ above.  When the CNM experiment's
+  hidden bit $b = 1$, $A'$ succeeds whenever $A$ correctly guesses which
+  plaintext was encrypted, which occurs with probability
+  \[  p = \frac{1}{2} + \frac{\Adv{nm-\id{atk}}{\mathcal{E}}(A)}{2}. \]
+  When $b = 0$, $A'$ fails when $A$ guesses correctly and $x_0 = x_1$
+  (probability $p/2$), or when $A$ guesses wrongly and $x_0 \ne x_1$
+  (probability $(1 - p)/2$).  Hence, finally,
+  \[ \Adv{cnm-\id{atk}}{\mathcal{E}}(A) =
+       \frac{\Adv{nm-\id{atk}}{\mathcal{E}}(A)}{2}. \]%
+  
+  For any arbitrary resource bounds $R$, we have
+  \[ \InSec{cnm-\id{atk}}(\mathcal{E}; R) \le
+     \InSec{nm-\id{atk}}(\mathcal{E}; R) \le
+     2 \cdot \InSec{cnm-\id{atk}}(\mathcal{E}; R'), \]%
+  where $R'$ is related to $R$, but may need to allow additional decryption
+  queries, to cope with NM-CCA2 adversaries which use decryption queries in
+  their final $\cookie{guess}$ stages.
+  
+  We conclude that the two notions are almost equivalent, as claimed.
+\end{proof}
+
+\xcalways\subsection{Relations between security notions}\x
+
+\begin{slide}
+  \head{Relations between security notions \cite{Bellare:1998:RAN}}
+
+  %%               3                   3
+  %%   NM-CPA <--------- NM-CCA1 <--------- NM-CCA2
+  %%      |  \__            |   \__           ^ ^
+  %%      |     \__  6      |      \__ 7      | |
+  %%     1|        \/_      |1        \/_    4| |1
+  %%      |       5/  \__   |         /  \_   | |
+  %%      v            __\  v             __\ v |
+  %%   IND-CPA <-------- IND-CCA1 <------- IND-CCA2
+  %%                2                 2
+
+  \[ \xymatrix @=2cm {
+    \txt{NM-CPA}   \ar[d]_1
+                   \POS !<1ex, 0pt> \ar[dr]!<1ex, 0pt>|-@{/}^6 &
+    \txt{NM-CCA1}  \ar[d]^1 \ar[l]_3 \ar[dr]|-@{/}^7 &
+    \txt{NM-CCA2}  \ar[l]_3 \ar@<0.5ex>[d]^1 \\
+    \txt{IND-CPA}  &
+    \txt{IND-CCA1} \ar[l]^2
+                   \POS !<-1ex, 0pt> \ar[ul]!<-1ex, 0pt>|-@{/}^5 &
+    \txt{IND-CCA2} \ar[l]^2 \ar@<0.5ex>[u]^4 \\
+  } \]
+
+  \begin{list}{}{
+      \settowidth{\labelwidth}{\textbf{Key}}
+      \leftmargin\labelwidth\advance\leftmargin\labelsep
+      \itemindent0pt\let\makelabel\textbf}
+  \item[Key] \begin{itemize}
+    \item An arrow $\xy\ar*+{A};<1.5cm, 0cm>*+{B}\endxy$ indicates an
+      \emph{implication}: if a scheme is secure in notion $A$ then it is also
+      secure in notion $B$.
+    \item A crossed arrow $\xy\ar|-@{/}*+{A};<1.5cm, 0cm>*+{B}\endxy$
+      indicates a \emph{separation}: there exists a scheme secure in notion
+      $A$ but not in $B$.
+    \item The numbers refer to sections of the proof provided in the notes.
+    \end{itemize}
+  \end{list}
+\end{slide}
+
+\begin{proof}
+  Most of these are fairly simple.  We use the indistinguishability-based
+  characterization of non-malleability that we showed earlier, because it
+  makes life much easier.  We assume that schemes meeting each of the
+  definitions exist, else the theorems are all vacuously true.
+
+  \begin{enumerate}
+    
+  \item We show $\text{NM-\id{atk}} \implies \text{IND-\id{atk}}$ for
+    $\id{atk} \in \{\text{CPA}, \text{CCA1}, \text{CCA2}\}$.  Let $[]$ denote
+    the empty vector.  Suppose that $A$ attacks $\mathcal{E}$ in the
+    $\text{IND-\id{atk}}$ sense.
+    \begin{program}
+      Adversary $A'^{D(\cdot)}(\cookie{find}, P)$: \+ \\
+        $(x_0, x_1, s) \gets A^{D(\cdot)}(\cookie{find}, P)$; \\
+        \RETURN $(x_0, x_1, s)$;
+    \newline
+      Adversary $A'^{D(\cdot)}(\cookie{choose}, y, s)$: \+ \\
+        \RETURN $([], (y, s))$;
+    \next
+      Adversary $A'^{D(\cdot)}(\cookie{guess}, \vect{x}, s')$: \+ \\
+        $(y, s) \gets s'$; \\
+        $b' \gets A^{D(\cdot)}(y, s)$; \\
+        \RETURN $b'$;
+    \end{program}
+    Evidently $\Adv{nm-\id{atk}}{\mathcal{E}}(A') =
+    \Adv{ind-\id{atk}}{\mathcal{E}}(A)$, and hence
+    \[ \InSec{ind-\id{atk}}(\mathcal{E}; t, q_D) \le
+         \InSec{nm-\id{atk}}(\mathcal{E}; t, q_D), \]%
+    proving the implication.
+    
+  \item We show that $\text{IND-$\id{atk}'$} \implies \text{IND-\id{atk}}$
+    for $(\id{atk}', \id{atk}) \in \{(\text{CCA1}, \text{CPA}), (\text{CCA2},
+    \text{CCA1})\}$.  Suppose that $A$ attacks $\mathcal{E}$ in the
+    $\text{IND-\id{atk}}$ sense.
+    \begin{program}
+      Adversary $A'^{D(\cdot)}(\cookie{find}, P)$: \+ \\
+        $(x_0, x_1, s) \gets A^{D_0(\cdot)}(\cookie{find}, P)$; \\
+        \RETURN $(x_0, x_1, s)$;
+    \next
+      Adversary $A'^{D(\cdot)}(\cookie{guess}, y, s)$: \+ \\
+        $b' \gets A^{D_1(\cdot)}(\cookie{guess}, y, s)$; \\
+        \RETURN $b'$;
+    \end{program}
+    The oracles $D_0$ and $D_1$ are defined according to \id{atk}, as
+    shown in table~\ref{tab:se-rel-oracle}.
+    \begin{table}
+      \begin{tabular}[C]{l Mc Mc}        \hlx*{hv}
+        \id{atk}    & D_0(x) & D_1(x) \\ \hlx{vhv}
+        CPA         & \bot   & \bot   \\
+        CCA1        & D(x)   & \bot   \\ \hlx*{vh}
+      \end{tabular}
+      \caption{Relations between notions of security for public key
+        encryption: decryption oracles}
+      \label{tab:se-rel-oracle}
+    \end{table}
+    Evidently $\Adv{ind-$\id{atk}'$}{\mathcal{E}}(A') =
+    \Adv{ind-\id{atk}}{\mathcal{E}}(A)$, and hence
+    \[ \InSec{ind-\id{atk}}(\mathcal{E}; t, q_D) \le
+         \InSec{ind-$\id{atk}'$}(\mathcal{E}; t, q_D), \]%
+    proving the implication.
+
+  \item We show that $\text{NM-$\id{atk}'$} \implies \text{NM-\id{atk}}$
+    for $(\id{atk}', \id{atk}) \in \{(\text{CCA1}, \text{CPA}), (\text{CCA2},
+    \text{CCA1})\}$.  Suppose that $A$ attacks $\mathcal{E}$ in the
+    $\text{NM-\id{atk}}$ sense.
+    \begin{program}
+      Adversary $A'^{D(\cdot)}(\cookie{find}, P)$: \+ \\
+        $(x_0, x_1, s) \gets A^{D_0(\cdot)}(\cookie{find}, P)$; \\
+        \RETURN $(x_0, x_1, s)$;
+    \newline
+      Adversary $A'^{D(\cdot)}(\cookie{choose}, y, s)$: \+ \\
+        $(\vect{y}, t) \gets A^{D_1(\cdot)}(\cookie{choose}, y, s)$; \\
+        \RETURN $(\vect{y}, t)$;
+    \next
+      Adversary $A'^{D(\cdot)}(\cookie{guess}, \vect{x}, t)$: \+ \\
+        $b' \gets A^{D_1(\cdot)}(\cookie{guess}, \vect{x}, t)$; \\
+        \RETURN $b'$;
+    \end{program}
+    The oracles $D_0$ and $D_1$ are defined according to $\id{atk}'$, as
+    shown in table~\ref{tab:se-rel-oracle}.  Evidently
+    $\Adv{nm-$\id{atk}'$}{\mathcal{E}}(A') =
+    \Adv{nm-\id{atk}}{\mathcal{E}}(A)$, and hence
+    \[ \InSec{nm-\id{atk}}(\mathcal{E}; t, q_D) \le
+         \InSec{nm-$\id{atk}'$}(\mathcal{E}; t, q_D), \]%
+    proving the implication.
+
+  \item We show that $\text{IND-CCA2} \implies \text{NM-CCA2}$.  Suppose that
+    $A$ is an adversary attacking $\mathcal{E}$ in the NM-CCA2 sense.
+    \begin{program}
+      Adversary $A'^{D(\cdot)}(\cookie{find}, P)$: \+ \\
+        $(x_0, x_1, s) \gets A^{D_0(\cdot)}(\cookie{find}, P)$; \\
+        \RETURN $(x_0, x_1, s)$;
+    \next
+      Adversary $A'^{D(\cdot)}(\cookie{guess}, y, s)$: \+ \\
+        $(\vect{y}, t) \gets A^{D(\cdot)}(\cookie{choose}, y, s)$; \\
+        $\vect{x} = D(\vect{y})$; \\
+        $b' \gets A^{D(\cdot)}(\cookie{guess}, \vect{x}, t)$; \\
+        \RETURN $b'$;
+    \end{program}
+    Evidently $\Adv{ind-cca2}{\mathcal{E}}(A') =
+    \Adv{nm-cca2}{\mathcal{E}}(A)$, and hence
+    \[ \InSec{nm-cca2}(\mathcal{E}; t, q_D) \le
+         \InSec{ind-cca2}(\mathcal{E}; t, q_D), \]%
+    proving the implication.
+    
+  \item We show that $\text{IND-CCA1} \not\implies \text{NM-CPA}$.  Suppose
+    that $\mathcal{E} = (G, E, D)$ is secure in the IND-CCA1 sense.  Consider
+    the encryption scheme $\mathcal{E}' = (G', E', D')$:
+    \begin{program}
+      Algorithm $G'(k)$: \+ \\
+        $(P, K) \gets G$; \\
+        \RETURN $(P, K)$;
+    \next
+      Algorithm $E'_P(x)$: \+ \\
+        $y \gets E_P(x)$; \\
+        \RETURN $(0, y)$;
+    \next
+      Algorithm $D'_K(y')$: \+ \\
+        $(b, y) \gets y'$; \\
+        $x \gets D_K(y)$; \\
+        \RETURN $x$;
+    \end{program}
+    This is a classic example of a malleable encryption scheme.
+
+    Firstly, we show that $\mathcal{E}'$ is still IND-CCA1.  Suppose that
+    $A'$ is an adversary attacking $\mathcal{E}'$ in the sense of IND-CCA1.
+    Consider $A$, attacking $\mathcal{E}$:
+    \begin{program}
+      Adversary $A^{D(\cdot)}(\cookie{find}, P)$: \+ \\
+        $(x_0, x_1, s) \gets A'^{\Xid{D'}{sim}(\cdot)}(\cookie{find}, P)$; \\
+        \RETURN $(x_0, x_1, s)$; \- \\[\smallskipamount]
+      Oracle $\Xid{D'}{sim}(y')$: \+ \\
+        $(b, y) \gets y'$; \\
+        $x \gets D(y)$; \\
+        \RETURN $x$;
+    \next
+      Adversary $A^\bot(\cookie{guess}, y, s)$: \+ \\
+        $b' \gets A'^\bot(\cookie{guess}, y, s)$; \\
+        \RETURN $b'$;
+    \end{program}
+    Clearly, $A$ simulates the expected environment for $A'$ perfectly; hence
+    \[ \Adv{ind-cca1}{\mathcal{E}'}(A') = \Adv{ind-cca1}{\mathcal{E}}(A). \]
+
+    Now, we show that $\mathcal{E}'$ is not NM-CPA.  Consider the adversary
+    $C'$:
+    \begin{program}
+      Adversary $C'^{D(\cdot)}(\cookie{find}, P')$: \+ \\
+        \RETURN $(0, 1, \bot)$;
+    \newline
+      Adversary $C'^{D(\cdot)}(\cookie{choose}, y', s)$: \+ \\
+        $(b, y) \gets y'$; \\
+        $\vect{y} \gets [(1, y)]$; \\
+        \RETURN $(\vect{y}, \bot)$;
+    \next
+      Adversary $C'^{D(\cdot)}(\cookie{guess}, \vect{x}, t)$: \+ \\
+        \RETURN $\vect{x}[0]$;
+    \end{program}
+    Since $(0, y) \ne (1, y)$, the experiment provides the plaintext
+    corresponding to the challenge ciphertext $y'$.
+    $\Adv{nm-cpa}{\mathcal{E}'}(C') = 1$.  Hence, $\mathcal{E}'$ is insecure
+    in the NM-CPA sense.
+
+  \item We show that $\text{NM-CPA} \not\implies \text{IND-CCA1}$.  Suppose
+    that $\mathcal{E} = (G, E, D)$ is secure in the NM-CPA sense.  Fix a
+    security parameter $k$.  Consider the encryption scheme $\mathcal{E}' =
+    (G', E', D')$:
+    \begin{program}
+      Algorithm $G'(k)$: \+ \\
+        $(P, K) \gets G(k)$; \\
+        $u \getsr \{0, 1\}^k$; $v \getsr \{0, 1\}^k$; \\
+        \RETURN $((P, u), (K, u, v))$;
+    \next
+      Algorithm $E'_{P'}(x)$: \+ \\
+        $(P, u) \gets P'$; \\
+        $y \gets E_P(x)$; \\
+        \RETURN $(0, y)$;
+    \next
+      Algorithm $D'_{K'}(y')$: \+ \\
+        $(b, y) \gets y'$; \\
+        $(K, u, v) \gets K'$; \\
+        \IF $b = 0$ \THEN $x \gets D_K(y)$; \\
+        \ELSE \IF $y = u$ \THEN $x \gets v$; \\
+        \ELSE \IF $y = v$ \THEN $x \gets K$; \\
+        \ELSE $x \gets \bot$; \\
+        \RETURN $x$;
+    \end{program}
+    The idea is that the decryption oracle can be used to leak the key by
+    following the little $u \to v \to K$ chain, but the parallel
+    non-malleability oracle can't do this.
+
+    We first show that $\mathcal{E}'$ is still NM-CPA.  Suppose that $A'$ is
+    an adversary attacking $\mathcal{E}'$ in the NM-CPA sense.  Consider
+    this adversary $A$:
+    \begin{program}
+      Adversary $A^\bot(\cookie{find}, P)$: \+ \\
+        $u \getsr \{0, 1\}^k$; $v \getsr \{0, 1\}^k$; \\        
+        $(x_0, x_1, s') \gets A'^\bot(\cookie{find}, (P, u))$; \\
+        \RETURN $(x_0, x_1, (s', u, v))$;
+    \newline
+      Adversary $A^\bot(\cookie{choose}, y, s)$: \+ \\
+        $(s', u, v) \gets s$; \\
+        $(\vect{y}', t') \gets A'^\bot(\cookie{choose}, y, s')$; \\
+        \FOR $j = 0$ \TO $|\vect{y}'| - 1$ \DO
+          $\vect{x}'[j] \gets \bot$; \\
+        \FOR $j = 0$ \TO $|\vect{y}'| - 1$ \DO \\ \quad \= \+ \kill
+          $(b', y') \gets \vect{y}'[j]$; \\
+          \IF $b' = 0$ \THEN $\vect{y}[j] \gets y'$; \\
+          \ELSE \IF $y' = u$ \THEN \= $\vect{x}'[j] \gets v$; \\
+                                   \> $\vect{y}[j] \gets \bot$; \\
+          \ELSE \IF $y' = v$ \THEN \ABORT; \\
+          \ELSE \= $\vect{x}'[j] \gets \bot$; \\
+                \> $\vect{y}[j] \gets \bot$; \\
+        \RETURN $(\vect{y}, (\vect{x}', t'))$;
+    \next
+      Adversary $A^\bot(\cookie{guess}, \vect{x}, t)$: \+ \\
+        $(\vect{x}', t') \gets t$; \\
+        \FOR $j = 0$ \TO $|\vect{x}| - 1$ \DO \\
+        \quad \IF $\vect{x}'[j] = \bot$ \THEN
+          $\vect{x}'[j] \gets \vect{x}[j]$; \\
+        $b' \gets A'^\bot(\cookie{guess}, \vect{x}', t')$; \\
+        \RETURN $b'$;
+    \end{program}
+    Clearly, $A$ behaves indistinguishably from the NM-CPA experiment
+    expected by $A'$, unless $A$ aborts because $A'$ guesses $v$ during its
+    $\cookie{choose}$ phase.  But $v$ is independent of $A'$'s view at that
+    moment, so the probability this occurs is $2^{-k}$.  Hence
+    \[ \Adv{nm-cpa}{\mathcal{E}'} \le
+         \Adv{nm-cpa}{\mathcal{E}} + \frac{1}{2^k}. \]%
+
+    Now to show that $\mathcal{E}'$ is insecure in the IND-CCA1 sense.
+    Consider this adversary:
+    \begin{program}
+      Adversary $C'^{D(\cdot)}(\cookie{find}, P')$: \+ \\
+        $(P, u) \gets P'$; \\
+        $v \gets D(u)$; $K \gets D(v)$; \\
+        \RETURN $(0, 1, K)$;
+    \next
+      Adversary $C'^\bot(\cookie{guess}, y, K)$: \+ \\
+        $b' \gets D_K(y)$; \\
+        \RETURN $b'$;
+    \end{program}
+    The adversary $C'$ makes 2 decryption queries, and
+    $\Adv{ind-cca1}{\mathcal{E}'}(C') = 1$.  Hence, $\mathcal{E}'$ is
+    insecure in the IND-CCA1 sense.
+    
+  \item We show that $\text{NM-CCA1} \not\implies \text{IND-CCA2}$.  Suppose
+    that $\mathcal{E} = (G, E, D)$ is secure in the NM-CCA1 sense.  Let
+    $\mathcal{M} = (T, V)$ be a secure MAC.  Consider the encryption scheme
+    $\mathcal{E}' = (G', E', D')$:
+    \begin{program}
+      Algorithm $G'(k)$: \+ \\
+        $(P, K) \gets G(k)$; \\
+        $i \getsr \keys \mathcal{M}$; \\
+        \RETURN $(P, (K, i))$;
+    \next
+      Algorithm $E'_P(x)$: \+ \\
+        $y \gets E_P(x)$; \\
+        \RETURN $(0, y, \bot)$;
+    \next
+      Algorithm $D'_{K'}(y')$: \+ \\
+        $(b, y, \tau) \gets y'$; \\
+        $(K, i) \gets K'$; \\
+        \IF $b = 0$ \THEN $x \gets D_K(y)$; \\
+        \ELSE \IF $\tau = \bot$ \THEN $x \gets T_i(y)$; \\
+        \ELSE \IF $V_i(y, \tau) = 1$ \THEN $x \gets D_K(y)$; \\
+        \ELSE $x \gets \bot$; \\
+        \RETURN $x$;
+    \end{program}
+    Answering decryption queries only when presented with a correct tag
+    ensures that the NM-CCA1 adversary can't obtain anything useful with its
+    queries (hence the scheme remains NM-CCA1-secure), but the IND-CCA2
+    adversary can use its adaptive queries to discover the required tag.
+
+    Firstly, we show that $\mathcal{E}'$ is still NM-CCA1-secure.  Suppose
+    $A'$ attacks $\mathcal{E}'$ in the sense of NM-CCA1.  Consider the two
+    adversaries shown in \ref{fig:nm-cca1-sep-ind-cca2}: $A$ attacks the
+    original $\mathcal{E}$; $A''$ attacks the MAC $\mathcal{M}$.
+    \begin{figure}
+    \begin{program}
+      Adversary $A^{D(\cdot)}(\cookie{find}, P)$: \+ \\
+        $i \getsr \keys F$; \\
+        $(x_0, x_1, s') \gets A'^{\Xid{D}{sim}(\cdot)}(\cookie{find}, P)$; \\
+        \RETURN $(x_0, x_1, (s', i))$; \- \\[\smallskipamount]
+      Adversary $A^\bot(\cookie{choose}, y, s)$: \+ \\
+        $(s', i) \gets s$; \\
+        $(\vect{y}', t') \gets A'^\bot(\cookie{choose}, y, s')$; \\
+        \FOR $j = 0$ \TO $|\vect{y}'| - 1$ \DO
+          $\vect{x}'[j] \gets \bot$; \\
+        \FOR $j = 0$ \TO $|\vect{y}'| - 1$ \DO \\ \quad \= \+ \kill
+          $(b', y', \tau') \gets \vect{y}'[j]$; \\
+          \IF $b' = 0$ \THEN $\vect{y}[j] \gets y'$; \\
+          \ELSE \IF $z' = \bot$ \THEN \= $\vect{x}'[j] \gets T_i(y')$; \\
+                                      \> $\vect{y}[j] \gets \bot$; \\
+          \ELSE \IF $V_i(y', \tau') \ne 1$
+          \THEN $\vect{y}[j] \gets \bot$; \\
+          \ELSE \IF $y' = y$ \THEN \ABORT; \\
+          \ELSE $\vect{y}[j] \gets y'$; \- \\
+        \RETURN $(\vect{y}, (\vect{x}', t'))$; \- \\[\smallskipamount]
+      Adversary $A^\bot(\cookie{guess}, \vect{x}, t)$: \+ \\
+        $(\vect{x}', t') \gets t$; \\
+        \FOR $j = 0$ \TO $|\vect{x}| - 1$ \DO \\
+        \quad \IF $\vect{x}'[j] = \bot$ \THEN
+          $\vect{x}'[j] \gets \vect{x}[j]$; \\
+        $b' \gets A'^\bot(\cookie{guess}, \vect{x}', t')$; \\
+        \RETURN $b'$;
+    \next
+      Adversary $A''^{T(\cdot), V(\cdot)}$: \+ \\
+        $(P, K) \gets G$;
+        $b \getsr \{0, 1\}$; \\
+        $(x_0, x_1, s) \gets A'^{\Xid{D}{sim}(\cdot)}(\cookie{find}, P)$; \\
+        $y \gets (0, E_P(x_b), \bot)$; \\
+        $(\vect{y}, t) \gets A'^\bot(\cookie{choose}, y, s)$; \\
+        \FOR $j = 0$ \TO $|\vect{y}| - 1$ \DO \\ \quad \= \+ \kill
+          $(b', y', \tau') \gets \vect{y}'[j]$; \\
+          \IF $b' = 1 \land V(y', \tau') = 1$
+          \THEN \RETURN $(y', \tau')$; \- \\
+        \ABORT; \- \\[\smallskipamount]
+      Oracle $\Xid{D}{sim}(y')$: \+ \\
+        $(b, y, \tau) \gets y'$; \\
+        \IF $b = 0$ \THEN $x \gets D_K(y)$; \\
+        \ELSE \IF $\tau = \bot$ \THEN $x \gets T(y)$; \\
+        \ELSE \IF $V(y, \tau) = 1$ \THEN $x \gets D_K(y)$; \\
+        \ELSE $x \gets \bot$; \\
+        \RETURN $x$;
+    \end{program}
+    \caption{From the proof that $\text{NM-CCA1} \not\implies
+      \text{IND-CCA2}$ -- adversaries $A$ and $A''$, attacking $\mathcal{E}$
+      in the NM-CCA1 sense and the MAC $\mathcal{M}$ respectively}
+    \label{fig:nm-cca1-sep-ind-cca2}
+    \end{figure}
+
+    Suppose, without loss of generality, that if the challenge ciphertext $y$
+    returned to $A'$ matches one of $A'$'s earlier decryption queries then
+    $A'$ wins without making its parallel chosen ciphertext query.
+
+    Let $B$ be the event that $A$ aborts; let $S$ be the event that $A$ wins,
+    and let $S'$ be the event that $A'$ wins.  Since unless it aborts, $A$
+    implements the NM-CCA1 game for $\mathcal{E}'$ perfectly, we must have
+    \[ \Pr[A] = \Pr[A'] - \Pr[B]. \]
+    But $B$ is precisely the event in which $A''$ wins its game.  Hence
+    \[ \Adv{nm-cca1}{\mathcal{E}'}(A') \le
+         \Adv{nm-cca1}{\mathcal{E}}(A) + \Adv{suf-cma}{\mathcal{E}}(A'') \]%
+    proving the claim that $\mathcal{E}'$ remains NM-CCA1 secure.
+
+    Now to show that $\mathcal{E}'$ is not IND-CCA2 secure.
+    \begin{program}
+      Adversary $C'^{D(\cdot)}(\cookie{find}, P')$: \+ \\
+        \RETURN $(0, 1, \bot)$;
+    \next
+      Adversary $C'^{D(\cdot)}(\cookie{guess}, y, s)$: \+ \\
+        $\tau \gets D(1, y, \bot)$; \\
+        $b' \gets D(1, y, \tau)$; \\
+        \RETURN $b'$;
+    \end{program}
+    The adversary $C'$ makes 2 decryption queries, and
+    $\Adv{ind-cca2}{\mathcal{E}'}(C') = 1$.  Hence, $\mathcal{E}'$ is
+    insecure in the IND-CCA2 sense.
+    
+  \end{enumerate}
+  All present and correct.
+\end{proof}
+
+\begin{exercise}
+  In \cite{Goldwasser:1984:PE}, Shafi Goldwasser and Silvio Micali first
+  introduced the concept of semantic security as a definition of
+  \emph{computationally} secure encryption, and presented the first
+  provably-secure probabilistic encryption scheme.  For this scheme, the
+  private key is a pair of $k$-bit primes $(p, q)$; the public key is their
+  product $n = pq$ and a \emph{pseudosquare} $z$.  Given this starting point,
+  define a public-key encryption scheme, and relate its security in the
+  IND-CPA sense to the difficulty of the Quadratic Residuosity Problem.  Show
+  that the encryption scheme is malleable.
+  
+  Hints:
+  \begin{parenum}
+  \item Encrypt the message one bit at a time.
+  \item Choosing a random element of $(\Z/n\Z)^*$ and squaring gives you a
+    random element of $Q_n$.
+  \item You will need to define a formal game for the Quadratic Residuosity
+    Problem.
+  \end{parenum}
+  \answer%
+  Encryption works as $\Xid{E}{GM}_{(n, z)}(x)$: \{ \FOREACH $1\colon x_i$
+  \FROM $x$ \DO \{ $a_i \getsr (\Z/n\Z)^*$; $\vect{y}[i] \gets a_i^2
+  z^{x_i}$~\} \RETURN $\vect{y}$;~\}.  Decryption works as $\Xid{D}{GM}_{(p,
+    q)}(\vect{y})$: \{ $x \gets \emptystring$; \FOR $i = 0$ \TO $|\vect{y}| -
+  1$ \DO \{ \IF $\jacobi{\vect{y}[i]}{p} = 1 \land \jacobi{\vect{y}[i]}{q} =
+  1$ \THEN $x \gets x \cat 0$; \ELSE $x \gets x \cat 1$;~\} \RETURN $x$;~\}.
+
+  To prove that this is meets IND-CPA, let $A$ be an adversary attacking the
+  GM scheme.  Now, we present an algorithm which, given an odd composite
+  integer $n$ and an element $z$ with $\jacobi{x}{n} = 1$, decides whether $x
+  \in Q_n$: Algorithm $B(n, z)$: $(x_0, x_1, s) \gets A(\cookie{find}, (n,
+  z))$; $b \getsr \{0, 1\}$; $y \gets \Xid{E}{GM}_{(n, z)}(x_b)$; $b' \gets
+  A(\cookie{guess}, y, s)$; \IF $b = b'$ \THEN \RETURN $0$; \ELSE \RETURN
+  $1$;~\}.  If $z$ is a nonresidue, then $B$ returns $0$ whenever $A$
+  successfully guesses the right bit; if $z \in Q_n$ then the ciphertext $y$
+  returned by $B$ is a string of random quadratic residues independent of the
+  challenge plaintext, and $A$ can have no advantage.  Hence
+  $\InSec{ind-cpa}(\Xid{\mathcal{E}}{GM-$k$}; t) \le 2 \cdot \InSec{qrp}(k;
+  t)$.
+  
+  To prove malleability, simply note that multiplying an element
+  $\vect{y}[i]$ by $z$ toggles the corresponding plaintext bit.
+\end{exercise}
+
+\xcalways\subsection{The ElGamal scheme}\x
+
+\begin{slide}
+  \topic{description}
+  \head{The ElGamal public-key encryption scheme}
+
+  ElGamal's encryption scheme is based on Diffie-Hellman.  Let $G = \langle g
+  \rangle$ be a cyclic group of order $q$.  Plaintexts and ciphertexts in the
+  scheme are elements of $G$.
+
+  The scheme $\Xid{\mathcal{E}}{ElGamal}^G = (\Xid{G}{ElGamal}^G,
+  \Xid{E}{ElGamal}^G, \Xid{D}{ElGamal}^G)$ is defined by:
+  \begin{program}
+    $\Xid{G}{ElGamal}^G$: \+ \\
+      $\alpha \getsr \{1, 2, \ldots, q - 1\}$; \\
+      \RETURN $(g^\alpha, \alpha)$;
+  \next
+    $\Xid{E}{ElGamal}^G_a(x)$: \+ \\
+      $\beta \getsr \{1, 2, \ldots, q - 1\}$; \\
+      \RETURN $(g^\beta, x a^\beta)$;
+  \next
+    $\Xid{D}{ElGamal}^G_\alpha(y)$: \+ \\
+      $(b, c) \gets y$; \\
+      $x \gets b^{-\alpha} c$; \\
+      \RETURN $x$;
+  \end{program}
+  This scheme is secure in the IND-CPA sense if the Decisional Diffie-Hellman
+  problem is hard in $G$.
+\end{slide}
+
+\begin{slide}
+  \topic{security proof}
+  \head{Security proof for ElGamal}
+
+  Suppose $A$ is an adversary attacking the ElGamal scheme in the IND-CPA
+  sense.  We construct from it an algorithm which solves the DDH problem
+  (i.e., given a triple $g^\alpha, g^\beta, c$, decides whether $c =
+  g^{\alpha\beta}$):
+  \begin{program}
+    Algorithm $D(a, b, c)$: \+ \\
+      $(x_0, x_1, s) \gets A(\cookie{find}, a)$; \\
+      $z \getsr \{0, 1\}$; \\
+      $y \gets (b, x_z c)$; \\
+      $z' \gets A(\cookie{guess}, y, s)$; \\
+      \IF $z = z'$ \THEN \RETURN $1$; \\
+      \ELSE \RETURN $0$;
+  \end{program}
+\end{slide}
+
+\begin{slide}
+  \head{Security proof for ElGamal (cont.)}
+  
+  Let $\alpha$ and $\beta$ be the discrete logs of $a$ and $b$.
+  
+  If $c = g^{\alpha\beta}$, then $D$'s success probability is equal to $A$'s
+  probability of guessing the hidden bit correctly, which is
+  \[ \frac{\Adv{ind-cpa}{\Xid{\mathcal{E}}{ElGamal}^G}(A)}{2} + \frac{1}{2}. \]%
+  If $c$ is random, then $x_z c$ is uniformly distributed in $G$, and
+  independent of $b$, so $A$ answers correctly with probability exactly
+  $\frac{1}{2}$.
+  
+  Hence, $\Adv{ddh}{G}(D) =
+  \Adv{ind-cpa}{\Xid{\mathcal{E}}{ElGamal}^G}(A)/2$, and
+  \[ \InSec{ind-cpa}(\Xid{\mathcal{E}}{ElGamal}^G; t) \le
+     2 \cdot \InSec{ddh}(G; t). \]%
+\end{slide}
+
+\begin{slide}
+  \topic{other notes}
+  \head{Notes about ElGamal}
+
+  \begin{itemize}
+    
+  \item We needed the Decisional Diffie-Hellman assumption to prove the
+    security.  As noted before, this is a very strong assumption.  Still, a
+    proof based on DDH is a lot better than nothing.
+    
+    We really do need the Decisional Diffie-Hellman assumption.  An adversary
+    with a DDH algorithm can submit $x_0 \inr G$ and $x_1 = 1$; it receives a
+    ciphertext $(b, y)$, and returns $1$ if $(a, b, y)$ looks like a
+    Diffie-Hellman triple, or $0$ if it looks random.
+    
+  \item The plaintexts must be elements of the cyclic group $G$.  For
+    example, if $G$ is a subgroup of $\F_p^*$, it's \emph{not} safe to allow
+    elements outside the subgroup as plaintexts: an adversary can compare
+    orders of ciphertext elements to break the semantic security of the
+    scheme.
+
+  \item ElGamal is malleable.  We can decrypt a challenge ciphertext $y =
+    (g^\beta, a^\beta x)$ by choosing a random $\gamma$ and requesting a
+    decryption of $y' = (g^{\beta\gamma}, a^{\beta\gamma} x^\gamma)$.
+
+  \end{itemize}
+\end{slide}
+
+\xcalways\subsection{Encrypting using trapdoor one-way functions}\x
+
+\begin{slide}
+  \head{Trapdoor one-way functions}
+  
+  Trapdoor one-way functions RSA ($x \mapsto x^e \bmod n$) and Rabin ($x
+  \mapsto x^2 \bmod n$) functions are well-studied.  Can we make secure
+  schemes from them?
+
+  We can't use them directly, however.  For a start, the functions are
+  deterministic, so `encrypting' messages just by doing the modular
+  exponentiation on the plaintext will leak equality of plaintexts.
+
+  How can we fix these schemes?
+
+  We'll focus on RSA, because decryption is simpler; Rabin works in a very
+  similar way.
+\end{slide}
+
+\begin{slide}
+  \topic{simple embedding schemes}
+  \head{Simple embedding schemes}
+  
+  If we restrict our attention to plaintext messages which are `about' the
+  input size of our one-way function, we'd like to be able to use ciphertexts
+  which are no bigger than the output size of the function.
+
+  Perhaps if we encode the message in some way before passing it through the
+  function, we can improve security.  Ideally, we'd like security against
+  chosen-ciphertext attacks.
+
+  An encryption scheme which processes messages like this is called a
+  \emph{simple embedding scheme}.
+\end{slide}
+
+\begin{slide}
+  \topic{\PKCS1 padding}
+  \head{\PKCS1 encryption padding for RSA \cite{RSA:1993:PRE}}
+
+  Let $n = p q$ be an RSA modulus, with $2^{8(k-1)} < n < 2^{8k)}$ -- i.e.,
+  $n$ is a $k$-byte number.  Let $m$ be the message to be signed.
+  
+  We compute $\hat{m} = 0^8 \cat T \cat r \cat 0^8 \cat x$ for
+  some hash function $m$, where:
+  \begin{itemize}
+  \item $|\hat{m}| = 8k$, i.e., $\hat{m}$ is a $k$-byte string;
+  \item $0^8$ is the string of 8 zero-bits;
+  \item $T = 00000002$ is an 8-bit \emph{type} code; and
+  \item $r$ is a string of random \emph{non-zero} bytes (\PKCS1 requires at
+    least 8 bytes)
+  \end{itemize}
+  This bit string is then converted into an integer, using a big-endian
+  representation.  Note that $\hat{m} < n$, since its top byte is zero.
+\end{slide}
+
+\begin{slide}
+  \head{\PKCS1 encryption padding for RSA (cont.)}
+
+  Diagramatically, \PKCS1 encryption padding looks like this:
+  \begin{tabular}[C]{r|c|c|c|c|} \hlx{c{2-5}v}
+    \hex{00} & \hex{01} &
+    \ldots\ random nonzero bytes \ldots &
+    \hex{00} &
+    $m$
+    \\ \hlx{vc{2-5}}
+  \end{tabular}
+  
+  Unfortunately, this scheme isn't capable of resisting chosen-ciphertext
+  attacks: Bleichenbacher \cite{Bleichenbacher:1998:CCA} shows how to decrypt
+  a ciphertext $y$ given an oracle (say, an SSL server) which returns whether
+  a given ciphertext is valid (i.e., its padding is correct).
+\end{slide}
+
+\begin{slide}
+  \topic{Optimal Asymmetric Encryption Padding (OAEP)}
+  \head{Optimal Asymmetric Encryption Padding (OAEP) \cite{Bellare:1995:OAE}}
+  
+  OAEP is a simple embedding scheme for use with an arbitrary trapdoor
+  one-way function.  It requires two hash functions, $H$ and $H'$, which we
+  model as random oracles.
+
+  We assume that our one-way function $f_P$ operates on $n$-bit strings.  Fix
+  a security parameter $k$.  We require two random oracles $G\colon \{0,
+  1\}^k \to \{0, 1\}^{n-k}$ and $H\colon \{0, 1\}^{n-k} \to \{0, 1\}^k$.
+  Given a message $x \in \{0, 1\}^{n-2k}$, we encrypt it as follows:
+  \begin{program}
+    Algorithm $\Xid{E}{OAEP}^{\mathcal{T}, G(\cdot), H(\cdot)}_P(x)$: \+ \\
+      $r \getsr \{0, 1\}^k$; \\
+      $s \gets (x \cat 0^k) \xor G(r)$; $t \gets r \xor H(s)$; \\
+      $w \gets s \cat t$; \\
+      \RETURN $f_P(w)$;
+  \next
+    Algorithm $\Xid{D}{OAEP}^{\mathcal{T}, G(\cdot), H(\cdot)}_K(y)$: \+ \\
+      $w \gets T_K(y)$; \\
+      \PARSE $w$ \AS $s, k\colon t$; \\
+      $r \gets t \xor H(s)$; $x' \gets s \xor G(r)$; \\
+      \PARSE $x'$ \AS $x, k\colon c$; \\
+      \IF $c = 0^k$ \THEN \RETURN $x$; \\
+      \ELSE \RETURN $\bot$;
+  \end{program}
+\end{slide}
+
+\begin{slide}
+  \head{Optimal Asymmetric Encryption Padding (OAEP) (cont.)}
+
+  %%        x <- {0, 1}^{n-2k}               r <-R {0, 1}^k
+  %%                |                              |
+  %%                |                              |
+  %%                v                              |
+  %%               [||]<--- 0^k                    |
+  %%                |                              |
+  %%                |                              |
+  %%                v                              |
+  %%               (+)<-----------[G]--------------|
+  %%                |                              |
+  %%                |                              |
+  %%                |                              v
+  %%                |-------------[H]------------>(+)
+  %%                |                              |
+  %%                |                              |
+  %%                v                              v
+  %%    < s = (x || 0^k) (+) G(r) >       < t = r (+) H(s) >
+
+  \vfil
+  \[ \begin{graph}
+    []!{0; <1cm, 0cm>:}
+    {x \in \{0, 1\}^n}="x" [rrrr] {r \inr \{0, 1\}^k}="r"
+    "r" :[ddd] *{\xor}="h-xor"
+        :[d] *++=(2, 0)[F:thicker]{\strut t = r \xor H(s)}
+    "x" :[d] *{\ocat}="cat"
+        [r] *+[r]{\mathstrut 0^k} :"cat"
+        :[d] *{\xor}="g-xor"
+        :[dd] *++=(4, 0)[F:thicker]{\strut s = (x \cat 0^k) \xor G(r)}
+        [u] :[rr] *+[F]{H'} :"h-xor"
+    "r" [dd] :[ll] *+[F]{H} :"g-xor"
+  \end{graph} \]
+  \vfil
+\end{slide}
+
+\begin{slide}
+  \head{Security of OAEP, 1: chosen plaintext attacks}
+  
+  We write the encryption scheme formed by using the trapdoor one-way
+  function $\mathcal{T}$ with OAEP embedding as
+  $\Xid{\mathcal{E}}{OAEP}^{\mathcal{T}}$.
+
+  The security of OAEP in the IND-CPA sense is given by:
+  \[ \InSec{ind-cpa}(\Xid{\mathcal{E}}{OAEP}^{\mathcal{T}}; t, q_G, q_H)
+     \le 2 \left (\frac{q_G}{2^k} +
+                    \InSec{owf}(\mathcal{T}; t + O(q_G q_H)) \right). \]%
+\end{slide}
+
+We omit the security proof of OAEP, because it's very similar to the proof
+for OAEP+ which is presented in full later.
+
+\begin{slide}
+  \topic{possible malleability of OAEP}
+  \head{OAEP is not (generically) non-malleable \cite{Shoup:2001:OAEPR}}
+  
+  If a one-way function might be \emph{XOR-malleable} -- i.e., given $f_P(x)$
+  and $\delta$, it's possible to construct $f_P(x \xor \delta)$ without
+  knowledge of the trapdoor -- then the OAEP encryption scheme is malleable.
+
+  \begin{graph}
+    []!{0; <1cm, 0cm>:}
+    *!UR{\parbox{0.5\linewidth}{
+      \raggedright
+      We need to suppose, in addition, that the function leaks the leftmost
+      $n-k$ bits of its input; e.g., $f(s \cat t) = s \cat f'(t)$.  Then
+      exploiting the differential (pictured right) only requires a pair of
+      queries to the $H$ oracle, not $G$.  Using its parallel
+      chosen-ciphertext query, the adversary can decrypt its target
+      plaintext.
+    }}
+    !{+(1.5, 0)}
+    {x}="x" [rr] {r}="r"
+    "r" :[ddd] *{\xor}="h-xor" ^{0}
+        :[d] {t}^{H(s) \xor H(s \xor (\delta \cat 0^k))}
+    "x" :[d] *{\ocat}="cat" _{\delta}
+        [r] *+[r]{\mathstrut 0^k} :"cat"
+        :[d] *{\xor}="g-xor" _{\delta \cat 0^k}
+        :[dd] {s}_{\delta \cat 0^k}
+        [u] :[r] *+[F]{H'} :"h-xor"
+    "r" [dd] :[l] *+[F]{H} :"g-xor"
+  \end{graph}
+
+  Nevertheless, OAEP \emph{does} provide security against chosen-ciphertext
+  attacks when used with the RSA and Rabin functions.
+\end{slide}
+
+\xcalways\subsection{Plaintext awareness and OAEP+}\x
+
+\begin{slide}
+  \head{Plaintext awareness}
+  
+  The intuitive idea behind plaintext awareness is that it's hard to
+  construct a new ciphertext for which you can't easily guess the plaintext
+  (or guess that the ciphertext is invalid).  Obviously, such an idea would
+  imply security against chosen ciphertext attack -- since the adversary
+  effectively knows the plaintext anyway, the decryption oracle is useless.
+  
+  The formalization introduces a \emph{plaintext extractor} -- an algorithm
+  which, given a ciphertext and the random oracle queries of the program
+  which created it, returns the corresponding plaintext.
+  
+  Note, then, that plaintext awareness (as currently defined) only works in
+  the random oracle model.
+\end{slide}
+
+\begin{slide}
+  \topic{definition of plaintext awareness}
+  \head{Definition of plaintext awareness \cite{Bellare:1998:RAN}}
+  
+  Let $\mathcal{E} = (G, E, D)$ be a public-key encryption scheme.  Consider
+  an adversary $A$ which takes a public key as input and returns a ciphertext
+  $y$.  We run the adversary, providing it with an \emph{encryption} oracle,
+  and record
+  \begin{itemize}
+  \item the set $Q_H$, which contains a pair $(q, h)$ for each random oracle
+    query $q$ made by the adversary, together with its reply $h$; and
+  \item the set $C$, which contains the responses (only: not the queries)
+    from $A$'s encryption oracle.
+  \end{itemize}
+  We write $(y, Q_H, C) \gets \id{transcript}(A; z)$ to denote running $A$ on
+  the arguments $z$ and collecting this information.
+
+  Note that $Q_H$ doesn't include the oracle queries required by the
+  encryption oracle.
+\end{slide}
+
+\begin{slide}
+  \head{Definition of plaintext awareness (cont.)}
+
+  An \emph{$\epsilon$-plaintext extractor} for $\mathcal{E}$ is an algorithm
+  $X$ for which
+  \begin{eqnarray*}[rl]
+     \min_A \Pr[
+     &
+       (P, K) \gets G;
+       (y, Q_H, C) \gets \id{transcript}(A^{E(\cdot), H(\cdot)}; P); \\
+     & x \gets X(y, P, Q_H, C) : x = D_K(y)] \ge 1 - \epsilon .
+  \end{eqnarray*}
+  
+  The scheme $\mathcal{E}$ is \emph{$\epsilon$-plaintext aware} (PA) if there
+  exists an $\epsilon$-plaintext extractor for $\mathcal{E}$ \emph{and}
+  $\mathcal{E}$ is IND-CPA.
+
+  (The requirement that the scheme meet IND-CPA prevents trivial schemes such
+  as the identity function from being considered plaintext aware.)
+\end{slide}
+
+\begin{slide}
+  \topic{chosen-ciphertext security}
+  \head{Plaintext awareness and chosen-ciphertext security
+    \cite{Bellare:1998:RAN}}
+
+  If a public key encryption scheme $\mathcal{E}$ is PA, then it is also
+  IND-CCA2 (and hence NM-CCA2).  This is proven using the plaintext extractor
+  to simulate the decryption oracle.  The encryption oracle in the PA
+  definition is used to feed the challenge ciphertext to the plaintext
+  extractor.
+
+  Quantatively, if $\mathcal{E}$ is $\epsilon$-plaintext aware, and the
+  plaintext extractor runs in time $t_X(q_H)$, then
+  \[ \InSec{ind-cca2}(\mathcal{E}; t, q_D, q_H) \le
+     \InSec{ind-cpa}(\mathcal{E}; t + q_D t_X(q_H), q_H) + q_D \epsilon. \]%
+
+  The converse is not true: IND-CCA2 does not imply plaintext awareness.
+\end{slide}
+
+\begin{proof}
+  Firstly, we prove that $\text{PA} \implies \text{IND-CCA2}$.  Let
+  $\mathcal{E} = (G, E, D)$ be an $\epsilon$-plaintext aware public-key
+  encryption scheme.  Suppose $A'$ is an adversary attacking $\mathcal{E}$ in
+  the IND-CCA2 sense.  Let $X$ be the $\epsilon$-plaintext extractor for
+  $\mathcal{E}$.  We construct an adversary attacking $\mathcal{E}$ in the
+  IND-CPA sense.
+  \begin{program}
+    Adversary $A^{H(\cdot)}(\cookie{find}, P)$: \+ \\
+      $\Xid{H}{map} \gets \emptyset$; $y^* \gets \bot$; \\
+      $(x_0, x_1, s') \gets A'^{\Xid{D}{sim}(\cdot), \Xid{H}{sim}(\cdot)}
+        (\cookie{find}, P)$; \\
+      \RETURN $(x_0, x_1, (\Xid{H}{map}, P, s')$; \- \\[\smallskipamount]
+    Oracle $\Xid{H}{sim}(x)$: \+ \\
+      $h \gets H(x)$; \\
+      $\Xid{H}{map} \gets \Xid{H}{map} \cup \{ x \mapsto h \}$; \\
+      \RETURN $h$;
+  \next
+    Adversary $A^{H(\cdot)}(\cookie{guess}, y^*, s)$: \+ \\
+      $(\Xid{H}{map}, P, s') \gets s$; \\
+      $b \gets A'^{\Xid{D}{sim}(\cdot), \Xid{H}{sim}(\cdot)}
+        (\cookie{guess}, y^*, s')$; \\
+      \RETURN $b$; \- \\[\smallskipamount]
+    Oracle $\Xid{D}{sim}(y)$: \+ \\
+      $x \gets X(y, P, \Xid{H}{map}, \{ y^* \})$; \\
+      \RETURN $x$;
+  \end{program}
+  Let $q_D$ be the number of decryption queries made by the adversary.  We
+  can see that, if the plaintext extractor does its job correctly, $A$
+  simulates the decryption oracle for $A'$ correctly.  Hence
+  \[ \Adv{ind-cpa}{\mathcal{E}}(A) \ge
+       \Adv{ind-cca2}{\mathcal{E}}(A') - q_D \epsilon. \]%
+  Notice how the encryption oracle from the plaintext awareness definition is
+  used in the proof.
+  
+  We can show that $\text{IND-CCA2} \not\implies \text{PA}$ easily enough by
+  amending an existing IND-CCA2 scheme $\mathcal{E}$ so that it has a
+  `magical' ciphertext, attached to the public key.  Here is our modified
+  scheme $\mathcal{E}' = (G', E', D')$:
+  \begin{program}
+    Algorithm $G'^{H(\cdot)}(k)$: \+ \\
+      $(P, K) \gets G^{H(\cdot)}$; \\
+      $p \getsr \dom E^{H(\cdot)}_P$; \\
+      $c \getsr \ran E^{H(\cdot)}_P$; \\
+      \RETURN $((P, c), (K, c, p))$;
+  \next
+    Algorithm $E'^{H(\cdot)}_{P, c}(x)$: \+ \\
+      $y \gets E^{H(\cdot)}_P(x)$; \\
+      \RETURN $y$;
+  \next
+    Algorithm $D'^{H(\cdot)}_{K, c, p}(y)$: \+ \\
+      \IF $y = c$ \THEN $x \gets p$; \\
+      \ELSE $x \gets D^{H(\cdot)}_K(y)$; \\
+      \RETURN $x$;
+  \end{program}
+  Firstly, we show that it still meets IND-CCA2.  Let $A'$ be an adversary
+  attacking $\mathcal{E}'$ in the IND-CCA2 sense.  Then $A$, below, acheives
+  the same advantage against $\mathcal{E}$.
+  \begin{program}
+    Adversary $A^{D(\cdot), H(\cdot)}(\cookie{find}, P)$: \+ \\
+      $p \getsr \dom E^{H(\cdot)}_P$; $c \getsr \ran E^{H(\cdot)}_P$; \\
+      $(x_0, x_1, s') \gets A'^{\Xid{D}{sim}(\cdot), H(\cdot)}
+        (\cookie{find}, (P, c))$; \\
+      \RETURN $(x_0, x_1, (p, c, s')$;
+  \next
+    Adversary $A^{D(\cdot), H(\cdot)}(\cookie{guess}, y, s)$: \+ \\
+      $(p, c, s') \gets s$; \\
+      $b \gets A'^{\Xid{D}{sim}(\cdot), H(\cdot)}(\cookie{guess}, y, s')$; \\
+      \RETURN $y$;
+  \newline
+    Oracle $\Xid{D}{sim}(y)$: \+ \\
+      \IF $y = c$ \THEN $x \gets p$; \\
+      \ELSE $x \gets D(y)$; \\
+      \RETURN $x$;
+  \end{program}
+  
+  Now to show that $\mathcal{E}'$ is not plaintext-aware.  Suppose that it
+  is, and there is a plaintext extractor $X$.  We show that $\mathcal{E}$ is
+  not IND-CPA (contradicting the earlier assumption that $\mathcal{E}$ was
+  IND-CCA2).
+  \begin{program}
+    Adversary $A''^{H(\cdot)}(\cookie{find}, P)$: \+ \\
+      $x_0 \getsr \dom E_P$; $x_1 \getsr \dom E_P$; \\
+      \RETURN $(x_0, x_1, (P, x_1))$;
+  \next
+    Adversary $A''^{H(\cdot)}(\cookie{guess}, y, s)$: \+ \\
+      $(P, x_1) \gets s$; \\
+      $x \gets X(y, (P, y), \emptyset, \emptyset)$; \\
+      \IF $x = x_1$ \THEN \RETURN $1$;
+      \ELSE \RETURN $0$;
+  \end{program}
+  Since $x_0$ and $x_1$ are uniform in $\dom E_P$, the transcript $(y, (P,
+  y), \emptyset, \emptyset)$ passed to the extractor is distributed exactly
+  as for the $\mathcal{E}'$ adversary which, given the public key $(P, c)$
+  returns $c$; hence it succeeds with its usual probability $1 - \epsilon$.
+  Then, $A''$'s advantage is
+  \[ \Adv{ind-cpa}{\mathcal{E}}(A'') =
+       1 - 2\epsilon - \frac{1}{|{\dom E_P}|}. \]%
+  This completes the proof.
+\end{proof}
+
+\begin{slide}
+  \topic{old definition}
+  \head{The old definition of plaintext awareness}
+
+  An earlier definition of plaintext awareness omitted the encryption oracle
+  provided to the adversary.  The designers of OAEP proved that it met this
+  definition of plaintext awareness and asserted, without proof, that this
+  implied security against adaptive chosen-ciphertext attacks.
+
+  The original definition of plaintext-awareness is sufficient to guarantee
+  IND-CCA1 security, but it doesn't provide any sort of non-malleability.
+\end{slide}
+
+\begin{slide}
+  \topic{OAEP+}
+  \head{Plaintext-aware encryption: OAEP+ \cite{Shoup:2001:OAEPR}, 1}
+  
+  OAEP+ is a simple embedding scheme, very similar to OAEP, which acheives
+  plaintext-awareness.
+  
+  We assume that our one-way function $f_P$ operates on $n$-bit strings.  Fix
+  a security parameter $k$.  We require three random oracles $G\colon \{0,
+  1\}^k \to \{0, 1\}^{n-2k}$, $H\colon \{0, 1\}^{n-k} \to \{0, 1\}^k$, and
+  $H'\colon \{0, 1\}^{n-k} \to \{0, 1\}^k$.
+  \begin{program}
+    Algorithm $\Xid{E}{OAEP+}
+                ^{\mathcal{T}, G(\cdot), H(\cdot), H'(\cdot)}_P(x)$: \+ \\
+      $r \getsr \{0, 1\}^k$; \\
+      $c \gets H'(x \cat r)$; \\
+      $s \gets (x \xor G(r)) \cat c$; \\
+      $t \gets r \xor H(s)$; \\
+      $w \gets s \cat t$; \\
+      \RETURN $f_P(w)$;
+  \next
+    Algorithm $\Xid{D}{OAEP+}
+                ^{\mathcal{T}, G(\cdot), H(\cdot), H'(\cdot)}_K(y)$: \+ \\
+      $w \gets T_K(y)$; \\
+      \PARSE $w$ \AS $s, k\colon t$; \\
+      $r \gets t \xor H(s)$; \\
+      \PARSE $s$ \AS $s', k\colon c$; \\
+      $x \gets s' \xor G(r)$; \\
+      \IF $c = H'(x \cat r)$ \THEN \RETURN $x$; \\
+      \ELSE \RETURN $\bot$;
+  \end{program}
+\end{slide}
+
+\begin{slide}
+  \head{Plaintext-aware encryption: OAEP+, 2}
+
+  %%        x <- {0, 1}^{n-2k}                r <-R {0, 1}^k
+  %%                |                              |
+  %%                |                              |
+  %%                |                              |
+  %%                |-------->[H']<----------------|
+  %%                |          |                   |
+  %%                |          |                   |
+  %%                v          |                   |
+  %%               (+)<-------------[G]<-----------|
+  %%                |          |                   |
+  %%                |          |                   |
+  %%                v          |                   |
+  %%               [||]<-------'                   |
+  %%                |                              |
+  %%                |                              |
+  %%                |                              v
+  %%                |-------------->[H]---------->(+)
+  %%                |                              |
+  %%                |                              |
+  %%                v                              v
+  %%  < s = (x (+) G(r)) || H'(x || r) >  < t = r (+) H(s) >
+
+  \vfil
+  \[ \begin{graph}
+    []!{0; <1cm, 0cm>:}
+    {x \in \{0, 1\}^n}="x" [rrrr] {r \inr \{0, 1\}^k}="r"
+    "x" [d] :[r] *+[F]{H'}="h'" "r" [d] :"h'"
+    "r" :[dddd] *{\xor}="h-xor"
+        :[d] *++=(2, 0)[F:thicker]{\strut t = r \xor H(s)}
+    "x" :[dd] *{\xor}="g-xor"
+        :[d] *{\ocat}="cat"
+        :[dd] *++=(4, 0)[F:thicker]
+          {\strut s = (x \xor G(r)) \cat H'(x \cat r)}
+        [u] :[rr] *+[F]{H} :"h-xor"
+    "r" [dd] :[ll] *+[F]{G} :"g-xor"
+    "h'" :'[d]*=<8pt>\cir<4pt>{r_l} `d"cat" "cat"
+  \end{graph} \]
+  \vfil
+\end{slide}
+
+\begin{slide}
+  \head{Plaintext-aware encryption: OAEP+, 3}
+  
+  Let $\mathcal{T}$ be a trapdoor one-way function.  Then the insecurity of
+  the public-key encryption scheme $\Xid{\mathcal{E}}{OAEP+}^{\mathcal{T}}$
+  in the IND-CPA sense is given by
+  \[ \InSec{ind-cpa}(\Xid{\mathcal{E}}{OAEP+}^{\mathcal{T}};
+                     t, q_G, q_H, q_{H'})
+     \le
+     2 \left (\frac{q_G}{2^k} +
+                \InSec{owf}(\mathcal{T}; t + O(q_G q_H)) \right). \]%
+  Also, $\Xid{\mathcal{E}}{OAEP+}^{\mathcal{T}}$ is $(q_G + 1)
+  2^{-k}$-plaintext aware, the plaintext extractor running time being
+  $O(q_{H'} t_f)$, where $t_f$ is the time required to execute the one-way
+  function $f$.
+
+  Tying up the various results, then, we can derive that
+  \begin{eqnarray*}[Ll]
+     \InSec{ind-cca2}(\Xid{\mathcal{E}}{OAEP+}^{\mathcal{T}};
+                      t, q_D, q_G, q_H, q_{H'}) \\
+     & \le
+     \frac{(2 + q_D) (q_G + 1) - 2}{2^k} +
+     2 \cdot \InSec{owf}(\mathcal{T}; t + O(q_G q_H + q_D q_{H'} t_f)).
+  \end{eqnarray*}
+\end{slide}
+
+Before we move on to the OAEP+ security results, we state and prove the
+following lemma, due (I believe) to Victor Shoup.
+
+\begin{lemma}[Shoup]
+  \label{lem:shoup}
+  If $X$, $Y$ and $F$ are events, and
+  \[ \Pr[X \land \lnot F] = \Pr[Y \land \lnot F] \]
+  then
+  \[ |{\Pr[X]} - \Pr[Y]| \le \Pr[F]. \]
+\end{lemma}
+\begin{proof}
+  We have:
+  \begin{eqnarray*}[rll]
+    \Pr[X] &= \Pr[X \land F] &+ \Pr[X \land \lnot F] \\
+    \Pr[Y] &= \Pr[Y \land F] &+ \Pr[Y \land \lnot F]
+  \end{eqnarray*}
+  Subtracting gives
+  \[ |{\Pr[X]} - \Pr[Y]| = |{\Pr[X \land F]} - \Pr[Y \land F]| \le \Pr[F] \]
+  as required.
+\end{proof}
+
+\begin{proof}[Proof of the main OAEP+ result]
+  
+  We assume throughout that, before querying its $H'$ oracle at $x \cat r$,
+  the adversary has queried $G$ at $r$.
+  
+  First, we show that OAEP+ meets the IND-CPA notion.  Suppose that $A$
+  attacks the OAEP+ scheme in the IND-CPA sense.  We shall play a series of
+  games with the adversary, and vary the rules as we go along.  The adversary
+  remains the same throughout.
+  
+  At any point in a game, let $Q_G$ be the list of queries the adversary has
+  made to the oracle $G$, let $Q_H$ be the queries made to $H$, and let
+  $Q_{H'}$ be the queries made to $H'$.
+  
+  \paragraph{Game $\G0$}
+  This is effectively the original attack game: the adversary chooses two
+  plaintexts: one is chosen unformly at random, the adversary is given the
+  ciphertext and asked to identify which plaintext was chosen.  Label the
+  selected plaintext $x^*$, and the target ciphertext $y^*$.  Let $c^*$,
+  $s^*$, $t^*$ and $w^*$ be the intermediate results of the OAEP+ padding, as
+  described above.  Let $S_0$ be the event that the adversary wins the game.
+  Our objective is to bound the probabilty $\Pr[S_0] =
+  (\Adv{ind-cpa}{\Xid{\mathcal{E}}{OAEP+}^{\mathcal{T}}}(A) + 1)/2$.
+
+  \paragraph{Game $\G1$}
+  At the beginning of the game, we select at random:
+  \begin{itemize}
+  \item $s^* \inr \{0, 1\}^{n-k}$;
+  \item $t^* \inr \{0, 1\}^k$; and
+  \item $r^* \inr \{0, 1\}^k$.
+  \end{itemize}
+  We compute $w^* = s^* \cat t^*$ and $y^* = f_P(w^*)$.  Note, then, that
+  \emph{$y^*$ is fixed at the start of the game}.
+
+  We also `rig' the random oracle $H$ so that $H(s^*) = r^* \xor t^*$.
+  This isn't much of a fix, because this value is evidently uniformly
+  distributed and independent of everything except $r^* \xor t^*$.
+  
+  We could rig $G$ and $H'$ too, once we knew $x^*$, so that $s^* = (x^* \xor
+  G(r)) \cat H'(x^* \cat r)$.  But we don't.  Instead, consider the event
+  $F_1$ that the adversary queries $G$ at $r^*$.  Since we have opted for the
+  bare-faced lie rather than oracle subterfuge, the target ciphertext $y^*$
+  and the oracles are entirely independent of the target plaintext $x^*$,
+  whatever that might be.  Hence, $A$'s probability of guessing which
+  plaintext was chosen, $\Pr[S_1] = \frac{1}{2}$.
+
+  When attempting to bound $\Pr[F_1]$, there are two cases to consider:
+  \begin{enumerate}
+
+  \item The adversary has not previously queried $H$ at $s^*$.  In this
+    case, the value of $r^*$ is independent of the adversary's view -- it has
+    no information at all about $r^*$.  This can occur, therefore, with
+    probability at most $q_G 2^{-k}$.
+
+  \item The adversary has previously queried $H$ at $s^*$.  Then we can
+    construct an inverter.  Note that $f_P$ is a permutation (by assumption);
+    hence, selecting a $y^*$ at random implies the values of $w^*$, and hence
+    $s^*$ and $t^*$, even though they can't be computed easily.
+    \begin{program}
+      Inverter $I(P, y)$: \+ \\
+        $\Xid{G}{map} \gets \emptyset$;
+          $\Xid{H}{map} \gets \emptyset$;
+          $\Xid{H'}{map} \gets \emptyset$; \\
+        $z \gets \bot$; \\
+        $(x_0, x_1, s) \gets A^{G(\cdot), H(\cdot), H'(\cdot)}
+          (\cookie{find}, P)$; \\
+        $b \gets A^{G(\cdot), H(\cdot), H'(\cdot)}(\cookie{guess}, y, s)$; \\
+        \RETURN $z$; \- \\[\smallskipamount]
+      Oracle $H(s)$: \+ \\
+        \IF $s \in \dom \Xid{H}{map}$ \\
+        \THEN \RETURN $\Xid{H}{map}(s)$; \\
+        $h \getsr \{0, 1\}^k$; \\
+        $\Xid{H}{map} \gets \Xid{H}{map} \cup \{s \mapsto h\}$; \\
+        \RETURN $h$;
+    \next
+      Oracle $H'(s)$: \+ \\
+        \IF $s \in \dom \Xid{H'}{map}$ \\
+        \THEN \RETURN $\Xid{H'}{map}(s)$; \\
+        $h' \getsr \{0, 1\}^k$; \\
+        $\Xid{H'}{map} \gets \Xid{H'}{map} \cup \{s \mapsto h'\}$; \\
+        \RETURN $h$; \- \\[\smallskipamount]
+      Oracle $G(r)$: \+ \\
+        \IF $r \in \dom \Xid{G}{map}$ \THEN \\
+        \RETURN $\Xid{G}{map}(r)$; \\
+        \FOR $s \in \dom \Xid{H}{map}$ \DO \\ \quad \= \+ \kill
+          $t \gets r \xor \Xid{H}{map}(s);$ \\
+          $w \gets s \cat t$; \\
+          \IF $f_P(w) = y$ \THEN $z \gets w$; \- \\
+        $g \getsr \{0, 1\}^{n-2k}$; \\
+        $\Xid{G}{map} \gets \Xid{G}{map} \cup \{r \mapsto g\}$; \\
+        \RETURN $g$;
+    \end{program}
+    Clearly, $I$ succeeds whenever $A$ queries both $H$ at $s^*$ and $G$ at
+    $r^*$.  $I$'s running time is about $O(q_G q_H)$ because of all of the
+    searching in the $G$ oracle.
+
+  \end{enumerate}
+  Thus,
+  \[ \Pr[F_1] \le
+       \frac{q_G}{2^k} + \InSec{owf}(\mathcal{T}; t + O(q_G q_H)). \]%
+  Now observe that, if $F_1$ doesn't occur, Games $\G0$ and~$\G1$ are
+  indistinguishable.  So
+  \[ \Pr[S_0 \land \lnot F_1] = \Pr[S_1 \land \lnot F_1], \]
+  so by Shoup's lemma,
+  \[ |{\Pr[S_0]} - \Pr[S_1]| \le \Pr[F_1]. \]
+  Rearranging yields:
+  \[ \Adv{ind-cpa}{\Xid{\mathcal{E}}{OAEP+}^{\mathcal{T}}}(A)
+     \le 2 \left (\frac{q_G}{2^k} +
+                  \InSec{owf}(\mathcal{T}; t + O(q_G q_H)) \right). \]%
+  But $A$ was arbitrary.  The result follows.
+
+  To complete the plaintext-awareness proof, we must present a plaintext
+  extractor.
+  \begin{program}
+    Plaintext extractor $X(y, P, Q_G, Q_H, Q_{H'}, C)$: \+ \\
+      \FOR $z \in \dom Q_{H'}$ \DO \\ \quad \= \+ \kill
+        \PARSE $z$ \AS $x, k\colon r$; \\
+        $c \gets Q_{H'}(z)$; \\
+        $s \gets (x \xor Q_G(r)) \cat c$; \\
+        \IF $s \in \dom Q_H$ \THEN \\ \quad \= \+ \kill
+          $t \gets r \xor Q_H(s)$; \\
+          $w \gets s \cat t$; \\
+          $y' \gets f_P(w)$; \\
+          \IF $y = y'$ \THEN \RETURN $x$; \-\- \\
+      \RETURN $\bot$;
+  \end{program}
+  To obtain a lower bound on the success probability of $X$, consider an
+  adversary $A$ which queries its various oracles and returns a ciphertext
+  $y$.  We aim to show that, if $A$ did not query all of its oracles as
+  required for $X$ to work then its probability of producing a valid
+  ciphertext (i.e., one for which the decryption algorithm does not return
+  $\bot$) is small.
+
+  We consider the decryption algorithm applied to $y$, and analyse the
+  probability that $y$ is a valid ciphertext even though $A$ did not make all
+  of the appropriate oracle queries.
+  
+  We shall take some of our notation from the decryption algorithm, which
+  proceeds as follows:
+  \begin{program}
+    Algorithm $\Xid{D}{OAEP+}^
+                {\mathcal{T}, G(\cdot), H(\cdot), H'(\cdot)}_K(y)$: \+ \\
+      $w \gets T_K(y)$; \\
+      \PARSE $w$ \AS $s, k\colon t$; \\
+      $r \gets t \xor H(s)$; \\
+      \PARSE $s$ \AS $s' \cat k\colon c$; \\
+      $x \gets s' \xor G(r)$; \\
+      \IF $c = H'(x \cat r)$ \THEN \RETURN $x$; \\
+      \ELSE \RETURN $\bot$;
+  \end{program}
+  
+  Firstly, suppose that $A$ did not query $H'$ at $x \cat r$.  Then obviously
+  there is only a $2^{-k}$ probability that $c$ is correct and the ciphertext
+  will be accepted.  By our assumption about adversary behaviour, if $A$
+  didn't query $G$ at $r$ then it also didn't query $H'$ at $x \cat r$ for
+  any $x$.  Hence, a decryption algorithm which rejected immediately if it
+  discovered that $G$ hadn't been queried at $r$, or that $H'$ hadn't been
+  queried at $x \cat r$, would be incorrect with probability at most
+  $2^{-k}$.
+
+  Secondly, suppose that $A$ did not query $H$ at $s$.  Then $c$ is still
+  fixed, but $r$ is now random and independent of $A$'s view.  The
+  probability that $G$ has been queried at $r$ is then at most $q_G 2^{-k}$.
+  So a decryption algorithm which, in addition to the changes above, also
+  rejected if it discovered that $H$ had not been queried at $s$ would be
+  incorrect with probability at most $q_G 2^{-k}$.
+
+  But our plaintext extractor $X$ is just such a decryption algorithm.
+  Hence, $X$'s failure probability is
+  \[ \epsilon = \frac{q_G + 1}{2^k}. \]
+\end{proof}
+
+\endinput
+
+%%% Local Variables: 
+%%% mode: latex
+%%% TeX-master: "ips"
+%%% End: 
diff --git a/enc-symm.tex b/enc-symm.tex
new file mode 100644 (file)
index 0000000..e0f3f01
--- /dev/null
@@ -0,0 +1,1188 @@
+\xcalways\section{Symmetric encryption}\x
+
+\xcalways\subsection{Syntax}\x
+
+\begin{slide}
+  \head{Symmetric encryption syntax}
+  
+  A \emph{symmetric encryption scheme} $\mathcal{E} = (E, D)$ is a set of
+  keys $\keys\mathcal{E}$ (usually $\{0, 1\}^k$ for some integer $k$) and
+  pair of algorithms:
+  \begin{itemize}
+  \item an \emph{encryption} algorithm $E\colon \keys\mathcal{E} \times \{0,
+    1\}^* \to \{0, 1\}^*$; and
+  \item a \emph{decryption} algorithm $D\colon \keys\mathcal{E} \times \{0,
+    1\}^* \to \{0, 1\}^*$.
+  \end{itemize}
+  We also have a \emph{correctness} requirement: for any $K \in
+  \keys\mathcal{E}$, and any plaintext $x$, if $E(K, x)$ returns $y$ then
+  $D(K, y)$ returns $x$.
+
+  We write $E_K(\cdot)$ rather than $E(K, \cdot)$, and $D_K(\cdot)$ rather
+  than $D(K, \cdot)$.
+\end{slide}
+
+\xcalways\subsection{Security notions}\x
+
+\begin{slide}
+  \head{Symmetric encryption security notions}
+  
+  In symmetric scheme, an adversary who doesn't know the key can't encrypt
+  data for itself.  To modify our security notions by providing the adversary
+  with an encryption oracle.
+  
+  We consider semantic security, indistinguishability and non-malleability,
+  against chosen-plaintext and chosen-ciphertext attacks.  To make life more
+  complicated, we have three different indistinguishability notions.
+
+  We use the same notation to decscribe the decryption oracles provided in
+  various types of attacks:
+  \begin{tabular}[C]{l Mc Mc }
+                                \hlx*{hv}
+    Attack & D_0(c) & D_1(c) \\ \hlx{vhv}
+    CPA    & \bot   & \bot   \\  
+    CCA1   & D_K(c) & \bot   \\  
+    CCA2   & D_K(c) & D_K(c) \\ \hlx*{vh}
+  \end{tabular}
+\end{slide}
+
+\begin{slide}
+  \topic{semantic security}
+  \head{Semantic security}
+
+  The semantic security game is as for the asymmetric case, except for the
+  presence of encryption oracles.
+
+  \begin{program}
+    Experiment $\Expt{sem-\id{atk}-$b$}{\mathcal{E}}(A)$: \+ \\
+      $K \getsr \keys\mathcal{E}$; \\
+      $(\mathcal{M}, s) \gets A^{E_K(\cdot), D_0(\cdot)}
+        (\cookie{select})$; \\
+      $x_0 \getsr \mathcal{M}$; $x_1 \getsr \mathcal{M}$; \\
+      $y \gets E_K(x_1)$; \\
+      $(f, \alpha) \gets A^{E_K(\cdot), D_1(\cdot)}
+        (\cookie{predict}, y, s)$; \\
+      \IF $f(x_b) = \alpha$ \THEN \RETURN $1$; \\
+      \ELSE \RETURN $0$;
+  \end{program}
+  The distribution $\mathcal{M}$ must be \emph{valid}: all strings in
+  $\supp\mathcal{M}$ must have the same length.
+\end{slide}
+
+\begin{slide}
+  \topic{find-then-guess}
+  \head{Find-then-guess indistinguishability}
+  
+  The `find-then-guess' (FTG) game corresponds to the standard public-key
+  indistinguishability notion.
+  \begin{program}
+    Experiment $\Expt{ftg-\id{atk}-$b$}{\mathcal{E}}(A)$: \+ \\
+      $K \getsr \keys\mathcal{E}$; \\
+      $(x_0, x_1, s) \gets A^{E_K(\cdot), D_0(\cdot)}(\cookie{find})$; \\
+      \IF $|x_0| \ne |x_1|$ \THEN \RETURN $0$; \\
+      $y \gets E_K(x_b)$; \\
+      $b' \gets A^{E_K(\cdot), D_1(\cdot)}(\cookie{guess}, y, s)$; \\
+      \RETURN $b'$;
+  \end{program}
+\end{slide}
+
+\begin{slide}
+  \topic{left-or-right}
+  \head{Left-or-right indistinguishability}
+  
+  The `left-or-right' (LOR) notion is a natural extension of find-then-guess.
+  Rather than having to guess the hidden bit using a single challenge
+  ciphertext, the adversary is allowed to request more as it likes.  It is
+  given an encryption oracle which accepts two arguments: it selects either
+  the left or the right plaintext, according to the hidden bit, and returns
+  the ciphertext.
+  \begin{program}
+    Experiment $\Expt{lor-\id{atk}-$b$}{\mathcal{E}}(A)$; \+ \\
+      $K \getsr \keys\mathcal{E}$; \\
+      $b' \gets A^{E_K(\id{lr}_b(\cdot, \cdot)), D_1(\cdot)}$; \\
+      \RETURN $b'$; \- \\[\smallskipamount]
+    Function $\id{lr}_b(x_0, x_1)$: \+ \\
+      \RETURN $x_b$;
+  \end{program}
+  Note that, because the adversary only runs in one stage, we can only
+  consider chosen-plaintext and adaptive chosen-ciphertext attacks.
+\end{slide}
+
+\begin{slide}
+  \topic{real-or-random}
+  \head{Real-or-random indistinguishability}
+
+  The `real-or-random' (ROR) notion is somewhat less natural, but turns out
+  to be useful for analysing block cipher encryption modes.
+
+  The adversary is given an oracle which, when given a plaintext $x$, either
+  returns an encryption of $x$ or an encryption of a random string with the
+  same length as $x$.
+  \begin{program}
+    Experiment $\Expt{ror-\id{atk}-$0$}{\mathcal{E}}(A)$; \+ \\
+      $K \getsr \keys\mathcal{E}$; \\
+      $b' \gets A^{E_K(\id{rand}(\cdot)), D_1(\cdot)}$; \\
+      \RETURN $b'$; \- \\[\smallskipamount]
+    Function $\id{rand}(x)$: \+ \\
+      $x' \getsr \{0, 1\}^{|x|}$; \\
+      \RETURN $x'$;
+  \next
+    Experiment $\Expt{ror-\id{atk}-$1$}{\mathcal{E}}(A)$; \+ \\
+      $K \getsr \keys\mathcal{E}$; \\
+      $b' \gets A^{E_K(\cdot), D_1(\cdot)}$; \\
+      \RETURN $b'$;
+  \end{program}
+  Again, only chosen-plaintext and adaptive chosen-ciphertext attacks are
+  applicable.
+\end{slide}
+
+\begin{slide}
+  \topic{relations between notions}
+  \head{Relations between notions \cite{Bellare:2000:CST}}
+
+  \[ \xymatrix @=2cm {
+    \txt{LOR} \ar@<0.5ex>[r]^1 \ar@<-0.5ex>[d]_3 &
+    \txt{ROR} \ar@<0.5ex>[l]^2 \\
+    \txt{FTG} \ar@{-->}@<-0.5ex>[u]_4 \ar@<0.5ex>[r] &
+    \txt{SEM} \ar@<0.5ex>[l]
+  } \]
+  \begin{list}{}{
+      \settowidth{\labelwidth}{\textbf{Key}}
+      \leftmargin\labelwidth\advance\leftmargin\labelsep
+      \itemindent0pt\let\makelabel\textbf}
+  \item[Key] \begin{itemize}
+    \item A solid arrow $\xy\ar*+{A};<1.5cm, 0cm>*+{B}\endxy$ indicates an
+      \emph{security-preserving} reduction: if a scheme is secure in notion
+      $A$ then it is as secure in notion $B$, to within a small constant
+      factor.
+    \item A broken arrow $\xy\ar@{-->}*+{A};<1.5cm, 0cm>*+{B}\endxy$
+      indicates a non-security-preserving reduction.
+    \item The numbers refer to sections of the proof provided in the notes.
+    \end{itemize}
+  \end{list}  
+\end{slide}
+
+\begin{proof}
+  We deal with the propositions one at a time.  Most of them are pretty easy,
+  with the exception of the security-losing reduction from FTG to LOR.
+  \begin{enumerate}
+
+  \item We show that $\text{LOR-\id{atk}} \implies \text{ROR-\id{atk}}$.
+    Suppose $A'$ attacks $\mathcal{E}$ in the real-or-random sense.
+    \begin{program}
+      Adversary $A^{E(\cdot, \cdot), D(\cdot)}$: \+ \\
+        \RETURN $A'^{\id{ror-hack}(\cdot), D(\cdot)}$; \-\\[\smallskipamount]
+      Oracle $\id{ror-hack}(x)$: \+ \\
+        $x' \getsr \{0, 1\}^{|x|}$; \\
+        \RETURN $E(x', x)$;
+    \end{program}
+    Since this provides a perfect simulation of the ROR game,
+    \[ \Adv{lor-\id{atk}}{\mathcal{E}}(A) =
+         \Adv{ror-\id{atk}}{\mathcal{E}}(A'), \]%
+    and hence
+    \[ \InSec{ror-\id{atk}}(\mathcal{E}; t, q_E, q_D) \le
+         \InSec{lor-\id{atk}}(\mathcal{E}; t, q_E, q_D). \]%
+
+  \item We show that $\text{ROR-\id{atk}} \implies \text{LOR-\id{atk}}$.
+    Suppose $A'$ attacks $\mathcal{E}$ in the left-or-right sense.
+    \begin{program}
+      Adversary $A^{E(\cdot), D(\cdot)}$: \+ \\
+        $\hat{b} \gets \{0, 1\}$; \\
+        $b' \gets A'^{\id{lor-hack}(\cdot, \cdot), D(\cdot)}$; \\
+        \IF $b' = \hat{b}$ \THEN \RETURN $1$; \\
+        \ELSE \RETURN $0$; \- \\[\smallskipamount]
+      Oracle $\id{lor-hack}(x_0, x_1)$: \+ \\
+        \RETURN $E(x_{\hat{b}})$;
+    \end{program}
+    If the ROR oracle is returning correct encryptions, then $A'$ will return
+    the correct bit $\hat{b}$ with probability
+    \[ \frac{\Adv{lor-\id{atk}}{\mathcal{E}}(A')}{2} + \frac{1}{2}; \]
+    if the ROR oracle is returning ciphertexts for random plaintexts, then
+    $A'$ is being given no information about $\hat{b}$, and hence guesses
+    correctly with probability exactly $\frac{1}{2}$.  We conclude that
+    \[ \Adv{ror-\id{atk}}{\mathcal{E}}(A) =
+         \frac{1}{2}\Adv{lor-\id{atk}}{\mathcal{E}}(A'), \]%
+    and hence
+    \[ \InSec{lor-\id{atk}}(\mathcal{E}; t, q_E, q_D) \le
+         2 \cdot \InSec{ror-\id{atk}}(\mathcal{E}; t, q_E, q_D). \]%
+
+  \item We show that $\text{LOR-\id{atk}} \implies \text{FTG-\id{atk}}$.
+    Suppose $A'$ attacks $\mathcal{E}$ in the find-then-guess sense.  We
+    assume, without loss of generality, that $A'$ never queries its
+    decryption oracle on ciphertexts it obtained from its encryption oracle.
+    \begin{program}
+      Adversary $A^{E(\cdot, \cdot), D(\cdot)}$: \+ \\
+        $(x_0, x_1, s) \gets A'^{\id{encrypt}(\cdot), D(\cdot)}
+          (\cookie{find})$; \\
+        $y \gets E(x_0, x_1)$; \\
+        $b' \gets A'^{\id{encrypt}(\cdot), D(\cdot)}
+          (\cookie{guess}, y, s);$ \\
+        \RETURN $b'$; \- \\[\smallskipamount]
+      Oracle $\id{encrypt}(x)$: \+ \\
+        \RETURN $E(x, x)$;
+    \end{program}
+    Since this provides a perfect simulation of the FTG game,
+    \[ \Adv{lor-\id{atk}}{\mathcal{E}}(A) =
+         \Adv{ftg-\id{atk}}{\mathcal{E}}(A'), \]%
+    and hence
+    \[ \InSec{ftg-\id{atk}}(\mathcal{E}; t, q_E, q_D) \le
+         \InSec{lor-\id{atk}}(\mathcal{E}; t, q_E + 1, q_D). \]%
+    
+  \item We show that $\text{FTG-\id{atk}} \implies \text{LOR-\id{atk}}$,
+    though with a factor of $q_E$ loss of security.
+
+    This proof is slightly more tricky than the others.  Consider the
+    following `hybrid' games, defined for $0 \le i \le q_E$:
+    \begin{program}
+      Experiment $\Expt{hyb-$i$-\id{atk}-$b$}{\mathcal{E}}$: \+ \\
+        $K \getsr \keys\mathcal{E}$; \\
+        $j \gets 0$; \\
+        $b' \gets A^{E(\id{lr-hack}_b(\cdot, \cdot)), D_1(\cdot)}$; \\
+        \RETURN $b'$; \- \\[\smallskipamount]
+    \next
+      Function $\id{lr-hack}_b(x_0, x_1)$: \+ \\
+        \IF $j < i$ \THEN $x \gets x_0$; \\
+        \ELSE \IF $j > i$ \THEN $x \gets x_1$; \\
+        \ELSE $x \gets x_b$; \\
+        $j \gets j + 1$; \\
+        \RETURN $E_K(x_b)$;
+    \end{program}
+    As usual, we define
+    \[ \Adv{hyb-$i$-\id{atk}}{\mathcal{E}}(A) =
+          \Pr[\Expt{hyb-$i$-\id{atk}-$1$}{\mathcal{E}} = 1] -
+          \Pr[\Expt{hyb-$i$-\id{atk}-$0$}{\mathcal{E}} = 1]. \]%
+
+    Observe that we have the identities
+    \begin{eqnarray*}[rl]
+       \Expt{lor-\id{atk}-$0$}{\mathcal{E}} &\equiv
+       \Expt{hyb-$(q_E{-}1)$-\id{atk}-$0$}{\mathcal{E}}
+       \\
+       \Expt{lor-\id{atk}-$1$}{\mathcal{E}} &\equiv
+       \Expt{hyb-$0$-\id{atk}-$1$}{\mathcal{E}}
+       \\
+    \tabpause{and, for $0 \le i < q_E$,}
+       \Expt{hyb-$i$-\id{atk}-$0$}{\mathcal{E}} &\equiv
+       \Expt{hyb-$(i{+}1)$-\id{atk}-$1$}{\mathcal{E}}.
+    \end{eqnarray*}
+    Thus,
+    \begin{eqnarray*}[rclclc]
+       \Adv{lor-\id{atk}}{\mathcal{E}}(A)
+       &=& \Pr[\Expt{lor-\id{atk}-$1$}{\mathcal{E}}(A) = 1] &-&
+           \Pr[\Expt{lor-\id{atk}-$0$}{\mathcal{E}}(A) = 1]
+       \\*
+       &=& \Pr[\Expt{hyb-$0$-\id{atk}-$1$}{\mathcal{E}}(A) = 1] &-&
+           \Pr[\Expt{hyb-$(q_E{-}1)$-\id{atk}-$0$}{\mathcal{E}}(A) = 1]
+       \\
+       &=& \Pr[\Expt{hyb-$0$-\id{atk}-$1$}{\mathcal{E}}(A) = 1] &-&
+           \Pr[\Expt{hyb-$0$-\id{atk}-$0$}{\mathcal{E}}(A) = 1] &+ \\*
+       & & \Pr[\Expt{hyb-$1$-\id{atk}-$1$}{\mathcal{E}}(A) = 1] &-&
+           \Pr[\Expt{hyb-$1$-\id{atk}-$0$}{\mathcal{E}}(A) = 1] &+ \\*
+       & & \multicolumn{1}{c}{\smash\vdots} &-&
+           \multicolumn{1}{c}{\smash\vdots} &+ \\*
+       & & \Pr[\Expt{hyb-$(q_E{-}1)$-\id{atk}-$1$}{\mathcal{E}}(A) = 1] &-&
+           \Pr[\Expt{hyb-$(q_E{-}1)$-\id{atk}-$0$}{\mathcal{E}}(A) = 1]
+       \\*
+       &=& \sum_{0\le i<q_E} \Adv{hyb-$i$-\id{atk}}{\mathcal{E}}(A)
+     \end{eqnarray*}
+     Now, there must be at least one $i$ for which
+     \[ \Adv{hyb-$i$-\id{atk}}{\mathcal{E}}(A) \ge
+        \frac{1}{q_E} \Adv{lor-\id{atk}}{\mathcal{E}}(A). \]%
+     
+     Suppose that $A'$ is an adversary attacking $\mathcal{E}$ in the LOR
+     sense.  We can construct an FTG adversary $A$ for which
+     \[ \Adv{ftg-\id{atk}}{\mathcal{E}}(A) \ge
+        \frac{1}{q_E} \Adv{lor-\id{atk}}{\mathcal{E}}(A') \]%
+     as follows:\footnote{%
+       The expression of the FTG adversary requires control flow operations
+       which aren't easily expressed in the pseudocode language we've used so
+       far, hence the cop-out into English.}
+     \begin{enumerate}
+     \item When invoked in the $\cookie{find}$ stage, run the LOR adversary,
+       passing it $A$'s decryption oracle.
+     \item Respond to its first $i$ left-or-right queries $(x_0, x_1)$ with
+       $E(x_0)$.
+     \item On the $(i + 1)$-th left-or-right query, $(x_0, x_1)$, package up
+       all of $A'$'s state, and return that, together with the pair $(x_0,
+       x_1)$ as the result of $A$'s $\cookie{find}$ stage.
+     \item When reinvoked in the $\cookie{guess}$ stage, return the challenge
+       ciphertext $y$ as the result of $A'$'s $(i + 1)$-th query.
+     \item Respond to the remaining $q_E - i - 1$ left-or-right queries
+       $(x_0, x_1)$ with $E(x_1)$.
+     \item Return the bit output by $A'$.
+     \end{enumerate}
+     This evidently simulates the environment of
+     $\Expt{hyb-$i$-\id{atk}-$b$}{\mathcal{E}}(A')$; hence $A$ achieves the
+     claimed advantage.  Thus,
+     \[ \InSec{lor-\id{atk}}(\mathcal{E}; t, q_E, q_D) \le
+        q_E \cdot \InSec{ftg-\id{atk}}(\mathcal{E}; t, q_E - 1, q_D). \]%
+     
+     Now we show that we can't obtain a better reduction.  Suppose that
+     $\mathcal{E} = (E, D)$ is $(t, q_E, q_D, \epsilon)$-secure in the FTG
+     sense.  We contruct a \emph{$p$-leaky} version, $\mathcal{E}' = (E',
+     D')$.  Let $\id{maybe}(p)$ denote a function which returns $1$ with
+     probability $p$.
+     \begin{program}
+       Algorithm $E'_K(x)$: \+ \\
+         \IF $\id{maybe}(p) = 1$ \THEN \RETURN $1 \cat x$; \\
+         \RETURN $0 \cat E_K(x)$;
+     \next
+       Algorithm $D'_K(y')$: \+ \\
+         \PARSE $y'$ \AS $1\colon b, y$; \\
+         \IF $b = 1$ \THEN \RETURN $y$; \\
+         \RETURN $D_K(y)$;
+     \end{program}
+     A simple simulation argument shows that this scheme is still secure,
+     except for an additional term $p$, handling the case where the challenge
+     ciphertext $y^* = 1 \cat x^*$.  It is easy to see that
+     \[ \InSec{lor-\id{atk}}(\mathcal{E}'; t, q_E, 0) \ge q_E p, \]
+     concluding the proof.
+  \end{enumerate}
+     
+  The proofs that $\text{FTG-\id{atk}} \implies \text{SEM-\id{atk}}$ and
+  $\text{SEM-\id{atk}} \implies \text{FTG-\id{atk}}$ are just as in the
+  public-key case (page~\pageref{pf:pub-ind-eq-sem}), except for the presence
+  of encryption oracles (which are passed on unmolested).  And that's all we
+  need.
+\end{proof}
+
+\begin{exercise}
+  Consider the following `ciphertext-or-random-string' security notion.
+  \begin{program}
+    Experiment $\Expt{cor-\id{atk}-$0$}{\mathcal{E}}(A)$: \+ \\
+      $b' \gets A^{\id{rand}(E_K(\cdot)), D_1(\cdot)}$; \\
+      \RETURN $b'$; \- \\[\smallskipamount]
+    Function $\id{rand}(x)$: \+ \\
+      $x' \getsr \{0, 1\}^{|x|}$; \\
+      \RETURN $x'$;
+  \next
+    Experiment $\Expt{cor-\id{atk}-$1$}{\mathcal{E}}(A)$; \+ \\
+      $K \getsr \keys\mathcal{E}$; \\
+      $b' \gets A^{E_K(\cdot), D_1(\cdot)}$; \\
+      \RETURN $b'$;
+  \end{program}
+  Relate this notion to the others we've already seen.
+  \answer%
+  It's not hard to see that $\text{COR} \implies \text{LOR}$; the proof is
+  similar to $\text{ROR} \implies \text{LOR}$, and we have
+  $\InSec{cor-\id{atk}}(\mathcal{E}; t, q_E, q_D) \le 2\cdot
+  \InSec{lor-\id{atk}}(\mathcal{E}; t, q_E, q_D)$.  On the other hand,
+  $\text{LOR} \not\implies \text{COR}$.  To see this, let $\mathcal{E} = (E,
+  D)$ be an encryption scheme secure in the LOR-\id{atk} sense, and define
+  $\mathcal{E}' = (E', D')$ by $E'_K(x) = 0^n \cat E_K(x)$ and $D'_K(y') =
+  D_K(y)$ if $y' = 0^n \cat y$ for some $y$, or $\bot$ otherwise.  Because
+  the fixed padding is independent of the plaintext,
+  $\InSec{lor-\id{atk}}(\mathcal{E}'; t, q_E, q_D) \le
+  \InSec{lor-\id{atk}}(\mathcal{E}; t, q_E, q_D)$.  But $\mathcal{E}'$ is not
+  COR-CPA secure because an adversary can check for the fixed padding; hence
+  $\InSec{cor-cpa}(\mathcal{E}'; t, q_E, q_D) \ge 1 - q_E 2^{-n}$.
+\end{exercise}
+
+\begin{exercise}
+  Let $F\colon \{0, 1\}^k \times \{0, 1\}^l \to \{0, 1\}^l$ be a PRF, and let
+  $g\colon \{0, 1\}^l \to \{0, 1\}^{2l}$ be a length-doubling PRG.  Recall
+  from Exercise~\ref{ex:dbl-prg} the construction $g^(i)$, defined by
+  \[ g^{(1)}(x) = g(x); \qquad
+     g^{(i+1)}(x) = g_0(x) \cat g^{(i)}(g_1(x)). \]%
+  We define an encryption scheme $\mathcal{E} = (E, D)$ as follows:
+  \begin{program}
+    Algorithm $E_K(x)$: \\
+      $i \getsr \{0, 1\}^l$; \\
+      $n \gets \bigl\lceil \frac{|x|}{L} \bigr\rceil - 1$; \\
+      $s \gets F_K(i)$; \\
+      $p \gets g^{(n)}(s)$; \\
+      $y \gets i \cat (x \xor p)$; \\
+      \RETURN $y$;
+  \next
+    Algorithm $D_K(y)$: \\
+      \PARSE $y$ \AS $l\colon i, y'$; \\
+      $n \gets \bigl\lceil \frac{|x|}{L} \bigr\rceil - 1$; \\
+      $s \gets F_K(i)$; \\
+      $p \gets g^{(n)}(s)$; \\
+      $x \gets y' \xor p$; \\
+      \RETURN $x$;
+  \end{program}
+  Prove that
+  \[ \InSec{lor-cpa}(\mathcal{E}; t, q, \mu) \le
+     2 \cdot \InSec{prf}(F; t, q) +
+     2 q \mu \cdot \InSec{prg}(g; t) + q(q - 1), \]%
+  where $\mu$ is the maximum value of $n$, as computed by $E_K(\cdot)$, for
+  any encryption query.
+  Hints:
+  \begin{parenum}
+  \item use a sequence of games, ending with one in which the `ciphertext'
+    are random strings of the right length;
+  \item attack the PRF first;
+  \item use a hybrid argument to attack the PRG, as was used in the proof
+    that $\text{FTG} \implies \text{LOR}$.
+  \end{parenum}
+  \answer%
+  For each game~$\G{i}$, $S_i$~is the event that the adversary guesses
+  correctly.  Game~$\G0$ is the original attack game (with the hidden bit~$b$
+  selected uniformly).  Game~$\G1$ is the same, except that if, for any pair
+  of ciphertexts, the $i$-values are equal, the game ends immediately: the
+  standard collision bound shows that $|{\Pr[S_1]} - \Pr[S_0]| \le q(q -
+  1)/2$.  In game~$\G2$, rather than using $F_K$ to compute the seeds~$s$, we
+  just choose $s \in \{0, 1\}^l$ at random each time.  Note that the
+  $i$-values are distinct; hence considering an adversary attacking $F$ as a
+  PRF, which simulates either $\G1$ or $\G2$ depending on whether its oracle
+  is an instance of~$F$ or a random function respectively, shows that
+  $|{\Pr[S_2]} - \Pr[S_1]| \le \InSec{prf}(F; t, q)$.
+  
+  In game~$\G3$, rather than using the PRG~$g^{(n)}$, we generate the strings
+  $p$ uniformly at random from $\{0, 1\}^{l(n+1)}$, and claim that
+  $|{\Pr[S_3]} - \Pr[S_2]| \le q \mu \cdot \InSec{prg}(g; t)$ (proven below).
+  Finally, in game~$\G4$, rather than computing the ciphertext as $i \cat (x
+  \xor p)$, we just generate a random string $\{0, 1\}^{l(n+2)}$.  Since $i$
+  and $p$ are uniform and random anyway, this doesn't affect the
+  distribution; it does show that the result is independent of the
+  adversary's ciphertext, however, so $\Pr[S_4] = \Pr[S_3] = \frac{1}{2}$.
+  Tying all of this together, $(\Adv{lor-cpa}{\mathcal{E}}(A) + 1)/2 \le
+  \frac{1}{2} + \InSec{prf}(F; t, q) + q\mu \cdot \InSec{prg}(g; t) + q(q -
+  1)/2$.  Multiplying through by~2 and rearranging yields the required
+  result.
+
+  \def\H#1{\G[H]{#1}}%
+  We finally turn to the claim made earlier.  In $\G2$, we use the PRG; in
+  $\G3$ we don't.  We construct a number of hybrid games~$\H{i}$ for $0 \le i
+  \le q$ in which encryption query~$j$ (for $0 \le j < q$) is handled as
+  follows: if $0 \le j < i$ then the query is handled as in $\G3$; if $i \le
+  j < q$ then the query is handed as in $\G2$.  Let $T_i$ be the event that
+  the adversary wins in game $\H{i}$.  Clearly, $\H0 \equiv \G2$, and $\H{q}
+  \equiv \G3$.  For each adjacent pair of hybrid games $\H{i}, \H{i+1}$ (for
+  $0 \le i < q$), we can bound $|{\Pr[T_{i+1}} - \Pr[T_i]|$ by considering an
+  adversary attacking~$g^{(n)}$ by running~$A$ and using its input as the XOR
+  mask~$p$ for query~$i$, and following the rules of game~$\H{i}$ for the
+  other queries: then if $y$~is random, it simulates $\H{i+1}$, whereas if
+  $y$ is the output of $g^{(n)}$ then it simulates $\H{i}$.  Thus
+  $|{\Pr[T_{i+1}} - \Pr[T_i]| \le \mu \cdot \InSec{prg}(g; t)$ (by the answer
+  to \ref{ex:dbl-prg}), and $|{\Pr[S_3]} - \Pr[S_2]| = |{\Pr[T_{q-1}]} -
+  \Pr[T_0]| \le q \mu \cdot \InSec{prg}(g; t)$ as claimed.
+\end{exercise}
+
+\xcalways\subsection{Block cipher modes}\x
+
+\begin{slide}
+  \head{Block cipher modes}
+
+  Block ciphers (which we model as PRPs) are readily available components,
+  and we have good tools for analysing their (heuristic) security.  It'd be
+  good if we could use them to construct secure encryption schemes.
+  
+  We analyse three standard \emph{modes of operation}:
+  \begin{description}
+  \item[Electronic Code Book (ECB)] Each plaintext block is encrypted
+    independently of the others, using the block cipher.
+  \item[Counter (CTR)] Choose a random starting point $i$.  The plaintext
+    blocks are XORed with the result of encrypting the counter values $i$,
+    $i + 1$, \ldots
+  \item[Ciphertext Block Chaining (CBC)] The first plaintext block is XORed
+    with a random \emph{initialization vector} and encrypted using the block
+    cipher; thereafter, each plaintext block are XORed with the previous
+    ciphertext block and then encrypted with the block cipher.
+  \end{description}
+\end{slide}
+
+\begin{slide}
+  \head{General notation}
+  
+  We consider pseudorandom permutations $E\colon \{0, 1\}^k \times \{0, 1\}^l
+  \to \{0, 1\}^l$ operating on $l$-bit blocks.  We write $E_K(\cdot)$ rather
+  than $E(K, \cdot)$.
+  
+  For the sake of simplicity, we assume that plaintexts are a multiple of $l$
+  bits in length.  We shall consider chosen-plaintext attacks, and we shall
+  be quantifying our results in terms of:
+  \begin{itemize}
+  \item the running time $t$ of adversaries;
+  \item the number $q$ of queries made to the encryption oracle; and
+  \item the maximum size in bits $\mu$ of any individual encryption query.
+  \end{itemize}
+  
+  We shall write `\FOREACH $l\colon z$ \FROM $x$ \DO \ldots' to denote
+  iteration over each $l$-bit block $z$ of $x$ in turn.
+  
+  We use $\emptystring$ to denote the empty string.
+\end{slide}
+
+\begin{slide}
+  \topic{ECB}
+  \head{Electronic Code Book (ECB), 1: description}
+  
+  We define the scheme $\Xid{\mathcal{E}}{ECB}^E = (\Xid{E}{ECB}^E,
+  \Xid{D}{ECB}^E))$ by setting $\keys\Xid{\mathcal{E}}{ECB}^E = \{0, 1\}^k$
+  and
+  \begin{program}
+    Algorithm $\Xid{E}{ECB}^E_K(x)$: \+ \\
+      $y \gets \emptystring$; \\
+      \FOREACH $l\colon z$ \FROM $x$ \DO \\
+      \quad $y \gets y \cat E_K(z)$; \\
+      \RETURN $y$;
+  \next
+    Algorithm $\Xid{D}{ECB}^E_K(y)$: \+ \\
+      $x \gets \emptystring$; \\
+      \FOREACH $l\colon z$ \FROM $y$ \DO \\
+      \quad $x \gets x \cat E_K^{-1}(z)$; \\
+      \RETURN $x$;
+  \end{program}
+\end{slide}
+
+\begin{slide}
+  \head{Electronic Code Book (ECB), 2: analysis}
+  
+  ECB fails to disguise equality of message blocks.  Hence, it is insecure in
+  the left-or-right sense.
+  \begin{program}
+    Adversary $A^{E(\cdot, \cdot)}$: \+ \\
+      $y \gets E(0^l \cat 1^l, 0^l \cat 0^l)$; \\
+      \PARSE $y$ \AS $l\colon y_0, l\colon y_1$; \\
+      \IF $y_0 = y_1$ \THEN \RETURN $1$; \\
+      \ELSE \RETURN $0$;
+  \end{program}
+  Since $\Xid{\mathcal{E}}{ECB}^E$ always encrypts blocks independently, and
+  the block cipher $E$ is deterministic, $A$ always succeeds.  Hence,
+  \[ \InSec{lor-cpa}(\Xid{\mathcal{E}}{ECB}^E; t, 1, 2 l) = 1 \]
+  for some small $t$ describing the running-time of the adversary $A$.
+  
+  According to our formal definitions, then, ECB mode is \emph{completely
+  insecure}.
+\end{slide}
+
+\begin{slide}
+  \topic{stateful counter mode}
+  \head{Counter (CTR), 1: a stateful mode}
+  
+  We define two schemes.  Firstly, a stateful-sender scheme
+  $\Xid{\mathcal{E}}{CTRS}^E = (\Xid{E}{CTRS}^E, \Xid{D}{CTRS}^E))$.  We set
+  $\keys\Xid{\mathcal{E}}{ECB}^E = \{0, 1\}^k$, initialize $i \gets 0$, and
+  define
+  \begin{program}
+    Algorithm $\Xid{E}{CTRS}^E_K(x)$: \+ \\
+      $y \gets i$; \\
+      \FOREACH $l\colon z$ \FROM $x$ \DO \\ \quad \= \+ \kill
+        $y \gets y \cat (z \xor E_K(i))$; \\
+        $i \gets i + 1$; \- \\
+      \RETURN $y$;
+  \next
+    Algorithm $\Xid{D}{CTRS}^E_K(y)$: \+ \\
+      \PARSE $y$ \AS $l\colon i, y$; \\
+      $x \gets \emptystring$; \\
+      \FOREACH $l\colon z$ \FROM $y$ \DO \\ \quad \= \+ \kill
+        $x \gets x \cat (z \xor E_K(i))$; \\
+        $i \gets i + 1$; \- \\
+      \RETURN $x$;
+  \end{program}
+\end{slide}
+
+\begin{slide}
+  \head{Counter (CTR), 2: analysis of the stateful version}
+  
+  We write $q' = q\mu/l$ for the total number of blocks queried by the
+  adversary, and we restrict our attention to the case $n \le 2^l$.
+  
+  Firstly, suppose that, rather than a block cipher, we use a completely
+  random function $R \in \Func{l}{l}$.  Then $E(0) \cat E(1) \cat \cdots$ is
+  a string of uniformly distributed and independent bits.  Hence
+  \[ \InSec{lor-cpa}(\Xid{\mathcal{E}}{CTRS}^{\Func{l}{l}}; t, q, \mu) = 0 \]
+  for arbitrary $t$, and for $q \mu/l \le 2^l$.
+  
+  A simple reduction shows that, for a pseudorandom function $F$, we have
+  \[ \InSec{ror-cpa}(\Xid{\mathcal{E}}{CTRS}^F; t, q, \mu) \le
+     \InSec{prf}(F; t, q'), \]%
+  and hence, for a pseudorandom permutation $E$,
+  \[ \InSec{ror-cpa}(\Xid{\mathcal{E}}{CTRS}^E; t, q, \mu) \le
+     \InSec{prp}(E; t, q') + \frac{q'(q' - 1)}{2^{l+1}}. \]%
+\end{slide}
+
+\begin{exercise}
+  Fill in the gaps in the above proof.
+  \answer%
+  The reduction from the PRF distinguisher to the counter-with-PRF scheme
+  works as follows.  Let $A$ attack $\Xid{\mathcal{E}}{CTRS}^F$ in the ROR
+  sense; consider adversary $B^{F(\cdot)}$: \{ $b \gets
+  A^{\Xid{E}{CTRS}^F(\cdot)}$; \RETURN $b$;~\}.  If $F(\cdot)$ is an instance
+  of the PRF then $B$ encrypts messages chosen by $A$ faithfully; if
+  $F(\cdot)$ is a random function then the ciphertexts $B$ returns consists
+  of a counter followed by a random string, which is therefore distributed
+  identically to a ciphertext of a \emph{random} plaintext.  Thus, $B$
+  simulates the real-or-random game perfectly.  The result for a PRP follows
+  because $\InSec{prf}(F; t, q) \le \InSec{prp}(F; t, q) + q(q - 1)
+  2^{-L-1}$.
+\end{exercise}
+
+\begin{slide}
+  \topic{randomized counter mode}
+  \head{Counter (CTR), 3: a randomized mode}
+  
+  The randomized scheme $\Xid{\mathcal{E}}{CTR$\$$}^E = (\Xid{E}{CTR$\$$}^E,
+  \Xid{D}{CTR$\$$}^E))$ differs from the stateful scheme in the encryption
+  algorithm only.  We simply choose the starting value for the counter at
+  random, rather than remembering it.
+  \begin{program}
+    Algorithm $\Xid{E}{CTR$\$$}^E_K(x)$: \+ \\
+      $i \getsr \{0, 1\}^l$; \\
+      $y \gets i$; \\
+      \FOREACH $l\colon z$ \FROM $x$ \DO \\ \quad \= \+ \kill
+        $y \gets y \cat (z \xor E_K(i))$; \\
+        $i \gets i + 1$; \- \\
+      \RETURN $y$;
+  \next
+    Algorithm $\Xid{D}{CTR$\$$}^E_K(y)$: \+ \\
+      \PARSE $y$ \AS $l\colon i, y$; \\
+      $x \gets \emptystring$; \\
+      \FOREACH $l\colon z$ \FROM $y$ \DO \\ \quad \= \+ \kill
+        $x \gets x \cat (z \xor E_K(i))$; \\
+        $i \gets i + 1$; \- \\
+      \RETURN $x$;
+  \end{program}
+\end{slide}
+
+\begin{slide}
+  \head{Counter (CTR), 4: analysis of the randomized version}
+
+  The randomized mode remains secure so long as a counter is never repeated.
+  This occurs with probability no greater than
+  \[ \frac{q\mu(q - 1)}{2^{l+1}}. \]
+  Hence, we have, for a pseudorandom function $F$,
+  \[ \InSec{ror-cpa}(\Xid{\mathcal{E}}{CTR$\$$}^F; t, q, \mu) \le
+     \InSec{prf}(F; t, q') + \frac{q\mu(q - 1)}{2^{l+1}}, \]%
+  and, for a pseudorandom permutation $E$,
+  \[ \InSec{ror-cpa}(\Xid{\mathcal{E}}{CTR$\$$}^E; t, q, \mu) \le
+     \InSec{prp}(E; t, q') + \frac{q'(q' - 1 + l(q - 1))}{2^{l+1}}. \]%
+\end{slide}
+
+\begin{proof}[Proof of the collision bound]
+  Suppose all of the queries are maximum length.  Then the probability that
+  two randomly started counter sequences overlap is $\mu\cdot 2^{-l}$.
+  Hence, an upper bound on the collision probability is given by
+  \begin{eqnarray*}[rl]
+    \Pr[\text{no collision}] &\le \frac{\mu}{2^l}(1 + 2 + \cdots + q - 1) \\
+                             &= \frac{\mu}{2^l} \frac{q(q - 1)}{2} \\
+                             &= \frac{q\mu(q - 1)}{2^{l+1}}
+  \end{eqnarray*}
+  as required.
+\end{proof}
+
+\begin{slide}
+  \topic{CBC}
+  \head{Ciphertext Block Chaining (CBC), 1: description}
+
+  We define the scheme $\Xid{\mathcal{E}}{CBC}^E = (\Xid{E}{CBC}^E,
+  \Xid{D}{CBC}^E))$ by setting $\keys\Xid{\mathcal{E}}{CBC}^E = \{0, 1\}^k$
+  and
+  \begin{program}
+    Algorithm $\Xid{E}{CBC}^E_K(x)$: \+ \\
+      $i \getsr \{0, 1\}^l$; \\
+      $y \gets i$; \\
+      \FOREACH $l\colon z$ \FROM $x$ \DO \\ \quad \= \+ \kill
+        $i \gets E_K(z \xor i)$; \\
+        $y \gets y \cat i$; \- \\
+      \RETURN $y$;
+  \next
+    Algorithm $\Xid{D}{CBC}^E_K(y)$: \+ \\
+      \PARSE $y$ \AS $l\colon i, y$; \\
+      $x \gets \emptystring$; \\
+      \FOREACH $l\colon z$ \FROM $y$ \DO \\ \quad \= \+ \kill
+        $x \gets x \cat (i \xor E_K^{-1}(z))$; \\
+        $i \gets z$; \- \\
+      \RETURN $x$;
+  \end{program}
+\end{slide}
+
+\begin{slide}
+  \head{Ciphertext Block Chaining (CBC), 2: analysis}
+
+  As before, we set $q' = q\mu/l$ as the number of blocks queried by an
+  adversary attacking the encryption scheme.
+
+  As if by magic,\footnote{%
+    The proof of this result is omitted.  The interested reader is directed
+    towards \cite{Bellare:2000:CST}.} %
+  we have the result
+  \[ \InSec{ror-cpa}(\Xid{\mathcal{E}}{CBC}^E; t, q, \mu) \le
+     \frac{q'(q' - 1)}{2^l}. \]%
+\end{slide}
+
+\begin{slide}
+  \topic{requirement for random IVs in CBC mode}
+  \head{Ciphertext Block Chaining (CBC), 3: on randomness of IVs}
+  
+  The initialization vector used in CBC encryption must be \emph{a priori}
+  unpredictable to the adversary.  Suppose that $P(i)$, given an IV for a
+  ciphertext, can predict the IV which will be used with the next ciphertext
+  with probability $\epsilon$.  Then we construct this adversary, attacking
+  $\Xid{\mathcal{E}}{CBC}$ in the ROR-CPA sense:
+  \begin{program}
+    Adversary $A^{E(\cdot)}$: \+ \\
+      $y \gets E(0^l)$; \PARSE $y$ \AS $l\colon i, z$; \\
+      $j \gets P(y)$; $y' \gets E(j)$; \PARSE $y'$ \AS $l\colon i', z'$; \\
+      \IF $i' = j \land y = y'$ \THEN \RETURN $1$; \\
+      \ELSE \RETURN $0$;
+  \end{program}
+  The adversary succeeds when it guesses the IV correctly, \emph{except} when
+  the random encryption oracle happens to choose the same plaintext as we
+  wanted to encrypt anyway.  So, therefore,
+  \[ \Adv{ror-cpa}{\Xid{\mathcal{E}}{CBC}^E} \ge \epsilon - 2^{-l}. \]
+\end{slide}
+
+\xcalways\subsection{Chosen-ciphertext security for symmetric encryption}\x
+
+\begin{exercise}
+  Show that CTR and CBC modes are not secure against adaptive
+  chosen-ciphertext attacks.
+  \answer%
+  We use the FTG-CCA2 notion.  For CTR mode: \cookie{find}: \RETURN $(0, 1,
+  \bot)$; \cookie{guess}: $y' \gets D(y \xor 0^L1)$; \RETURN $y' \xor 1$;
+  For CBC mode: same find stage, $y' \gets D(y \xor 1)$; \RETURN $y' \xor 1$;
+\end{exercise}
+  
+\begin{slide}
+  \topic{integrity of ciphertexts}
+  \head{Integrity of ciphertexts \cite{Bellare:2000:AER}}
+
+  Informally, we say that an encryption scheme $\mathcal{E} = (E, D)$ has
+  \emph{integrity of ciphertexts} (whose confusing short name is INT-CTXT) if
+  it's hard for an adversary equipped with an encryption oracle to come with
+  a new \emph{valid} ciphertext, i.e., one for which the decryption function
+  $D_K$ does not return the symbol $\bot$.
+
+  We shall see later that integrity of ciphertexts \emph{and}
+  indistinguishability under chosen-plaintext attacks together imply
+  chosen-ciphertext security.  This is intuitively clear, but it's worth
+  proving anyway.
+\end{slide}
+
+\begin{slide}
+  \head{Integrity of ciphertexts (cont.)}
+  
+  Consider the following game played by an adversary $A$:
+  \begin{program}
+    Experiment $\Expt{int-ctxt}{\mathcal{E}}(A)$: \+ \\
+      $K \getsr \keys\mathcal{E}$; $\Xid{y}{list} \gets \emptyset$;
+      $y \gets A^{\id{encrypt}(\cdot), D_K(\cdot)}$; \\
+      \IF $y \notin \Xid{y}{list} \land D_K(y) \ne \bot$
+      \THEN \RETURN $1$; \\
+      \ELSE \RETURN $0$;
+  \next
+    Oracle $\id{encrypt}(x)$: \+ \\
+      $y \gets E_K(x)$; \\
+      $\Xid{y}{list} \gets \Xid{y}{list} \cup \{y\}$; \\
+      \RETURN $y$;
+  \end{program}
+  We define $A$'s success probability in this game by
+  \[ \Succ{int-ctxt}{\mathcal{E}}(A) =
+     \Pr[\Expt{int-ctxt}{\mathcal{E}}(A) = 1] \]%
+  and write that
+  \[ \InSec{int-ctxt}(\mathcal{E}; t, q_E, q_D) =
+     \max_A \Succ{int-ctxt}{\mathcal{E}}(A), \]%
+  where the maximum is over all adversaries running in time $t$ and issuing
+  $q_E$ encryption and $q_D$ decryption queries.
+\end{slide}
+
+\begin{slide}
+  \topic{INT-CTXT and LOR-CPA imply LOR-CCA}
+  \head{INT-CTXT and LOR-CPA together imply LOR-CCA}
+  
+  We now prove the claim made earlier.  Suppose that the adversary $A$
+  attacks $\mathcal{E}$ in the LOR-CCA sense.  We consider these two
+  adversaries, attacking the chosen-plaintext security and ciphertext
+  integrity of $\mathcal{E}$ respectively.
+  \begin{program}
+    Adversary $B^{E(\cdot, \cdot)}$: \+ \\
+      $b \gets A^{E(\cdot, \cdot), \Xid{D}{sim}(\cdot)}$; \\
+      \RETURN $b$; \- \\[\smallskipamount]
+    Oracle $\Xid{D}{sim}(y)$; \+ \\
+      \RETURN $\bot$;
+  \next
+    Adversary $C^{E(\cdot), D(\cdot)}$: \+ \\
+      $b \getsr \{0, 1\}$; $y^* \gets \bot$; \\
+      $b' \gets A^{E(\id{lr}_b(\cdot, \cdot)), \Xid{D}{sim}(\cdot)}$; \\
+      \RETURN $y^*$; \- \\[\smallskipamount]
+    Function $\id{lr}_b(x_0, x_1)$: \+ \\
+      \RETURN $x_b$; \- \\[\smallskipamount]
+    Oracle $\Xid{D}{sim}(y)$: \+ \\
+      $x \gets D(y)$; \\
+      \IF $x \ne \bot$ \THEN $y^* \gets y$; \\
+      \RETURN $x$;
+  \end{program}
+\end{slide}
+
+\begin{slide}
+  \head{INT-CTXT and LOR-CPA together imply LOR-CCA2 (cont.)}
+  
+  We analyse the advantage of $B$, attacking $\mathcal{E}$ in the LOR-CPA
+  sense.  Obviously, $B$ is lying through its teeth in its simulation of
+  $A$'s decryption oracle.  If in fact all of $A$'s decryption queries were
+  for invalid ciphertexts, $B$ can't notice.  So let $V$ be the event that at
+  least one of $A$'s ciphertexts was valid.  Then
+  \[ \Adv{lor-cpa}{\mathcal{E}}(A) \ge
+       \Adv{lor-cca}{\mathcal{E}}(A) - 2\Pr[V]. \]%
+  To bound $\Pr[V]$, we consider adversary $C$, which simply records any of
+  $A$'s decryption queries which returns a valid ciphertext.  Since $A$ is
+  forbidden from passing any ciphertexts obtained from its encryption oracle
+  to its decryption oracle, $C$'s returned ciphertext $y^*$ is not one it
+  obtained from \emph{its} encryption oracle.  So
+  \[ \Succ{int-ctxt}{\mathcal{E}}(A) = \Pr[V]. \]
+  Concluding, then,
+  \[ \InSec{lor-cca}(\mathcal{E}; t, q_E, q_D) \le
+       \InSec{lor-cpa}(\mathcal{E}; t, q_E) +
+       2 \cdot \InSec{int-ctxt}(\mathcal{E}; t, q_E, q_D). \]%
+\end{slide}
+
+\begin{slide}
+  \topic{strong MACs provide INT-CTXT}
+  \head{A strong MAC provides integrity of ciphertexts}
+
+  That's a very nice result, but how do we acheive INT-CTXT?  Well, the
+  game in the definition looks very much like the forgery games we played
+  when we were thinking about MACs.
+  
+  Suppose that $\mathcal{E} = (E, D)$ is an encryption scheme secure in the
+  LOR-CPA sense, and $\mathcal{M} = (T, V)$ is a strong MAC (in the SUF-CMA
+  sense).  Then we can define $\Xid{\mathcal{E}}{auth}^{\mathcal{M}} =
+  (\Xid{E}{auth}^{\mathcal{E}, \mathcal{M}}, \Xid{D}{auth}^{\mathcal{E},
+    \mathcal{M}})$ by
+  \[ \keys\Xid{\mathcal{E}}{auth}^{\mathcal{E}, \mathcal{M}} =
+     \keys\mathcal{E} \times \keys\mathcal{M} \]%
+  and
+  \begin{program}
+    Algorithm
+    $\Xid{E}{auth}^{\mathcal{E}, \mathcal{M}}_{K_E, K_T}(x)$: \+ \\
+      $y \gets E_{K_E}(x)$; \\
+      $\tau \gets T_{K_T}(y)$; \\
+      \RETURN $\tau \cat y$;
+  \next
+    Algorithm
+    $\Xid{D}{auth}^{\mathcal{E}, \mathcal{M}}_{K_E, K_T}(y')$: \+ \\
+      \PARSE $y'$ \AS $\tau, y$; \\
+      \IF $V_{K_T}(y, \tau) = 0$ \THEN \RETURN $\bot$; \\
+      \ELSE \RETURN $D_{K_E}(y)$;
+  \end{program}
+\end{slide}
+
+\begin{slide}
+  \head{A strong MAC provides integrity of ciphertexts (cont.)}
+
+  The security proof for $\Xid{\mathcal{E}}{auth}^{\mathcal{E}, \mathcal{M}}$
+  is left as a trivial exercise.  We end up with the result that
+  \[ \InSec{int-ctxt}(\Xid{\mathcal{E}}{auth}^{\mathcal{E}, \mathcal{M}};
+                     t, q_E, q_D) \le
+    \InSec{suf-cma}(\mathcal{M}; t, q_E, q_D) \]%
+  and hence
+  \begin{eqnarray*}[Ll]
+    \InSec{lor-cca}(\Xid{\mathcal{E}}{auth}^{\mathcal{E}, \mathcal{M}};
+                     t, q_E, q_D) \\
+    & \le \InSec{lor-cpa}(\mathcal{E}; t, q_E) +
+          2 \cdot \InSec{suf-cma}(\mathcal{M}; t, q_E, q_D).
+  \end{eqnarray*}
+  A MAC, therefore, can help us to attain a strong notion of secrecy, even if
+  no actual integrity appears to be required.  This is an important lesson.
+\end{slide}
+
+\begin{exercise}
+  Prove the above result.
+  \answer%
+  Let $A$ attack INT-CTXT.  Construct adversary $B^{T(\cdot), V(\cdot)}$: \{
+  $K \getsr \keys\mathcal{E}$; $(y, \tau) \gets A^{\id{encrypt}(\cdot),
+  \id{decrypt}(\cdot)}$; \RETURN $(y, \tau)$;~\} Oracle $\id{encrypt}(x)$:
+  \{ $y \gets E_K(x)$; $\tau \gets T(y)$; \RETURN $(y, \tau)$;~\} Oracle
+  $\id{decrypt}(y, \tau)$: \{ \IF $V(y, \tau) = 1$ \THEN \RETURN $D_K(y)$;
+  \ELSE \RETURN $\bot$;~\}.  The simulation of the INT-CTXT game is perfect.
+\end{exercise}
+
+\begin{slide}
+  \topic{mixing encryption and MACs}
+  \head{Notes on mixing encryption and MACs}
+
+  To construct $\Xid{\mathcal{E}}{auth}^{\mathcal{E}, \mathcal{M}}$, we
+  applied a MAC to the \emph{ciphertext}.  This isn't perhaps the most
+  intuitive way to combine an encryption scheme with a MAC.
+
+  There are three constructions which look plausible.
+  \begin{description}
+  \item[Encrypt-then-MAC:]
+    %
+    $y \gets E_{K_E}(x)$; $\tau \gets T_{K_T}(y)$; \RETURN $\tau \cat y$;
+    \\
+    Encrypt the plaintext, and MAC the ciphertext; used in IPsec and nCipher
+    Impath; we've proven its generic security, using the notion of integrity
+    of ciphertexts.
+    %
+  \item[MAC-then-encrypt:]
+    %
+    $\tau \gets T_{K_T}(x)$; $y \gets E_{K_E}(\tau \cat x)$; \RETURN $y$;
+    \\
+    MAC the plaintext, and encrypt both the plaintext and tag; used in SSL
+    and TLS; not \emph{generically} secure against chosen-ciphertext attacks.
+    %
+  \item[Encrypt-and-MAC:]
+    %
+    $y \gets E_{K_E}(x)$; $\tau \gets T_{K_T}(x)$; \RETURN $\tau \cat y$;
+    \\
+    Separately MAC and encrypt the plaintext; used in SSH; \emph{never}
+    secure against chosen-ciphertext, not generically secure against
+    chosen-plaintext!
+  \end{description}
+\end{slide}
+
+\begin{proof}
+  We begin with a few words on our approach, before we embark on the proof
+  proper.
+  
+  To demonstrate the generic insecurity of a scheme, we assume the existance
+  of an encryption scheme and MAC (since if they don't exist, the result is
+  vacuously true) and construct modified schemes whose individual security
+  relates tightly to the originals, but the combined scheme is weak.
+  
+  We demonstrate \emph{universal} insecurity by showing an attack which works
+  given \emph{any} component encryption and MAC schemes.
+  
+  We prove security relationships using the LOR-CPA notion because this is
+  strongest, and bounds for other notions can be derived readily from the
+  left-or-right analysis.  We prove insecurity using the FTG-CCA or FTG-CPA
+  notions, because they are weakest and show the strength of our results
+  best.
+  
+  We've dealt with the generic security of encrypt-then-MAC already.  We turn
+  our attention first first to the generic insecurity of the MAC-then-encrypt
+  scheme.
+
+  Let $\mathcal{E} = (E, D)$ be a symmetric encryption scheme, and let
+  $\mathcal{M} = (T, V)$ be a MAC.  We define the MAC-then-encrypt scheme
+  $\Xid{\mathcal{E}}{MtE}^{\mathcal{E}, \mathcal{M}} =
+  (\Xid{E}{MtE}^{\mathcal{E}, \mathcal{M}}, \Xid{D}{MtE}^{\mathcal{E},
+    \mathcal{M}})$ as follows:
+  \[ \keys\Xid{\mathcal{E}}{MtE}^{\mathcal{E}, \mathcal{M}} =
+     \keys\mathcal{E} \times \keys\mathcal{M} \]%
+  and
+  \begin{program}
+    Algorithm
+    $\Xid{E}{MtE}^{\mathcal{E}, \mathcal{M}}_{K_E, K_T}(x)$: \+ \\
+      $\tau \gets T_{K_T}(x)$; \\
+      $\RETURN E_{K_E}(\tau \cat x)$;
+  \next
+    Algorithm
+    $\Xid{D}{MtE}^{\mathcal{E}, \mathcal{M}}_{K_E, K_T}(y)$: \+ \\
+      $x' \gets D_{K_E}(y)$; \\
+      \PARSE $x'$ \AS $\tau, x$; \\
+      \IF $V_{K_T}(x, \tau) = 0$ \THEN \RETURN $\bot$; \\
+      \ELSE \RETURN $x$;
+  \end{program}
+  We construct a new encryption scheme $\mathcal{E}' = (E', D')$ in terms of
+  $\mathcal{E}$, such that the combined scheme
+  $\Xid{\mathcal{E}}{MtE}^{\mathcal{E}', \mathcal{M}}$ is insecure in the
+  FTG-CCA sense.  Our modified encryption scheme has $\keys\mathcal{E}' =
+  \keys\mathcal{E}$, and works as follows:
+  \begin{program}
+    Algorithm $E'_K(x)$: \+ \\
+      \RETURN $0 \cat E_K(x)$;
+  \next
+    Algorithm $D'_K(y')$: \+ \\
+      \PARSE $y'$ \AS $1\colon b, y$; \\
+      \RETURN $D_K(y)$;
+  \end{program}
+  That is, the encryption scheme prepends a single bit to the ciphertext, and
+  doesn't check its value during decryption.  Intuitively, this makes the
+  scheme malleable: we can change the ciphertext by flipping the first bit,
+  but the MAC tag remains valid because the plaintext is unaffected.
+  
+  Firstly, we prove that $\mathcal{E}'$ is LOR-CPA if $\mathcal{E}$ is.
+  Suppose $A'$ attacks $\mathcal{E}'$ in the LOR-CPA sense: then
+  \begin{program}
+    Adversary $A^{E(\cdot, \cdot)}$: \+ \\
+      \RETURN $A'^{0 \cat E(\cdot, \cdot)}$;
+  \end{program}
+  has the same advantage.
+  
+  Secondly, we show that the combined MAC-then-encrypt scheme
+  $\Xid{\mathcal{E}}{MtE}^{\mathcal{E}', \mathcal{M}}$ is insecure in the
+  FTG-CCA sense.  Consider this adversary:
+  \begin{program}
+    Adversary $B^{E(\cdot), D(\cdot)}(\cookie{find})$: \+ \\
+      \RETURN $(0, 1, \bot)$;
+  \next
+    Adversary $B^{E(\cdot), D(\cdot)}(\cookie{guess}, y', s)$: \+ \\
+      \PARSE $y'$ \AS $1\colon b, y$; \\
+      \RETURN $D(1 \cat y)$;
+  \end{program}
+  The ciphertext $1 \cat y$ was never returned by the encryption oracle
+  (because it always returns the first bit zero); but the plaintext of $1
+  \cat y$ is the challenge plaintext.  Hence, $B$ wins always, and
+  \[ \InSec{ftg-cca}(\Xid{\mathcal{E}}{MtE}^{\mathcal{E}', \mathcal{M}};
+                     t, 0, 1) = 1, \]%
+  where $t$ is the running time of the adversary $B$ above.
+  
+  We now address the separate encrypt-and-MAC scheme, which we define
+  formally.  Let $\mathcal{E} = (E, D)$ be a symmetric encryption scheme, and
+  let $\mathcal{M} = (T, V)$ be a MAC.  Then the the encrypt-and-MAC scheme
+  $\Xid{\mathcal{E}}{E\&M}^{\mathcal{E}, \mathcal{M}} =
+  (\Xid{E}{E\&M}^{\mathcal{E}, \mathcal{M}}, \Xid{D}{E\&M}^{\mathcal{E},
+    \mathcal{M}})$ is defined by:
+  \[ \keys\Xid{\mathcal{E}}{E\&M}^{\mathcal{E}, \mathcal{M}} =
+     \keys\mathcal{E} \times \keys\mathcal{M} \]%
+  and
+  \begin{program}
+    Algorithm
+    $\Xid{E}{E\&M}^{\mathcal{E}, \mathcal{M}}_{K_E, K_T}(x)$: \+ \\
+      $y \gets E_{K_E}(x)$; \\
+      $\tau \gets T_{K_T}(x)$; \\
+      $\RETURN \tau \cat y$;
+  \next
+    Algorithm
+    $\Xid{D}{E\&M}^{\mathcal{E}, \mathcal{M}}_{K_E, K_T}(y')$: \+ \\
+      \PARSE $y'$ \AS $\tau, y$; \\
+      $x \gets D_{K_E}(y)$; \\
+      \IF $V_{K_T}(x, \tau) = 0$ \THEN \RETURN $\bot$; \\
+      \ELSE \RETURN $x$;
+  \end{program}
+  
+  We first show that this scheme is \emph{universally} insecure against
+  chosen-ciphertext attack.  Let $\mathcal{E}$ and $\mathcal{M}$ be an
+  arbitrary symmetric encryption scheme and MAC, respectively.  The attack
+  works because the MACs can be detached and used in chosen-ciphertext
+  queries to test for equality of messages.
+  \begin{program}
+    Adversary $B^{E(\cdot), D(\cdot)}(\cookie{find})$: \+ \\
+      \RETURN $(0, 1, \bot)$;
+  \next
+    Adversary $B^{E(\cdot), D(\cdot)}$(\cookie{guess}, y', s): \+ \\
+      $y_1' \gets E(1)$; \\
+      \PARSE $y'$ \AS $\tau, y$; \\
+      \PARSE $y_1'$ \AS $\tau_1, y_1$; \\
+      \IF $\tau = \tau_1 \lor D(\tau \cat y_1) \ne \bot$
+      \THEN \RETURN $1$; \\
+      \ELSE \RETURN $0$;
+  \end{program}
+  After receiving the challenge ciphertext, the adversary requests an
+  additional encryption of the plaintext $1$.  If the tags are equal on the
+  two ciphertexts then we announce that the hidden bit is $1$.  Otherwise, we
+  attempt a decryption of the new ciphertext, using the tag from the
+  challenge.  If it decrypts successfully, we announce that the bit is $1$;
+  otherwise we claim it is zero.
+  
+  Certainly, this strategy is always correct when the hidden bit is indeed
+  $1$.  However, there is a possibility that the MACs are equal or verify
+  correctly even when the hidden bit is $0$.  To bound this probability, we
+  construct the following simple adversary against the MAC:
+  \begin{program}
+    Adversary $B'^{T(\cdot), V(\cdot)}$: \+ \\
+      $\tau \gets T(1)$; \\
+      \RETURN $(0, \tau)$;
+  \end{program}
+  We see readily that
+  \[ \InSec{ftg-cca}(\Xid{\mathcal{E}}{E\&M}^{\mathcal{E}, \mathcal{M}};
+                     t, 1, 1) \ge
+     1 - \InSec{suf-cma}(\mathcal{M}; t', 1, 0), \]%
+  where $t$ and $t'$ are the running times of adversaries $B$ and $B'$
+  respectively.
+  
+  Finally, we show that the encrypt-and-MAC scheme is generically insecure
+  against chosen-plaintext attacks only.  There are two strategies we could
+  use.  Since both offer useful insights into the properties of MACs, we
+  present both here.
+  \begin{itemize}
+    
+  \item \emph{Deterministic MACs.}  In the proof of the universal weakness of
+    the encrypt-and-MAC scheme, we used the check on the MAC to decide on the
+    equality of two plaintexts given the ciphertexts.  If the MAC is
+    deterministic (e.g., a PRF) then we don't need a decryption query.
+
+    Let $\mathcal{E} = (E, D)$ be a symmetric cipher, and let $\mathcal{M} =
+    (T, V)$ be a deterministic MAC, e.g., a PRF, or HMAC.  Then consider this
+    adversary:
+    \begin{program}
+      Adversary $B^{E(\cdot)}(\cookie{find})$: \+ \\
+        \RETURN $(0, 1, \bot)$;
+    \next
+      Adversary $B^{E(\cdot)}(\cookie{guess}, y', s)$: \+ \\
+        $y_1' \gets E(1)$; \\
+        \PARSE $y'$ \AS $\tau, y$; \\
+        \PARSE $y_1'$ \AS $\tau_1, y_1$; \\
+        \IF $\tau = \tau_1$ \THEN \RETURN $1$; \\
+        \ELSE \RETURN $0$;
+    \end{program}
+    Since the MAC is deterministic, the tag attached to a ciphertext $1$ is
+    always the same.  We bound the probability that $T_K(0) = T_K(1)$ using
+    the adversary $B'$ above, and conclude that
+    \[ \InSec{ftg-cpa}(\Xid{\mathcal{E}}{E\&M}^{\mathcal{E}, \mathcal{M}};
+                       t, 1) \ge
+       1 - \InSec{suf-cma}(\mathcal{M}; t', 1, 0), \]%
+    where $t$ and $t'$ are the running times of adversaries $B$ and $B'$
+    respectively.
+      
+  \item \emph{Leaky MACs.}  A MAC doesn't have to conceal information about
+    messages.  Suppose $\mathcal{M} = (T, V)$ is a secure MAC.  We define the
+    leaky MAC $\mathcal{M}' = (T', V')$ by stating that $\keys\mathcal{M}' =
+    \keys\mathcal{M}''$ and
+    \begin{program}
+      Algorithm $T'_K(x)$: \+ \\
+        \PARSE $x$ \AS $1\colon x_0, z$; \\
+        \RETURN $x_0 \cat T_K(x)$;
+    \next
+      Algorithm $V'_K(x, \tau')$: \+ \\
+        \PARSE $\tau'$ \AS $1\colon \tau_0, \tau$; \\
+        \PARSE $x$ \AS $1\colon x_0, z$; \\
+        \IF $x_0 \ne \tau_0$ \THEN \RETURN $0$; \\
+        \ELSE \RETURN $V_K(x, \tau)$;
+    \end{program}
+    We must first prove that $\mathcal{M}'$ remains secure.  To do this,
+    consider an adversary $A'$ attacking $\mathcal{M}'$.  We construct $A$
+    attacking $\mathcal{M}$ in the obvious way:
+    \begin{program}
+      Algorithm $A^{T(\cdot), V(\cdot)}$: \\
+        $(x', \tau') \gets
+          A'^{\Xid{T'}{sim}(\cdot), \Xid{V'}{sim}(\cdot)}$; \\
+        \PARSE $\tau'$ \AS $1\colon \tau_0, \tau$; \\
+        \RETURN $(x, \tau)$;
+    \next
+      Oracle $\Xid{T'}{sim}(x)$: \+ \\
+        \PARSE $x$ \AS $1\colon x_0, z'$; \\
+        \RETURN $x_0 \cat T(x)$; \- \\[\smallskipamount]
+      Oracle $\Xid{V'}{sim}(x, \tau')$: \+ \\
+        \PARSE $\tau'$ \AS $1\colon \tau_0, \tau$; \\
+        \PARSE $x$ \AS $1\colon x_0, z$; \\
+        \IF $x_0 \ne \tau_0$ \THEN \RETURN $0$; \\
+        \ELSE \RETURN $V(x, \tau)$;
+    \end{program}
+    Here, $A$ simply simulates the environment expected by $A'$.  It is clear
+    that $A$ succeeds whenver $A'$ returns a valid tag for a \emph{new}
+    message.  However, suppose that $A'$ returns a new tag $\tau'$ for some
+    old message $x$, for which the tag $\tau$ was returned by the tagging
+    oracle.  Let $x_0$, $\tau_0$ and $\tau'_0$ be the first bits of $x$,
+    $\tau$ and $\tau'$ respectively, and let $\tau^*$ be the remaining bits
+    of $\tau'$.  If the pair $(x, \tau')$ is to be a valid
+    $\mathcal{M}'$-forgery, we must have $x_0 = \tau_0 = \tau'_0$.  Hence,
+    $\tau$ and $\tau'$ must differ in at least one other bit, and $(x,
+    \tau^*)$ is a valid $\mathcal{M}$-forgery.  We conclude that
+    \[ \InSec{suf-cma}(\mathcal{M}'; t, q_T, q_V) \le
+       \InSec{suf-cma}(\mathcal{M}; t, q_T, q_V) \]%
+    as required.
+
+    Now we show that the combined encrypt-and-MAC scheme is weak in the
+    FTG-CPA sense.  Consider this adversary attacking the scheme:
+    \begin{program}
+      Adversary $B^{E(\cdot), D(\cdot)}(\cookie{find})$: \+ \\
+        \RETURN $(0, 1, \bot)$;
+    \next
+      Adversary $B^{E(\cdot), D(\cdot)}$(\cookie{guess}, y', s): \+ \\
+        \PARSE $y'$ \AS $\tau, y$; \\
+        \PARSE $\tau$ \AS $1\colon b, \tau^*$; \\
+        \RETURN $b$;
+    \end{program}
+    The leaky MAC simply tells us the right answer.  So
+    \[ \InSec{ftg-cpa}(\Xid{\mathcal{E}}{E\&M}^{\mathcal{E}, \mathcal{M}'};
+                       t, 0) = 1, \]%
+    where $t$ is the running time of adversary $B$ above.
+
+  \end{itemize}
+
+  This concludes the proof.
+\end{proof}
+
+%% TO DO: Include stuff about integrity-aware encryption modes some day.
+
+\endinput
+
+%%% Local Variables: 
+%%% mode: latex
+%%% TeX-master: "ips"
+%%% End: 
diff --git a/ips.sty b/ips.sty
new file mode 100644 (file)
index 0000000..8375e7b
--- /dev/null
+++ b/ips.sty
@@ -0,0 +1,265 @@
+%%% -*-latex-*-
+%%%
+%%% Definitions for IPS course
+
+%%%----- Required packages --------------------------------------------------
+
+\RequirePackage[palatino, helvetica, courier, maths=cmr]{mdwfonts}
+\RequirePackage{fancyhdr}
+\RequirePackage[T1]{fontenc}
+\RequirePackage{mdwtab, mathenv, crypto, mdwthm, mdwmath}
+\RequirePackage[within=section]{exercise}
+\RequirePackage{amssymb}
+\RequirePackage{url}
+
+\ifx\pdfoutput\@@undefined
+  \def\next{\RequirePackage[all, dvips]{xy}}
+\else
+  \def\next{\RequirePackage[all]{xy}}
+  \ifarticle
+    \pdfpagewidth\paperwidth
+    \pdfpageheight\paperheight
+  \else
+    \pdfpagewidth\paperheight
+    \pdfpageheight\paperwidth
+  \fi
+\fi
+\next
+
+%%%----- Miscellaneous settings ---------------------------------------------
+
+\errorcontextlines=999
+
+%%%----- Style choices ------------------------------------------------------
+
+\renewcommand{\qedsymbol}{$\square$}
+\def\hex#1{\texttt{0x#1}}
+\let\emptystring\lambda
+\let\epsilon\varepsilon
+\let\phi\varphi
+\let\emptyset\varnothing
+\bibliographystyle{mdwalpha}
+\sloppy
+
+%%%----- Exercise typesetting -----------------------------------------------
+
+\exctrcheck{section}{\answrite{\protect\subsection*{Section \thesection}}}
+
+%%%----- Diagram typesetting ------------------------------------------------
+
+\turnradius={4pt}
+
+\def\ocat{
+  \setbox\z@\hbox{$\scriptstyle\|$}%
+  \dimen8\ht\z@\advance\dimen8\dp\z@%
+  \mathbin{\xy *=<\dimen8>[o][F]{\box\z@} \endxy}%
+}
+
+%%%------ Sectioning and headings -------------------------------------------
+
+\ifarticle
+  \fancyhf{}
+  \fancyhead[LE]{\textbf{\nouppercase{\leftmark}}}
+  \fancyhead[RO]{\textbf{\nouppercase{\rightmark}}}
+  \fancyfoot[LE, RO]{\thepage}
+  \def\headrulewidth{\z@}
+  \fancypagestyle{plain}{\fancyhead{}}
+  \def\sectionmark#1{\markboth{\thesection.\ #1}{}}
+  \def\subsectionmark#1{\markright{\thesubsection.\ #1}}
+  \let\emptyslide\relax
+  \let\topic\@gobble
+  \pagestyle{fancy}
+\else
+  \def\sectionmark#1{\markboth{#1}{}}
+  \def\subsectionmark#1{\markright{ -- #1}}
+  \newcount\@emptyslide
+  \newpagestyle{slide}%
+    {\ifnum\@emptyslide=\c@slide\else\textbf{\thetitle}\hfil\fi}%
+    {\ifnum\@emptyslide=\c@slide\else\leftmark\rightmark\hfil\theslide\fi}
+  \slidepagestyle{slide}
+  \def\emptyslide{\global\@emptyslide\c@slide}%
+  \def\@sect#1#2#3#4#5#6[#7]#8{%
+    \ifnum #2>\c@secnumdepth\else%
+      \refstepcounter{#1}%
+    \fi%
+    \csname #1mark\endcsname{#7}%
+    \csname #1summary\endcsname{#7}%
+  }
+  \def\@ssect#1#2#3#4#5{\relax}
+  \newtoks\sm@toks
+  \newcount\sm@count
+  \def\sm@auxwrite#1#2{\immediate\write\@auxout
+    {\noexpand\sm@entry{\the\sm@count}{#1}{#2}}}
+  \def\sectionsummary{%
+    \global\advance\sm@count\@ne%
+    \expandafter\let\expandafter\@next%
+      \csname sm@summary@\the\sm@count\endcsname%
+    \ifx\@next\relax\else\@next\sm@endsummary\fi%
+    \sm@auxwrite{section}%
+  }
+  \def\subsectionsummary{\sm@auxwrite{subsection}}
+  \def\sm@entry#1#2#3{%
+    \begingroup%
+      \let\protect\@unexpandable@protect%
+      \expandafter\ifx\csname sm@summary@#1\endcsname\relax%
+        \expandafter\let\csname sm@summary@#1\endcsname\@empty%
+      \fi%
+      \expandafter\xdef\csname sm@summary@#1\endcsname{%
+        \csname sm@summary@#1\endcsname%
+        \expandafter\protect\csname sm@do#2\endcsname{#3}%
+      }%
+    \endgroup%
+  }
+  \def\sm@dosection#1{%
+    \begingroup\slide%
+    \emptyslide%
+    \noindent\vadjust{}%
+    \vskip10\p@%
+    \centerline{\sffamily\bfseries\Large #1 -- overview}
+    \vskip10\p@%
+    \begingroup\itemize
+  }
+  \def\sm@endsummary{\enditemize\endgroup\endslide\endgroup}
+  \def\sm@dosubsection#1{\item\relax\@tempswatrue#1}
+  \def\sm@dotopic#1{\if@tempswa:\@tempswafalse\else;\fi\space#1}
+  \def\topic{\sm@auxwrite{topic}}
+\fi
+
+\def\head#1{{\sffamily\bfseries\large #1}\par}
+
+%%%------ Page layout for notes ---------------------------------------------
+
+\ifarticle
+  \raggedbottom
+  \def\slide@clearpage{\if@nobreak\else\par\penalty\z@\fi}
+\fi
+
+%%%----- Slide layout -------------------------------------------------------
+
+\slidewidth=260mm
+\slideheight=160mm
+
+\slideplacement{here}
+\slideframewidth=1pt
+\centerslidesfalse
+
+\def\slidefuzz{20pt}
+
+\def\slideleftmargin{15mm} \let\sliderightmargin=\slideleftmargin
+\def\slidetopmargin{15mm} \let\slidebottommargin=\slidetopmargin
+
+\def\slidearraystretch{1}
+
+%%%----- Hacking xcomment ---------------------------------------------------
+%%%
+%%% Supports \xcalways...\x which is an escape from the usual commenting.
+
+\begingroup
+\catcode`\!=12\catcode`\[=12\catcode`\]=12\catcode`\"=12
+\lccode`\!=`\\\lccode`\[=`\{\lccode`\]=`\}\lccode`\"=`\%
+\catcode`\~=\active\lccode`\~=`\^^M
+\lowercase{
+  %%
+  %% Hack the main loop code
+  %%
+  \gdef\xcomment@@@#1\@nil{%
+    \def\@tempa{#1}%
+    \ifx\@tempa\@empty
+      \let\next\xcomment@
+    \else
+      \def\next{\xcomment@@#1\@nil}%
+      \@testtrue
+      \xc@checkbegin#1\relax begin[]\relax\relax
+      \if@test
+        \xc@checkend#1\relax end[]\relax\relax
+        \if@test
+          \xc@checkinput#1\relax input[]\relax\relax
+          \if@test
+            \xc@checkinclude#1\relax include[]\relax\relax
+            \if@test
+              \xc@checkendinput#1\relax endinputss\relax\relax
+              \if@test
+                \xc@checkalways#1\relax xcalwaysfoo!x\relax\relax
+    \fi\fi\fi\fi\fi\fi
+    \next}
+  %%
+  %% New command support
+  %%
+  \gdef\xc@checkalways#1xcalways#2!x#3\relax#4\relax{%
+    \def\@tempa{#1}%
+    \ifx\@tempa\@empty%
+      \@testfalse%
+      \endgroup%
+      \toks@{#2}\rescan\toks@%
+      \def\next{\the\toks@\xc@always#3\@nil}%
+    \fi%
+  }
+}
+\endgroup
+
+\def\xc@always#1\@nil{\xc@begin#1}
+\def\xcalways#1\x{#1}
+
+%%%----- Bugfix -------------------------------------------------------------
+
+\def\@bugfix#1\let\@listi#2#3\@@%
+  {\def\normalsize{#1#3}}
+\expandafter\@bugfix\normalsize\@@
+
+\toks@\expandafter{\slidebox@restore}
+\edef\slidebox@restore{\the\toks@\advance\topsep\parskip}
+
+\ifarticle
+  \def\@bugfix#1\penalty\z@#2\@@{\def\outputloop@savedslides{#1#2}}
+  \expandafter\@bugfix\outputloop@savedslides\@@
+  \toks@\expandafter{\output@savedslide}
+  \def\@strange@hack{\begingroup\output{\unvbox\@cclv}\penalty-\@M\endgroup}
+  \edef\output@savedslide{\noexpand\@strange@hack\the\toks@}
+\fi
+
+\def\@makeslide{%
+  \setbox\@slidebox\vbox{%
+    \@begindvi % added 1997/04/15 SPQR
+    \unvbox\@cclv
+    \ifvoid\slide@footins\else
+      \vskip\skip\slide@footins
+      \vfil
+      \footnoterule
+      \unvbox\slide@footins
+      \unskip
+    \fi
+    \vskip\z@}}
+
+\def\@makecol{%
+   \setbox\@outputbox \box\@cclv
+   \xdef\@freelist{\@freelist\@midlist}%
+   \global \let \@midlist \@empty
+   \@combinefloats
+   \ifvoid\footins \else
+     \setbox\@outputbox \vbox {%
+       \boxmaxdepth \@maxdepth                   
+       \unvbox \@outputbox
+       \vskip \skip\footins
+       \vfil
+       \footnoterule
+       \unvbox \footins
+       \unskip
+       }%
+   \fi
+   \ifvbox\@kludgeins
+     \@makespecialcolbox
+   \else
+     \setbox\@outputbox \vbox to\@colht {%
+       \@texttop
+       \dimen@ \dp\@outputbox
+       \unvbox \@outputbox
+       \vskip -\dimen@
+       \@textbottom
+       }%
+   \fi
+   \global \maxdepth \@maxdepth
+}
+
+%%%----- That's all, folks --------------------------------------------------
+
+\endinput
diff --git a/ips.tex b/ips.tex
new file mode 100644 (file)
index 0000000..8702127
--- /dev/null
+++ b/ips.tex
@@ -0,0 +1,63 @@
+%%% -*-latex-*-
+%%%
+%%% Main document sequencing for IPS course
+
+\ifx\printslides\notdef\documentclass[a4, article, twoside]{seminar}\fi
+\usepackage{ips}
+
+\title{Introduction to Provable Security}
+\author{Mark Wooding}
+
+\begin{document}
+
+\maketitle
+
+\ifarticle\else
+\begin{slide}
+  \emptyslide
+  \noindent\vadjust{}
+  \vfil
+  \centerline{\sffamily\bfseries\huge Introduction to Provable Security}
+  \vfil
+  \leavevmode\hfill{\sffamily\bfseries\large Mark Wooding}
+  \vfil
+\end{slide}
+\fi
+
+\tableofcontents
+
+\include{basics}
+
+\include{auth-mac}
+\include{auth-sig}
+
+\include{enc-intro}
+\include{enc-pub}
+\include{enc-symm}
+\include{enc-ies}
+
+%% \include{zk}
+%% \include{chan}
+
+\section{Answers to exercises}
+\answers
+
+\section{Acknowledgements}
+
+A great deal of the material covered here, particularly the earlier stuff, is
+derived from \cite{Goldwasser:1999:LNC}.  Of course, any errors are mine
+alone.
+
+\ifarticle\bibliography{cryptography,mdw-crypto}\else
+\begin{slide}
+  \head{References} \sectionmark{References}
+  \bibliography{cryptography,mdw-crypto}
+\end{slide}
+\fi
+
+\end{document}
+
+%%% Local Variables: 
+%%% mode: latex
+%%% TeX-master: t
+%%% End: 
diff --git a/mkimages.pl b/mkimages.pl
new file mode 100644 (file)
index 0000000..79a8a5f
--- /dev/null
@@ -0,0 +1,34 @@
+#! /usr/bin/perl
+
+$| = 1;
+$np = 0;
+
+$in = shift or die("bad arg");
+open IN, $in or die("open `$in': $!");
+while (<IN>) {
+  $np = $1, last if /^%%Pages:\s*(\d+)\s*$/;
+}
+die("no pages") unless $np;
+
+sub sel($) { sprintf("tmp.sel.%05d.ps", $_[0]); }
+sub image($) { sprintf("img%05d.png", $_[0]); }
+
+END {
+  for ($i = 1; $i <= $np; $i++) {
+    unlink(sel($i));
+  }
+}
+
+print "converting...";
+for ($p = 1; $p <= $np; $p++) {
+  print " [";
+  system qw"psselect -q -p", $p, $in, sel($p) and die("psselect: $?");
+  print "$p";
+  system qw"convert
+    -density 300 -antialias -geometry 600x800 -rotate -90
+    -bordercolor black -border 0x17",
+    sel($p), image($p) and die("convert: $?");
+  print "]";
+  unlink(sel($p));
+}
+print "\n";
diff --git a/slides.tex b/slides.tex
new file mode 100644 (file)
index 0000000..2773783
--- /dev/null
@@ -0,0 +1,10 @@
+\documentclass[a4, slidesonly]{seminar}
+\let\printslides\relax
+\input{ips}
+
+\endinput
+
+%%% Local Variables: 
+%%% mode: latex
+%%% TeX-master: t
+%%% End: 
diff --git a/zk.tex b/zk.tex
new file mode 100644 (file)
index 0000000..dc2e2f7
--- /dev/null
+++ b/zk.tex
@@ -0,0 +1,13 @@
+\xcalways\section{Interactive proofs and zero knowledge}\x
+
+%%%         * Interactive proofs and zero knowledge: perfect/statistical/
+%%%           computational zero knowledge; proofs of knowledge and
+%%%           identity; non-interactive zero knowledge proofs and digital
+%%%           signatures.
+
+\endinput
+
+%%% Local Variables: 
+%%% mode: latex
+%%% TeX-master: "ips"
+%%% End: